new approaches for evaluation and treatment of appendicitis · ultrasound or ct scan showing...

66
New approaches for evaluation and treatment of appendicitis Loren Berman, MD Pediatric surgeon, Nemours-AI DuPont Hospital for Children ACOP meeting 2017

Upload: others

Post on 02-Feb-2020

1 views

Category:

Documents


0 download

TRANSCRIPT

Page 1: New approaches for evaluation and treatment of appendicitis · Ultrasound or CT scan showing appendicitis with an appendix ≤ 1.1 centimeter thick and no evidence of abscess or fecalith

New approaches for evaluation and treatment of

appendicitis

Loren Berman MD

Pediatric surgeon Nemours-AI DuPont Hospital for Children

ACOP meeting 2017

Disclosures

I have no relevant financial relationships with the manufacturer(s) of any commercial product(s) andor provider(s) of commercial services discussed in this CME activity

I do not intend to discuss an unapprovedinvestigative use of a commercial productdevice in my presentation

Appendicitis scope of the disease

Over 70000 cases annually in children in USA

Lifetime risk of appendicitis 9 for boys 7 for girls

Peak incidence 12 to 18 years

Journal of Pediatric Surgery 2017 52 669-676DOI (101016jjpedsurg201701013)

Copyright copy 2017 Elsevier Inc Terms and Conditions

Burden of Appendectomy Hospital-Acquired Infection

Journal of Pediatric Surgery 2017 52 669-676DOI (101016jjpedsurg201701013)

Copyright copy 2017 Elsevier Inc Terms and Conditions

Burden of Appendectomy Cost Variation

Objectives

Why do children get appendicitis

What are typical presenting features

How do we make the diagnosis

How do we decide WHETHER to operate

How do we decide WHEN to operate

What is the difference in how we approach perforated vs non-perforated appendicitis

Does every child with appendicitis actually need surgery

Why does appendicitis occur

Who is more likely to present with perforated

appendicitis

A Younger patients

B Patients with developmental delay

C Uninsured patients

D Minorities

E All of the above

More likely to present with perforated appendicitis

Perforation rates are reported from 20-80 in children

Younger kids

ndash 82 in children under 5

ndash Nearly 100 of 1-year-olds

Developmental delay

Uninsured

Minorities

Kokoska ER1 Bird TM Robbins JM Smith SD Corsi JM Campbell BT Racial disparities

in the management of pediatric appendicitis J Surg Res 2007 Jan137(1)83-8

Which of the following symptoms is LEAST likely to

be consistent with appendicitis

A Fever

B Nauseavomiting

C Diarrhea

D Peri-umbilical pain

E Anorexia

How do children present with appendicitis

Pain starts peri-umbilical and then migrates to RLQ

Pain starts insidiously is persistent and worsens over time

Pain is worse with movement

Fever nausea with or without vomiting and anorexia

Features suggesting alternative diagnoses

ndash Waxing and waning pain

ndash Diarrhea cough sore throat myalgias rhinorrhea and sick contacts

ndash (BUT there are patients with many of the above symptoms who DO have appendicitis)

Differential diagnosis

Virus

Pneumonia

Constipation

In adolescent females

ndash PID

ndash Ovarian cyst

ndash Mittelschmertz

Approach to physical exam in suspected appendicitis

Child does NOT want to move around

Focal tenderness

Percussion of the abdomen causes discomfort (this is better than rebound tenderness)

Psoas obturator Rovsingrsquos heel strike

Watch them walk around

ldquoDoes this child have appendicitisrdquo Meta-Analysis

Fever

ndash If present LR 34 (24-48)

ndash If absent LR 032 (016-064)

Rebound tenderness

ndash If present LR 30 (23-39)

ndash If absent LR 028 (014-055)

Migration of pain LR range 19-31

RLQ pain LR 12 (10-15)

WBC less than 10 LR 022 (017-030)

ANC less than 6750 LR 006 (003-016) Bundy DG Byerley JS Liles EA Perrin EM Katznelson J Rice HE

Does this child have appendicitis JAMA 2007 Jul 25298(4)438-51

Alvarado A (1986) A practical score for the early diagnosis

of acute appendicitis Ann Emerg Med 15557ndash564

You are seeing a child in your office with 1-day history of

RLQ pain You are suspicious for appendicitis Where would

you send the patient

A Emergency room

B Same day outpatient surgery clinic

C CT scan

D Ultrasound

E Other

What to do with suspected appendicitis

Obtain imaging vs send to ED

ndash If any suspicion for viral syndrome with dehydration to ED

ndash If VERY HIGH suspicion for appendicitis and good pediatric radiologist is available consider sending child for ultrasound

Operating Room

further wu per surgeon

Further workup and imaging in

consultation with

Surgery team

1 IV placed 20 cckg bolus CBC

2 Surgical consult within one hour

Strong suspicion on

HPE Equivocal history

physical exam

1 IV placed 20 cckg bolus CBC

2 Re-examine surgical consult within one

hour if still tender

Abdominal pain

ro appendicitis

What is the best way to image the appendix

A CT scan

B Ultrasound

C MRI

D Abdominal X ray

E Other

Ultrasound

CT scan

MRI

Summary of approach to imaging

Ultrasound is good if you have a high pretest probability good radiologist thin patient

MRI is good if you are in a center that uses it routinely to evaluate children with appendicitis

CT scan is best if above criteria are not met

Alternative to imaging

ndash OBSERVATION

Once diagnosis of appendicitis has been establishedhellip

Non-ruptured

ndash Short duration of symptoms no suggestion of rupture on imaging

ndash Start antibiotics

ndash Perform appendectomy

Ruptured

ndash Have radiology evaluate for drainable fluid collection

ndash Antibiotics

ndash Operation if above management fails

Once diagnosis of appendicitis has been establishedhellip

Non-ruptured

ndash Short duration of symptoms no suggestion of rupture on imaging

ndash Start antibiotics

ndash Perform appendectomy

Ruptured

ndash Have radiology evaluate for drainable fluid collection

ndash Antibiotics

ndash Operation if above management fails

When should appendectomy be performed in a

patient with non-ruptured acute appendicitis

A Immediately after diagnosis

B Within first 24 hours of hospital stay once appropriate antibiotics and IVF resuscitation have been provided

C After completion of a one-week course of antibiotics

D It is reasonable to treat these patients with antibiotics alone and not perform an appendectomy

Is the appendix a ticking time bomb

Traditional teaching appendicitis is an emergency and should be treated with surgery immediately upon diagnosis

Several studies have now shown that delays of 12 hours up to 48 hours do not increase risk of perforation

Current standard admit IVF resuscitation antibiotics appendectomy during daylight hours

Almstroumlm M1 Svensson JF Patkova B Svenningsson A Wester TAnn Surg 2016 Mar 8 In-hospital

Surgical Delay Does Not Increase the Risk for Perforated Appendicitis in Children A Single-center

Retrospective Cohort Study

Port placement

Lap appy with Endo-loops

Lap appy with stapler

Open appendectomy

Appendix with fecalith

Laparoscopic vs open appendectomy

Laparoscopy is standard of care

Lower complication rate

Less scar

Less pain

Ability to evaluate other intra-abdominal organs if the appendix looks normal

ndash Gallbladder

ndash Ovaries

ndash Inguinal canal

Sauerland S Jaschinski T Neugebauer EA Laparoscopic versus open surgery for

suspected appendicitis Cochrane Database Syst Rev 2010 Oct 6(10)

Post-op course

Most patients can go home on the day of surgery

Recovery is usually quick

Back to school within a week sports within 2 weeks

Very low risk surgery with good outcomes

Risk of infection is about 5 at port sites or in abdomen

You see a 7 year old male in your office with 1-day history of abdominal pain

now localized to RLQ with focal guarding and poor appetite You send him for

ultrasound and it shows appendicitis His mother has heard about some new

research and wants to know if you would recommend just treating with

antibiotics rather than surgery How would you counsel this mom

A This data is preliminary and surgery is still standard of

care

B This may be a reasonable option for her son and she

should discuss it with her surgeon

C If it were your son you wouldnrsquot let him have surgery

D Other

Do all patients with appendicitis require surgery

Pilot study at Nationwide Childrenrsquos Hospital

102 patients

ndash 7 to 17 years of age

ndash Uncomplicated appendicitis defined by

Abdominal pain le 48 hours

White blood cell count le 18000

Ultrasound or CT scan showing appendicitis with an appendix le 11 centimeter thick and no

evidence of abscess or fecalith

Patients and families chose to have appendectomy or antibiotics alone

Non-operative management at least 24 hours of in-hospital observation and IV antibiotics until symptoms improved followed by completion of 10 days of treatment with antibiotics by mouth

Minneci PC Mahida JB et al The effectiveness of patient choice in non-operative

versus surgical management of uncomplicated acute appendicitis JAMA Surgery

2015 Dec 16

Results of non-operative management

65 families chose appendectomy 37 families chose non-operative management

Success rate of non-operative management (defined as not undergoing an appendectomy) 89 at 30 days 76 at 1 year

The 24 who failed did NOT have a higher rate of ruptured appendicitis compared to the patients who had immediate appendectomy

1 year follow-up the children managed non-operatively compared with the surgery group had fewer disability days (8 vs 21 days) lower appendicitis-related health care costs (median $4219 vs $5029) and no difference in health-related quality of life

Non-Operative Treatment of Appendicitis Rationale

Appendectomy is invasive

Children may miss up to two weeks of schoolactivities

Caregivers miss work

Postop complications after appendectomy for uncomplicated appendicitis 5-10

Serious complications (reoperations or readmissions) 1-7

Adult data suggest one-year success rates of 63-85 no difference in rates of complicated appendicitis

bull Meta-analysis of 10 articles reporting 413 children receiving non

operative treatment (NOT) for appendicitis ndash all published in past 10

years

bull 5 comparative

bull 1 RCT

bull 4 case series of NOT

Interpreting meta-analyses Forest plot

Favors

antibiotics

Favors

appendectomy

Overall success of NOT

97 during initial episode

Incidence of recurrent appendicitis during follow-up period (range 2-51

months)

Long term efficacy of NOT (no appendectomy at end of follow-up

period)

LOS shortened by about 05 days in those undergoing appendectomy

compared to NOT

Increased rate of complications for appendectomy compared to NOT

Authorsrsquo Conclusions

ldquoCurrent data suggest that NOT is safe It appears effective as initial treatment in 97 of children with AUA and the rate of recurrent appendicitis is 14rdquo

The study highlights the lack of robust evidence comparing NOT with appendectomy in children

Confirms a position of equipoise between treatment approaches

We recommend that NOT of children with AUA be reserved for those participating in carefully designed research studies

Take-home point

Lap appy is still standard of care for uncomplicated appendicitis

Once diagnosis of appendicitis has been establishedhellip

Non-ruptured

ndash Short duration of symptoms no suggestion of rupture on imaging

ndash Start antibiotics

ndash Perform appendectomy

Ruptured

ndash Have radiology evaluate for drainable fluid collection

ndash Antibiotics

ndash Operation in acute setting if above management fails

ndash Interval appendectomy 6-8 weeks laterhellip

ndash ALTERNATIVELY Just take out the appendix

CT ruptured appendix with abscess

After placement of percutaneous drain

How to counsel families of children with ruptured appendicitis

If kids tolerate a diet pain resolves fever resolves oral antibiotics and home

Plan interval appy 6-8 weeks later

Non-operative management may not work and surgery may be needed (Failure rate 20)

At home they should watch for signs of persistentrecurrent appendicitis

High-anxiety time for patients and families

A 10-year-old boy comes to see you in the office after recent

hospitalization for perforated appendicitis He has 3 more days of

antibiotics left His mom is worried because appetite is poor and his

energy level is low On exam he has diffuse lower abdominal

tenderness You recommend

A Extending course of oral antibiotics

B CT scan to evaluate for persistent or

recurrent appendicitis

C CT scan to evaluate for intra-abdominal

abscess

D Follow-up with surgeon

Is interval appendectomy necessary

In adults many surgeons do not do this operation

In kids data are limited

ndash 2-year follow-up of 96 patients

ndash Perforated appendicitis treated non-operatively with antibiotics

ndash 6 became worse 41 had interval appendectomy

ndash 49 received no further treatment

ndash 57 no recurrence

ndash 43 had recurrence within one month to 2 years

ndash Presence of appendicolith 72 rate of recurrence vs 26 in those without appendicolith

Ein SH1 Langer JC Daneman A Nonoperative management of pediatric ruptured appendix

with inflammatory mass or abscess presence of an appendicolith predicts recurrent

appendicitis J Pediatr Surg 2005 Oct40(10)1612-5

Approach to interval appendectomy

More strongly recommended if fecalith present

Observation is a reasonable option

Best choice for an individual patient depends on their anxiety and parental anxiety

Immediate operation for ruptured appendicitis perhaps a better

option

Meta-analysis operative vs non-operative management of pediatric

ruptured appendicitis

2 RCTs identified

Total of 171 pediatric patients

Compared early vs interval appendectomy

Early appendectomy reduced incidence of adverse event

Ruptured appendicitis may have abscess or phlegmon

Early surgery was more strongly favored when there was no abscess at

time of presentation

Decreased antibiotic duration length of stay and total charges for

abscess and no abscess groups

You are seeing a 7 year old with constipation A

fecalith was seen on abdominal X ray during recent

ED visit How would you counsel the family

A They should be referred to surgery for appendectomy

B Their child is at higher risk for appendicitis so they should be aware of this in case he develops symptoms

C It is uncertain whether this child is at higher risk for appendicitis

D A course of Miralax may help wash out the fecalith

Take home points

Uncomplicated appendicitis

ndash Lap appy is still standard of care

ndash Non-operative management may be an acceptable option but not enough is known about long-term risk of recurrent appendicitis

Complicated appendicitis

ndash Can be managed with immediate operation delayed appendectomy or no appendectomy

ndash Immediate operation is probably more efficient and less stressful for patients and parents

Thank you very much

Questions

Page 2: New approaches for evaluation and treatment of appendicitis · Ultrasound or CT scan showing appendicitis with an appendix ≤ 1.1 centimeter thick and no evidence of abscess or fecalith

Disclosures

I have no relevant financial relationships with the manufacturer(s) of any commercial product(s) andor provider(s) of commercial services discussed in this CME activity

I do not intend to discuss an unapprovedinvestigative use of a commercial productdevice in my presentation

Appendicitis scope of the disease

Over 70000 cases annually in children in USA

Lifetime risk of appendicitis 9 for boys 7 for girls

Peak incidence 12 to 18 years

Journal of Pediatric Surgery 2017 52 669-676DOI (101016jjpedsurg201701013)

Copyright copy 2017 Elsevier Inc Terms and Conditions

Burden of Appendectomy Hospital-Acquired Infection

Journal of Pediatric Surgery 2017 52 669-676DOI (101016jjpedsurg201701013)

Copyright copy 2017 Elsevier Inc Terms and Conditions

Burden of Appendectomy Cost Variation

Objectives

Why do children get appendicitis

What are typical presenting features

How do we make the diagnosis

How do we decide WHETHER to operate

How do we decide WHEN to operate

What is the difference in how we approach perforated vs non-perforated appendicitis

Does every child with appendicitis actually need surgery

Why does appendicitis occur

Who is more likely to present with perforated

appendicitis

A Younger patients

B Patients with developmental delay

C Uninsured patients

D Minorities

E All of the above

More likely to present with perforated appendicitis

Perforation rates are reported from 20-80 in children

Younger kids

ndash 82 in children under 5

ndash Nearly 100 of 1-year-olds

Developmental delay

Uninsured

Minorities

Kokoska ER1 Bird TM Robbins JM Smith SD Corsi JM Campbell BT Racial disparities

in the management of pediatric appendicitis J Surg Res 2007 Jan137(1)83-8

Which of the following symptoms is LEAST likely to

be consistent with appendicitis

A Fever

B Nauseavomiting

C Diarrhea

D Peri-umbilical pain

E Anorexia

How do children present with appendicitis

Pain starts peri-umbilical and then migrates to RLQ

Pain starts insidiously is persistent and worsens over time

Pain is worse with movement

Fever nausea with or without vomiting and anorexia

Features suggesting alternative diagnoses

ndash Waxing and waning pain

ndash Diarrhea cough sore throat myalgias rhinorrhea and sick contacts

ndash (BUT there are patients with many of the above symptoms who DO have appendicitis)

Differential diagnosis

Virus

Pneumonia

Constipation

In adolescent females

ndash PID

ndash Ovarian cyst

ndash Mittelschmertz

Approach to physical exam in suspected appendicitis

Child does NOT want to move around

Focal tenderness

Percussion of the abdomen causes discomfort (this is better than rebound tenderness)

Psoas obturator Rovsingrsquos heel strike

Watch them walk around

ldquoDoes this child have appendicitisrdquo Meta-Analysis

Fever

ndash If present LR 34 (24-48)

ndash If absent LR 032 (016-064)

Rebound tenderness

ndash If present LR 30 (23-39)

ndash If absent LR 028 (014-055)

Migration of pain LR range 19-31

RLQ pain LR 12 (10-15)

WBC less than 10 LR 022 (017-030)

ANC less than 6750 LR 006 (003-016) Bundy DG Byerley JS Liles EA Perrin EM Katznelson J Rice HE

Does this child have appendicitis JAMA 2007 Jul 25298(4)438-51

Alvarado A (1986) A practical score for the early diagnosis

of acute appendicitis Ann Emerg Med 15557ndash564

You are seeing a child in your office with 1-day history of

RLQ pain You are suspicious for appendicitis Where would

you send the patient

A Emergency room

B Same day outpatient surgery clinic

C CT scan

D Ultrasound

E Other

What to do with suspected appendicitis

Obtain imaging vs send to ED

ndash If any suspicion for viral syndrome with dehydration to ED

ndash If VERY HIGH suspicion for appendicitis and good pediatric radiologist is available consider sending child for ultrasound

Operating Room

further wu per surgeon

Further workup and imaging in

consultation with

Surgery team

1 IV placed 20 cckg bolus CBC

2 Surgical consult within one hour

Strong suspicion on

HPE Equivocal history

physical exam

1 IV placed 20 cckg bolus CBC

2 Re-examine surgical consult within one

hour if still tender

Abdominal pain

ro appendicitis

What is the best way to image the appendix

A CT scan

B Ultrasound

C MRI

D Abdominal X ray

E Other

Ultrasound

CT scan

MRI

Summary of approach to imaging

Ultrasound is good if you have a high pretest probability good radiologist thin patient

MRI is good if you are in a center that uses it routinely to evaluate children with appendicitis

CT scan is best if above criteria are not met

Alternative to imaging

ndash OBSERVATION

Once diagnosis of appendicitis has been establishedhellip

Non-ruptured

ndash Short duration of symptoms no suggestion of rupture on imaging

ndash Start antibiotics

ndash Perform appendectomy

Ruptured

ndash Have radiology evaluate for drainable fluid collection

ndash Antibiotics

ndash Operation if above management fails

Once diagnosis of appendicitis has been establishedhellip

Non-ruptured

ndash Short duration of symptoms no suggestion of rupture on imaging

ndash Start antibiotics

ndash Perform appendectomy

Ruptured

ndash Have radiology evaluate for drainable fluid collection

ndash Antibiotics

ndash Operation if above management fails

When should appendectomy be performed in a

patient with non-ruptured acute appendicitis

A Immediately after diagnosis

B Within first 24 hours of hospital stay once appropriate antibiotics and IVF resuscitation have been provided

C After completion of a one-week course of antibiotics

D It is reasonable to treat these patients with antibiotics alone and not perform an appendectomy

Is the appendix a ticking time bomb

Traditional teaching appendicitis is an emergency and should be treated with surgery immediately upon diagnosis

Several studies have now shown that delays of 12 hours up to 48 hours do not increase risk of perforation

Current standard admit IVF resuscitation antibiotics appendectomy during daylight hours

Almstroumlm M1 Svensson JF Patkova B Svenningsson A Wester TAnn Surg 2016 Mar 8 In-hospital

Surgical Delay Does Not Increase the Risk for Perforated Appendicitis in Children A Single-center

Retrospective Cohort Study

Port placement

Lap appy with Endo-loops

Lap appy with stapler

Open appendectomy

Appendix with fecalith

Laparoscopic vs open appendectomy

Laparoscopy is standard of care

Lower complication rate

Less scar

Less pain

Ability to evaluate other intra-abdominal organs if the appendix looks normal

ndash Gallbladder

ndash Ovaries

ndash Inguinal canal

Sauerland S Jaschinski T Neugebauer EA Laparoscopic versus open surgery for

suspected appendicitis Cochrane Database Syst Rev 2010 Oct 6(10)

Post-op course

Most patients can go home on the day of surgery

Recovery is usually quick

Back to school within a week sports within 2 weeks

Very low risk surgery with good outcomes

Risk of infection is about 5 at port sites or in abdomen

You see a 7 year old male in your office with 1-day history of abdominal pain

now localized to RLQ with focal guarding and poor appetite You send him for

ultrasound and it shows appendicitis His mother has heard about some new

research and wants to know if you would recommend just treating with

antibiotics rather than surgery How would you counsel this mom

A This data is preliminary and surgery is still standard of

care

B This may be a reasonable option for her son and she

should discuss it with her surgeon

C If it were your son you wouldnrsquot let him have surgery

D Other

Do all patients with appendicitis require surgery

Pilot study at Nationwide Childrenrsquos Hospital

102 patients

ndash 7 to 17 years of age

ndash Uncomplicated appendicitis defined by

Abdominal pain le 48 hours

White blood cell count le 18000

Ultrasound or CT scan showing appendicitis with an appendix le 11 centimeter thick and no

evidence of abscess or fecalith

Patients and families chose to have appendectomy or antibiotics alone

Non-operative management at least 24 hours of in-hospital observation and IV antibiotics until symptoms improved followed by completion of 10 days of treatment with antibiotics by mouth

Minneci PC Mahida JB et al The effectiveness of patient choice in non-operative

versus surgical management of uncomplicated acute appendicitis JAMA Surgery

2015 Dec 16

Results of non-operative management

65 families chose appendectomy 37 families chose non-operative management

Success rate of non-operative management (defined as not undergoing an appendectomy) 89 at 30 days 76 at 1 year

The 24 who failed did NOT have a higher rate of ruptured appendicitis compared to the patients who had immediate appendectomy

1 year follow-up the children managed non-operatively compared with the surgery group had fewer disability days (8 vs 21 days) lower appendicitis-related health care costs (median $4219 vs $5029) and no difference in health-related quality of life

Non-Operative Treatment of Appendicitis Rationale

Appendectomy is invasive

Children may miss up to two weeks of schoolactivities

Caregivers miss work

Postop complications after appendectomy for uncomplicated appendicitis 5-10

Serious complications (reoperations or readmissions) 1-7

Adult data suggest one-year success rates of 63-85 no difference in rates of complicated appendicitis

bull Meta-analysis of 10 articles reporting 413 children receiving non

operative treatment (NOT) for appendicitis ndash all published in past 10

years

bull 5 comparative

bull 1 RCT

bull 4 case series of NOT

Interpreting meta-analyses Forest plot

Favors

antibiotics

Favors

appendectomy

Overall success of NOT

97 during initial episode

Incidence of recurrent appendicitis during follow-up period (range 2-51

months)

Long term efficacy of NOT (no appendectomy at end of follow-up

period)

LOS shortened by about 05 days in those undergoing appendectomy

compared to NOT

Increased rate of complications for appendectomy compared to NOT

Authorsrsquo Conclusions

ldquoCurrent data suggest that NOT is safe It appears effective as initial treatment in 97 of children with AUA and the rate of recurrent appendicitis is 14rdquo

The study highlights the lack of robust evidence comparing NOT with appendectomy in children

Confirms a position of equipoise between treatment approaches

We recommend that NOT of children with AUA be reserved for those participating in carefully designed research studies

Take-home point

Lap appy is still standard of care for uncomplicated appendicitis

Once diagnosis of appendicitis has been establishedhellip

Non-ruptured

ndash Short duration of symptoms no suggestion of rupture on imaging

ndash Start antibiotics

ndash Perform appendectomy

Ruptured

ndash Have radiology evaluate for drainable fluid collection

ndash Antibiotics

ndash Operation in acute setting if above management fails

ndash Interval appendectomy 6-8 weeks laterhellip

ndash ALTERNATIVELY Just take out the appendix

CT ruptured appendix with abscess

After placement of percutaneous drain

How to counsel families of children with ruptured appendicitis

If kids tolerate a diet pain resolves fever resolves oral antibiotics and home

Plan interval appy 6-8 weeks later

Non-operative management may not work and surgery may be needed (Failure rate 20)

At home they should watch for signs of persistentrecurrent appendicitis

High-anxiety time for patients and families

A 10-year-old boy comes to see you in the office after recent

hospitalization for perforated appendicitis He has 3 more days of

antibiotics left His mom is worried because appetite is poor and his

energy level is low On exam he has diffuse lower abdominal

tenderness You recommend

A Extending course of oral antibiotics

B CT scan to evaluate for persistent or

recurrent appendicitis

C CT scan to evaluate for intra-abdominal

abscess

D Follow-up with surgeon

Is interval appendectomy necessary

In adults many surgeons do not do this operation

In kids data are limited

ndash 2-year follow-up of 96 patients

ndash Perforated appendicitis treated non-operatively with antibiotics

ndash 6 became worse 41 had interval appendectomy

ndash 49 received no further treatment

ndash 57 no recurrence

ndash 43 had recurrence within one month to 2 years

ndash Presence of appendicolith 72 rate of recurrence vs 26 in those without appendicolith

Ein SH1 Langer JC Daneman A Nonoperative management of pediatric ruptured appendix

with inflammatory mass or abscess presence of an appendicolith predicts recurrent

appendicitis J Pediatr Surg 2005 Oct40(10)1612-5

Approach to interval appendectomy

More strongly recommended if fecalith present

Observation is a reasonable option

Best choice for an individual patient depends on their anxiety and parental anxiety

Immediate operation for ruptured appendicitis perhaps a better

option

Meta-analysis operative vs non-operative management of pediatric

ruptured appendicitis

2 RCTs identified

Total of 171 pediatric patients

Compared early vs interval appendectomy

Early appendectomy reduced incidence of adverse event

Ruptured appendicitis may have abscess or phlegmon

Early surgery was more strongly favored when there was no abscess at

time of presentation

Decreased antibiotic duration length of stay and total charges for

abscess and no abscess groups

You are seeing a 7 year old with constipation A

fecalith was seen on abdominal X ray during recent

ED visit How would you counsel the family

A They should be referred to surgery for appendectomy

B Their child is at higher risk for appendicitis so they should be aware of this in case he develops symptoms

C It is uncertain whether this child is at higher risk for appendicitis

D A course of Miralax may help wash out the fecalith

Take home points

Uncomplicated appendicitis

ndash Lap appy is still standard of care

ndash Non-operative management may be an acceptable option but not enough is known about long-term risk of recurrent appendicitis

Complicated appendicitis

ndash Can be managed with immediate operation delayed appendectomy or no appendectomy

ndash Immediate operation is probably more efficient and less stressful for patients and parents

Thank you very much

Questions

Page 3: New approaches for evaluation and treatment of appendicitis · Ultrasound or CT scan showing appendicitis with an appendix ≤ 1.1 centimeter thick and no evidence of abscess or fecalith

Appendicitis scope of the disease

Over 70000 cases annually in children in USA

Lifetime risk of appendicitis 9 for boys 7 for girls

Peak incidence 12 to 18 years

Journal of Pediatric Surgery 2017 52 669-676DOI (101016jjpedsurg201701013)

Copyright copy 2017 Elsevier Inc Terms and Conditions

Burden of Appendectomy Hospital-Acquired Infection

Journal of Pediatric Surgery 2017 52 669-676DOI (101016jjpedsurg201701013)

Copyright copy 2017 Elsevier Inc Terms and Conditions

Burden of Appendectomy Cost Variation

Objectives

Why do children get appendicitis

What are typical presenting features

How do we make the diagnosis

How do we decide WHETHER to operate

How do we decide WHEN to operate

What is the difference in how we approach perforated vs non-perforated appendicitis

Does every child with appendicitis actually need surgery

Why does appendicitis occur

Who is more likely to present with perforated

appendicitis

A Younger patients

B Patients with developmental delay

C Uninsured patients

D Minorities

E All of the above

More likely to present with perforated appendicitis

Perforation rates are reported from 20-80 in children

Younger kids

ndash 82 in children under 5

ndash Nearly 100 of 1-year-olds

Developmental delay

Uninsured

Minorities

Kokoska ER1 Bird TM Robbins JM Smith SD Corsi JM Campbell BT Racial disparities

in the management of pediatric appendicitis J Surg Res 2007 Jan137(1)83-8

Which of the following symptoms is LEAST likely to

be consistent with appendicitis

A Fever

B Nauseavomiting

C Diarrhea

D Peri-umbilical pain

E Anorexia

How do children present with appendicitis

Pain starts peri-umbilical and then migrates to RLQ

Pain starts insidiously is persistent and worsens over time

Pain is worse with movement

Fever nausea with or without vomiting and anorexia

Features suggesting alternative diagnoses

ndash Waxing and waning pain

ndash Diarrhea cough sore throat myalgias rhinorrhea and sick contacts

ndash (BUT there are patients with many of the above symptoms who DO have appendicitis)

Differential diagnosis

Virus

Pneumonia

Constipation

In adolescent females

ndash PID

ndash Ovarian cyst

ndash Mittelschmertz

Approach to physical exam in suspected appendicitis

Child does NOT want to move around

Focal tenderness

Percussion of the abdomen causes discomfort (this is better than rebound tenderness)

Psoas obturator Rovsingrsquos heel strike

Watch them walk around

ldquoDoes this child have appendicitisrdquo Meta-Analysis

Fever

ndash If present LR 34 (24-48)

ndash If absent LR 032 (016-064)

Rebound tenderness

ndash If present LR 30 (23-39)

ndash If absent LR 028 (014-055)

Migration of pain LR range 19-31

RLQ pain LR 12 (10-15)

WBC less than 10 LR 022 (017-030)

ANC less than 6750 LR 006 (003-016) Bundy DG Byerley JS Liles EA Perrin EM Katznelson J Rice HE

Does this child have appendicitis JAMA 2007 Jul 25298(4)438-51

Alvarado A (1986) A practical score for the early diagnosis

of acute appendicitis Ann Emerg Med 15557ndash564

You are seeing a child in your office with 1-day history of

RLQ pain You are suspicious for appendicitis Where would

you send the patient

A Emergency room

B Same day outpatient surgery clinic

C CT scan

D Ultrasound

E Other

What to do with suspected appendicitis

Obtain imaging vs send to ED

ndash If any suspicion for viral syndrome with dehydration to ED

ndash If VERY HIGH suspicion for appendicitis and good pediatric radiologist is available consider sending child for ultrasound

Operating Room

further wu per surgeon

Further workup and imaging in

consultation with

Surgery team

1 IV placed 20 cckg bolus CBC

2 Surgical consult within one hour

Strong suspicion on

HPE Equivocal history

physical exam

1 IV placed 20 cckg bolus CBC

2 Re-examine surgical consult within one

hour if still tender

Abdominal pain

ro appendicitis

What is the best way to image the appendix

A CT scan

B Ultrasound

C MRI

D Abdominal X ray

E Other

Ultrasound

CT scan

MRI

Summary of approach to imaging

Ultrasound is good if you have a high pretest probability good radiologist thin patient

MRI is good if you are in a center that uses it routinely to evaluate children with appendicitis

CT scan is best if above criteria are not met

Alternative to imaging

ndash OBSERVATION

Once diagnosis of appendicitis has been establishedhellip

Non-ruptured

ndash Short duration of symptoms no suggestion of rupture on imaging

ndash Start antibiotics

ndash Perform appendectomy

Ruptured

ndash Have radiology evaluate for drainable fluid collection

ndash Antibiotics

ndash Operation if above management fails

Once diagnosis of appendicitis has been establishedhellip

Non-ruptured

ndash Short duration of symptoms no suggestion of rupture on imaging

ndash Start antibiotics

ndash Perform appendectomy

Ruptured

ndash Have radiology evaluate for drainable fluid collection

ndash Antibiotics

ndash Operation if above management fails

When should appendectomy be performed in a

patient with non-ruptured acute appendicitis

A Immediately after diagnosis

B Within first 24 hours of hospital stay once appropriate antibiotics and IVF resuscitation have been provided

C After completion of a one-week course of antibiotics

D It is reasonable to treat these patients with antibiotics alone and not perform an appendectomy

Is the appendix a ticking time bomb

Traditional teaching appendicitis is an emergency and should be treated with surgery immediately upon diagnosis

Several studies have now shown that delays of 12 hours up to 48 hours do not increase risk of perforation

Current standard admit IVF resuscitation antibiotics appendectomy during daylight hours

Almstroumlm M1 Svensson JF Patkova B Svenningsson A Wester TAnn Surg 2016 Mar 8 In-hospital

Surgical Delay Does Not Increase the Risk for Perforated Appendicitis in Children A Single-center

Retrospective Cohort Study

Port placement

Lap appy with Endo-loops

Lap appy with stapler

Open appendectomy

Appendix with fecalith

Laparoscopic vs open appendectomy

Laparoscopy is standard of care

Lower complication rate

Less scar

Less pain

Ability to evaluate other intra-abdominal organs if the appendix looks normal

ndash Gallbladder

ndash Ovaries

ndash Inguinal canal

Sauerland S Jaschinski T Neugebauer EA Laparoscopic versus open surgery for

suspected appendicitis Cochrane Database Syst Rev 2010 Oct 6(10)

Post-op course

Most patients can go home on the day of surgery

Recovery is usually quick

Back to school within a week sports within 2 weeks

Very low risk surgery with good outcomes

Risk of infection is about 5 at port sites or in abdomen

You see a 7 year old male in your office with 1-day history of abdominal pain

now localized to RLQ with focal guarding and poor appetite You send him for

ultrasound and it shows appendicitis His mother has heard about some new

research and wants to know if you would recommend just treating with

antibiotics rather than surgery How would you counsel this mom

A This data is preliminary and surgery is still standard of

care

B This may be a reasonable option for her son and she

should discuss it with her surgeon

C If it were your son you wouldnrsquot let him have surgery

D Other

Do all patients with appendicitis require surgery

Pilot study at Nationwide Childrenrsquos Hospital

102 patients

ndash 7 to 17 years of age

ndash Uncomplicated appendicitis defined by

Abdominal pain le 48 hours

White blood cell count le 18000

Ultrasound or CT scan showing appendicitis with an appendix le 11 centimeter thick and no

evidence of abscess or fecalith

Patients and families chose to have appendectomy or antibiotics alone

Non-operative management at least 24 hours of in-hospital observation and IV antibiotics until symptoms improved followed by completion of 10 days of treatment with antibiotics by mouth

Minneci PC Mahida JB et al The effectiveness of patient choice in non-operative

versus surgical management of uncomplicated acute appendicitis JAMA Surgery

2015 Dec 16

Results of non-operative management

65 families chose appendectomy 37 families chose non-operative management

Success rate of non-operative management (defined as not undergoing an appendectomy) 89 at 30 days 76 at 1 year

The 24 who failed did NOT have a higher rate of ruptured appendicitis compared to the patients who had immediate appendectomy

1 year follow-up the children managed non-operatively compared with the surgery group had fewer disability days (8 vs 21 days) lower appendicitis-related health care costs (median $4219 vs $5029) and no difference in health-related quality of life

Non-Operative Treatment of Appendicitis Rationale

Appendectomy is invasive

Children may miss up to two weeks of schoolactivities

Caregivers miss work

Postop complications after appendectomy for uncomplicated appendicitis 5-10

Serious complications (reoperations or readmissions) 1-7

Adult data suggest one-year success rates of 63-85 no difference in rates of complicated appendicitis

bull Meta-analysis of 10 articles reporting 413 children receiving non

operative treatment (NOT) for appendicitis ndash all published in past 10

years

bull 5 comparative

bull 1 RCT

bull 4 case series of NOT

Interpreting meta-analyses Forest plot

Favors

antibiotics

Favors

appendectomy

Overall success of NOT

97 during initial episode

Incidence of recurrent appendicitis during follow-up period (range 2-51

months)

Long term efficacy of NOT (no appendectomy at end of follow-up

period)

LOS shortened by about 05 days in those undergoing appendectomy

compared to NOT

Increased rate of complications for appendectomy compared to NOT

Authorsrsquo Conclusions

ldquoCurrent data suggest that NOT is safe It appears effective as initial treatment in 97 of children with AUA and the rate of recurrent appendicitis is 14rdquo

The study highlights the lack of robust evidence comparing NOT with appendectomy in children

Confirms a position of equipoise between treatment approaches

We recommend that NOT of children with AUA be reserved for those participating in carefully designed research studies

Take-home point

Lap appy is still standard of care for uncomplicated appendicitis

Once diagnosis of appendicitis has been establishedhellip

Non-ruptured

ndash Short duration of symptoms no suggestion of rupture on imaging

ndash Start antibiotics

ndash Perform appendectomy

Ruptured

ndash Have radiology evaluate for drainable fluid collection

ndash Antibiotics

ndash Operation in acute setting if above management fails

ndash Interval appendectomy 6-8 weeks laterhellip

ndash ALTERNATIVELY Just take out the appendix

CT ruptured appendix with abscess

After placement of percutaneous drain

How to counsel families of children with ruptured appendicitis

If kids tolerate a diet pain resolves fever resolves oral antibiotics and home

Plan interval appy 6-8 weeks later

Non-operative management may not work and surgery may be needed (Failure rate 20)

At home they should watch for signs of persistentrecurrent appendicitis

High-anxiety time for patients and families

A 10-year-old boy comes to see you in the office after recent

hospitalization for perforated appendicitis He has 3 more days of

antibiotics left His mom is worried because appetite is poor and his

energy level is low On exam he has diffuse lower abdominal

tenderness You recommend

A Extending course of oral antibiotics

B CT scan to evaluate for persistent or

recurrent appendicitis

C CT scan to evaluate for intra-abdominal

abscess

D Follow-up with surgeon

Is interval appendectomy necessary

In adults many surgeons do not do this operation

In kids data are limited

ndash 2-year follow-up of 96 patients

ndash Perforated appendicitis treated non-operatively with antibiotics

ndash 6 became worse 41 had interval appendectomy

ndash 49 received no further treatment

ndash 57 no recurrence

ndash 43 had recurrence within one month to 2 years

ndash Presence of appendicolith 72 rate of recurrence vs 26 in those without appendicolith

Ein SH1 Langer JC Daneman A Nonoperative management of pediatric ruptured appendix

with inflammatory mass or abscess presence of an appendicolith predicts recurrent

appendicitis J Pediatr Surg 2005 Oct40(10)1612-5

Approach to interval appendectomy

More strongly recommended if fecalith present

Observation is a reasonable option

Best choice for an individual patient depends on their anxiety and parental anxiety

Immediate operation for ruptured appendicitis perhaps a better

option

Meta-analysis operative vs non-operative management of pediatric

ruptured appendicitis

2 RCTs identified

Total of 171 pediatric patients

Compared early vs interval appendectomy

Early appendectomy reduced incidence of adverse event

Ruptured appendicitis may have abscess or phlegmon

Early surgery was more strongly favored when there was no abscess at

time of presentation

Decreased antibiotic duration length of stay and total charges for

abscess and no abscess groups

You are seeing a 7 year old with constipation A

fecalith was seen on abdominal X ray during recent

ED visit How would you counsel the family

A They should be referred to surgery for appendectomy

B Their child is at higher risk for appendicitis so they should be aware of this in case he develops symptoms

C It is uncertain whether this child is at higher risk for appendicitis

D A course of Miralax may help wash out the fecalith

Take home points

Uncomplicated appendicitis

ndash Lap appy is still standard of care

ndash Non-operative management may be an acceptable option but not enough is known about long-term risk of recurrent appendicitis

Complicated appendicitis

ndash Can be managed with immediate operation delayed appendectomy or no appendectomy

ndash Immediate operation is probably more efficient and less stressful for patients and parents

Thank you very much

Questions

Page 4: New approaches for evaluation and treatment of appendicitis · Ultrasound or CT scan showing appendicitis with an appendix ≤ 1.1 centimeter thick and no evidence of abscess or fecalith

Journal of Pediatric Surgery 2017 52 669-676DOI (101016jjpedsurg201701013)

Copyright copy 2017 Elsevier Inc Terms and Conditions

Burden of Appendectomy Hospital-Acquired Infection

Journal of Pediatric Surgery 2017 52 669-676DOI (101016jjpedsurg201701013)

Copyright copy 2017 Elsevier Inc Terms and Conditions

Burden of Appendectomy Cost Variation

Objectives

Why do children get appendicitis

What are typical presenting features

How do we make the diagnosis

How do we decide WHETHER to operate

How do we decide WHEN to operate

What is the difference in how we approach perforated vs non-perforated appendicitis

Does every child with appendicitis actually need surgery

Why does appendicitis occur

Who is more likely to present with perforated

appendicitis

A Younger patients

B Patients with developmental delay

C Uninsured patients

D Minorities

E All of the above

More likely to present with perforated appendicitis

Perforation rates are reported from 20-80 in children

Younger kids

ndash 82 in children under 5

ndash Nearly 100 of 1-year-olds

Developmental delay

Uninsured

Minorities

Kokoska ER1 Bird TM Robbins JM Smith SD Corsi JM Campbell BT Racial disparities

in the management of pediatric appendicitis J Surg Res 2007 Jan137(1)83-8

Which of the following symptoms is LEAST likely to

be consistent with appendicitis

A Fever

B Nauseavomiting

C Diarrhea

D Peri-umbilical pain

E Anorexia

How do children present with appendicitis

Pain starts peri-umbilical and then migrates to RLQ

Pain starts insidiously is persistent and worsens over time

Pain is worse with movement

Fever nausea with or without vomiting and anorexia

Features suggesting alternative diagnoses

ndash Waxing and waning pain

ndash Diarrhea cough sore throat myalgias rhinorrhea and sick contacts

ndash (BUT there are patients with many of the above symptoms who DO have appendicitis)

Differential diagnosis

Virus

Pneumonia

Constipation

In adolescent females

ndash PID

ndash Ovarian cyst

ndash Mittelschmertz

Approach to physical exam in suspected appendicitis

Child does NOT want to move around

Focal tenderness

Percussion of the abdomen causes discomfort (this is better than rebound tenderness)

Psoas obturator Rovsingrsquos heel strike

Watch them walk around

ldquoDoes this child have appendicitisrdquo Meta-Analysis

Fever

ndash If present LR 34 (24-48)

ndash If absent LR 032 (016-064)

Rebound tenderness

ndash If present LR 30 (23-39)

ndash If absent LR 028 (014-055)

Migration of pain LR range 19-31

RLQ pain LR 12 (10-15)

WBC less than 10 LR 022 (017-030)

ANC less than 6750 LR 006 (003-016) Bundy DG Byerley JS Liles EA Perrin EM Katznelson J Rice HE

Does this child have appendicitis JAMA 2007 Jul 25298(4)438-51

Alvarado A (1986) A practical score for the early diagnosis

of acute appendicitis Ann Emerg Med 15557ndash564

You are seeing a child in your office with 1-day history of

RLQ pain You are suspicious for appendicitis Where would

you send the patient

A Emergency room

B Same day outpatient surgery clinic

C CT scan

D Ultrasound

E Other

What to do with suspected appendicitis

Obtain imaging vs send to ED

ndash If any suspicion for viral syndrome with dehydration to ED

ndash If VERY HIGH suspicion for appendicitis and good pediatric radiologist is available consider sending child for ultrasound

Operating Room

further wu per surgeon

Further workup and imaging in

consultation with

Surgery team

1 IV placed 20 cckg bolus CBC

2 Surgical consult within one hour

Strong suspicion on

HPE Equivocal history

physical exam

1 IV placed 20 cckg bolus CBC

2 Re-examine surgical consult within one

hour if still tender

Abdominal pain

ro appendicitis

What is the best way to image the appendix

A CT scan

B Ultrasound

C MRI

D Abdominal X ray

E Other

Ultrasound

CT scan

MRI

Summary of approach to imaging

Ultrasound is good if you have a high pretest probability good radiologist thin patient

MRI is good if you are in a center that uses it routinely to evaluate children with appendicitis

CT scan is best if above criteria are not met

Alternative to imaging

ndash OBSERVATION

Once diagnosis of appendicitis has been establishedhellip

Non-ruptured

ndash Short duration of symptoms no suggestion of rupture on imaging

ndash Start antibiotics

ndash Perform appendectomy

Ruptured

ndash Have radiology evaluate for drainable fluid collection

ndash Antibiotics

ndash Operation if above management fails

Once diagnosis of appendicitis has been establishedhellip

Non-ruptured

ndash Short duration of symptoms no suggestion of rupture on imaging

ndash Start antibiotics

ndash Perform appendectomy

Ruptured

ndash Have radiology evaluate for drainable fluid collection

ndash Antibiotics

ndash Operation if above management fails

When should appendectomy be performed in a

patient with non-ruptured acute appendicitis

A Immediately after diagnosis

B Within first 24 hours of hospital stay once appropriate antibiotics and IVF resuscitation have been provided

C After completion of a one-week course of antibiotics

D It is reasonable to treat these patients with antibiotics alone and not perform an appendectomy

Is the appendix a ticking time bomb

Traditional teaching appendicitis is an emergency and should be treated with surgery immediately upon diagnosis

Several studies have now shown that delays of 12 hours up to 48 hours do not increase risk of perforation

Current standard admit IVF resuscitation antibiotics appendectomy during daylight hours

Almstroumlm M1 Svensson JF Patkova B Svenningsson A Wester TAnn Surg 2016 Mar 8 In-hospital

Surgical Delay Does Not Increase the Risk for Perforated Appendicitis in Children A Single-center

Retrospective Cohort Study

Port placement

Lap appy with Endo-loops

Lap appy with stapler

Open appendectomy

Appendix with fecalith

Laparoscopic vs open appendectomy

Laparoscopy is standard of care

Lower complication rate

Less scar

Less pain

Ability to evaluate other intra-abdominal organs if the appendix looks normal

ndash Gallbladder

ndash Ovaries

ndash Inguinal canal

Sauerland S Jaschinski T Neugebauer EA Laparoscopic versus open surgery for

suspected appendicitis Cochrane Database Syst Rev 2010 Oct 6(10)

Post-op course

Most patients can go home on the day of surgery

Recovery is usually quick

Back to school within a week sports within 2 weeks

Very low risk surgery with good outcomes

Risk of infection is about 5 at port sites or in abdomen

You see a 7 year old male in your office with 1-day history of abdominal pain

now localized to RLQ with focal guarding and poor appetite You send him for

ultrasound and it shows appendicitis His mother has heard about some new

research and wants to know if you would recommend just treating with

antibiotics rather than surgery How would you counsel this mom

A This data is preliminary and surgery is still standard of

care

B This may be a reasonable option for her son and she

should discuss it with her surgeon

C If it were your son you wouldnrsquot let him have surgery

D Other

Do all patients with appendicitis require surgery

Pilot study at Nationwide Childrenrsquos Hospital

102 patients

ndash 7 to 17 years of age

ndash Uncomplicated appendicitis defined by

Abdominal pain le 48 hours

White blood cell count le 18000

Ultrasound or CT scan showing appendicitis with an appendix le 11 centimeter thick and no

evidence of abscess or fecalith

Patients and families chose to have appendectomy or antibiotics alone

Non-operative management at least 24 hours of in-hospital observation and IV antibiotics until symptoms improved followed by completion of 10 days of treatment with antibiotics by mouth

Minneci PC Mahida JB et al The effectiveness of patient choice in non-operative

versus surgical management of uncomplicated acute appendicitis JAMA Surgery

2015 Dec 16

Results of non-operative management

65 families chose appendectomy 37 families chose non-operative management

Success rate of non-operative management (defined as not undergoing an appendectomy) 89 at 30 days 76 at 1 year

The 24 who failed did NOT have a higher rate of ruptured appendicitis compared to the patients who had immediate appendectomy

1 year follow-up the children managed non-operatively compared with the surgery group had fewer disability days (8 vs 21 days) lower appendicitis-related health care costs (median $4219 vs $5029) and no difference in health-related quality of life

Non-Operative Treatment of Appendicitis Rationale

Appendectomy is invasive

Children may miss up to two weeks of schoolactivities

Caregivers miss work

Postop complications after appendectomy for uncomplicated appendicitis 5-10

Serious complications (reoperations or readmissions) 1-7

Adult data suggest one-year success rates of 63-85 no difference in rates of complicated appendicitis

bull Meta-analysis of 10 articles reporting 413 children receiving non

operative treatment (NOT) for appendicitis ndash all published in past 10

years

bull 5 comparative

bull 1 RCT

bull 4 case series of NOT

Interpreting meta-analyses Forest plot

Favors

antibiotics

Favors

appendectomy

Overall success of NOT

97 during initial episode

Incidence of recurrent appendicitis during follow-up period (range 2-51

months)

Long term efficacy of NOT (no appendectomy at end of follow-up

period)

LOS shortened by about 05 days in those undergoing appendectomy

compared to NOT

Increased rate of complications for appendectomy compared to NOT

Authorsrsquo Conclusions

ldquoCurrent data suggest that NOT is safe It appears effective as initial treatment in 97 of children with AUA and the rate of recurrent appendicitis is 14rdquo

The study highlights the lack of robust evidence comparing NOT with appendectomy in children

Confirms a position of equipoise between treatment approaches

We recommend that NOT of children with AUA be reserved for those participating in carefully designed research studies

Take-home point

Lap appy is still standard of care for uncomplicated appendicitis

Once diagnosis of appendicitis has been establishedhellip

Non-ruptured

ndash Short duration of symptoms no suggestion of rupture on imaging

ndash Start antibiotics

ndash Perform appendectomy

Ruptured

ndash Have radiology evaluate for drainable fluid collection

ndash Antibiotics

ndash Operation in acute setting if above management fails

ndash Interval appendectomy 6-8 weeks laterhellip

ndash ALTERNATIVELY Just take out the appendix

CT ruptured appendix with abscess

After placement of percutaneous drain

How to counsel families of children with ruptured appendicitis

If kids tolerate a diet pain resolves fever resolves oral antibiotics and home

Plan interval appy 6-8 weeks later

Non-operative management may not work and surgery may be needed (Failure rate 20)

At home they should watch for signs of persistentrecurrent appendicitis

High-anxiety time for patients and families

A 10-year-old boy comes to see you in the office after recent

hospitalization for perforated appendicitis He has 3 more days of

antibiotics left His mom is worried because appetite is poor and his

energy level is low On exam he has diffuse lower abdominal

tenderness You recommend

A Extending course of oral antibiotics

B CT scan to evaluate for persistent or

recurrent appendicitis

C CT scan to evaluate for intra-abdominal

abscess

D Follow-up with surgeon

Is interval appendectomy necessary

In adults many surgeons do not do this operation

In kids data are limited

ndash 2-year follow-up of 96 patients

ndash Perforated appendicitis treated non-operatively with antibiotics

ndash 6 became worse 41 had interval appendectomy

ndash 49 received no further treatment

ndash 57 no recurrence

ndash 43 had recurrence within one month to 2 years

ndash Presence of appendicolith 72 rate of recurrence vs 26 in those without appendicolith

Ein SH1 Langer JC Daneman A Nonoperative management of pediatric ruptured appendix

with inflammatory mass or abscess presence of an appendicolith predicts recurrent

appendicitis J Pediatr Surg 2005 Oct40(10)1612-5

Approach to interval appendectomy

More strongly recommended if fecalith present

Observation is a reasonable option

Best choice for an individual patient depends on their anxiety and parental anxiety

Immediate operation for ruptured appendicitis perhaps a better

option

Meta-analysis operative vs non-operative management of pediatric

ruptured appendicitis

2 RCTs identified

Total of 171 pediatric patients

Compared early vs interval appendectomy

Early appendectomy reduced incidence of adverse event

Ruptured appendicitis may have abscess or phlegmon

Early surgery was more strongly favored when there was no abscess at

time of presentation

Decreased antibiotic duration length of stay and total charges for

abscess and no abscess groups

You are seeing a 7 year old with constipation A

fecalith was seen on abdominal X ray during recent

ED visit How would you counsel the family

A They should be referred to surgery for appendectomy

B Their child is at higher risk for appendicitis so they should be aware of this in case he develops symptoms

C It is uncertain whether this child is at higher risk for appendicitis

D A course of Miralax may help wash out the fecalith

Take home points

Uncomplicated appendicitis

ndash Lap appy is still standard of care

ndash Non-operative management may be an acceptable option but not enough is known about long-term risk of recurrent appendicitis

Complicated appendicitis

ndash Can be managed with immediate operation delayed appendectomy or no appendectomy

ndash Immediate operation is probably more efficient and less stressful for patients and parents

Thank you very much

Questions

Page 5: New approaches for evaluation and treatment of appendicitis · Ultrasound or CT scan showing appendicitis with an appendix ≤ 1.1 centimeter thick and no evidence of abscess or fecalith

Journal of Pediatric Surgery 2017 52 669-676DOI (101016jjpedsurg201701013)

Copyright copy 2017 Elsevier Inc Terms and Conditions

Burden of Appendectomy Cost Variation

Objectives

Why do children get appendicitis

What are typical presenting features

How do we make the diagnosis

How do we decide WHETHER to operate

How do we decide WHEN to operate

What is the difference in how we approach perforated vs non-perforated appendicitis

Does every child with appendicitis actually need surgery

Why does appendicitis occur

Who is more likely to present with perforated

appendicitis

A Younger patients

B Patients with developmental delay

C Uninsured patients

D Minorities

E All of the above

More likely to present with perforated appendicitis

Perforation rates are reported from 20-80 in children

Younger kids

ndash 82 in children under 5

ndash Nearly 100 of 1-year-olds

Developmental delay

Uninsured

Minorities

Kokoska ER1 Bird TM Robbins JM Smith SD Corsi JM Campbell BT Racial disparities

in the management of pediatric appendicitis J Surg Res 2007 Jan137(1)83-8

Which of the following symptoms is LEAST likely to

be consistent with appendicitis

A Fever

B Nauseavomiting

C Diarrhea

D Peri-umbilical pain

E Anorexia

How do children present with appendicitis

Pain starts peri-umbilical and then migrates to RLQ

Pain starts insidiously is persistent and worsens over time

Pain is worse with movement

Fever nausea with or without vomiting and anorexia

Features suggesting alternative diagnoses

ndash Waxing and waning pain

ndash Diarrhea cough sore throat myalgias rhinorrhea and sick contacts

ndash (BUT there are patients with many of the above symptoms who DO have appendicitis)

Differential diagnosis

Virus

Pneumonia

Constipation

In adolescent females

ndash PID

ndash Ovarian cyst

ndash Mittelschmertz

Approach to physical exam in suspected appendicitis

Child does NOT want to move around

Focal tenderness

Percussion of the abdomen causes discomfort (this is better than rebound tenderness)

Psoas obturator Rovsingrsquos heel strike

Watch them walk around

ldquoDoes this child have appendicitisrdquo Meta-Analysis

Fever

ndash If present LR 34 (24-48)

ndash If absent LR 032 (016-064)

Rebound tenderness

ndash If present LR 30 (23-39)

ndash If absent LR 028 (014-055)

Migration of pain LR range 19-31

RLQ pain LR 12 (10-15)

WBC less than 10 LR 022 (017-030)

ANC less than 6750 LR 006 (003-016) Bundy DG Byerley JS Liles EA Perrin EM Katznelson J Rice HE

Does this child have appendicitis JAMA 2007 Jul 25298(4)438-51

Alvarado A (1986) A practical score for the early diagnosis

of acute appendicitis Ann Emerg Med 15557ndash564

You are seeing a child in your office with 1-day history of

RLQ pain You are suspicious for appendicitis Where would

you send the patient

A Emergency room

B Same day outpatient surgery clinic

C CT scan

D Ultrasound

E Other

What to do with suspected appendicitis

Obtain imaging vs send to ED

ndash If any suspicion for viral syndrome with dehydration to ED

ndash If VERY HIGH suspicion for appendicitis and good pediatric radiologist is available consider sending child for ultrasound

Operating Room

further wu per surgeon

Further workup and imaging in

consultation with

Surgery team

1 IV placed 20 cckg bolus CBC

2 Surgical consult within one hour

Strong suspicion on

HPE Equivocal history

physical exam

1 IV placed 20 cckg bolus CBC

2 Re-examine surgical consult within one

hour if still tender

Abdominal pain

ro appendicitis

What is the best way to image the appendix

A CT scan

B Ultrasound

C MRI

D Abdominal X ray

E Other

Ultrasound

CT scan

MRI

Summary of approach to imaging

Ultrasound is good if you have a high pretest probability good radiologist thin patient

MRI is good if you are in a center that uses it routinely to evaluate children with appendicitis

CT scan is best if above criteria are not met

Alternative to imaging

ndash OBSERVATION

Once diagnosis of appendicitis has been establishedhellip

Non-ruptured

ndash Short duration of symptoms no suggestion of rupture on imaging

ndash Start antibiotics

ndash Perform appendectomy

Ruptured

ndash Have radiology evaluate for drainable fluid collection

ndash Antibiotics

ndash Operation if above management fails

Once diagnosis of appendicitis has been establishedhellip

Non-ruptured

ndash Short duration of symptoms no suggestion of rupture on imaging

ndash Start antibiotics

ndash Perform appendectomy

Ruptured

ndash Have radiology evaluate for drainable fluid collection

ndash Antibiotics

ndash Operation if above management fails

When should appendectomy be performed in a

patient with non-ruptured acute appendicitis

A Immediately after diagnosis

B Within first 24 hours of hospital stay once appropriate antibiotics and IVF resuscitation have been provided

C After completion of a one-week course of antibiotics

D It is reasonable to treat these patients with antibiotics alone and not perform an appendectomy

Is the appendix a ticking time bomb

Traditional teaching appendicitis is an emergency and should be treated with surgery immediately upon diagnosis

Several studies have now shown that delays of 12 hours up to 48 hours do not increase risk of perforation

Current standard admit IVF resuscitation antibiotics appendectomy during daylight hours

Almstroumlm M1 Svensson JF Patkova B Svenningsson A Wester TAnn Surg 2016 Mar 8 In-hospital

Surgical Delay Does Not Increase the Risk for Perforated Appendicitis in Children A Single-center

Retrospective Cohort Study

Port placement

Lap appy with Endo-loops

Lap appy with stapler

Open appendectomy

Appendix with fecalith

Laparoscopic vs open appendectomy

Laparoscopy is standard of care

Lower complication rate

Less scar

Less pain

Ability to evaluate other intra-abdominal organs if the appendix looks normal

ndash Gallbladder

ndash Ovaries

ndash Inguinal canal

Sauerland S Jaschinski T Neugebauer EA Laparoscopic versus open surgery for

suspected appendicitis Cochrane Database Syst Rev 2010 Oct 6(10)

Post-op course

Most patients can go home on the day of surgery

Recovery is usually quick

Back to school within a week sports within 2 weeks

Very low risk surgery with good outcomes

Risk of infection is about 5 at port sites or in abdomen

You see a 7 year old male in your office with 1-day history of abdominal pain

now localized to RLQ with focal guarding and poor appetite You send him for

ultrasound and it shows appendicitis His mother has heard about some new

research and wants to know if you would recommend just treating with

antibiotics rather than surgery How would you counsel this mom

A This data is preliminary and surgery is still standard of

care

B This may be a reasonable option for her son and she

should discuss it with her surgeon

C If it were your son you wouldnrsquot let him have surgery

D Other

Do all patients with appendicitis require surgery

Pilot study at Nationwide Childrenrsquos Hospital

102 patients

ndash 7 to 17 years of age

ndash Uncomplicated appendicitis defined by

Abdominal pain le 48 hours

White blood cell count le 18000

Ultrasound or CT scan showing appendicitis with an appendix le 11 centimeter thick and no

evidence of abscess or fecalith

Patients and families chose to have appendectomy or antibiotics alone

Non-operative management at least 24 hours of in-hospital observation and IV antibiotics until symptoms improved followed by completion of 10 days of treatment with antibiotics by mouth

Minneci PC Mahida JB et al The effectiveness of patient choice in non-operative

versus surgical management of uncomplicated acute appendicitis JAMA Surgery

2015 Dec 16

Results of non-operative management

65 families chose appendectomy 37 families chose non-operative management

Success rate of non-operative management (defined as not undergoing an appendectomy) 89 at 30 days 76 at 1 year

The 24 who failed did NOT have a higher rate of ruptured appendicitis compared to the patients who had immediate appendectomy

1 year follow-up the children managed non-operatively compared with the surgery group had fewer disability days (8 vs 21 days) lower appendicitis-related health care costs (median $4219 vs $5029) and no difference in health-related quality of life

Non-Operative Treatment of Appendicitis Rationale

Appendectomy is invasive

Children may miss up to two weeks of schoolactivities

Caregivers miss work

Postop complications after appendectomy for uncomplicated appendicitis 5-10

Serious complications (reoperations or readmissions) 1-7

Adult data suggest one-year success rates of 63-85 no difference in rates of complicated appendicitis

bull Meta-analysis of 10 articles reporting 413 children receiving non

operative treatment (NOT) for appendicitis ndash all published in past 10

years

bull 5 comparative

bull 1 RCT

bull 4 case series of NOT

Interpreting meta-analyses Forest plot

Favors

antibiotics

Favors

appendectomy

Overall success of NOT

97 during initial episode

Incidence of recurrent appendicitis during follow-up period (range 2-51

months)

Long term efficacy of NOT (no appendectomy at end of follow-up

period)

LOS shortened by about 05 days in those undergoing appendectomy

compared to NOT

Increased rate of complications for appendectomy compared to NOT

Authorsrsquo Conclusions

ldquoCurrent data suggest that NOT is safe It appears effective as initial treatment in 97 of children with AUA and the rate of recurrent appendicitis is 14rdquo

The study highlights the lack of robust evidence comparing NOT with appendectomy in children

Confirms a position of equipoise between treatment approaches

We recommend that NOT of children with AUA be reserved for those participating in carefully designed research studies

Take-home point

Lap appy is still standard of care for uncomplicated appendicitis

Once diagnosis of appendicitis has been establishedhellip

Non-ruptured

ndash Short duration of symptoms no suggestion of rupture on imaging

ndash Start antibiotics

ndash Perform appendectomy

Ruptured

ndash Have radiology evaluate for drainable fluid collection

ndash Antibiotics

ndash Operation in acute setting if above management fails

ndash Interval appendectomy 6-8 weeks laterhellip

ndash ALTERNATIVELY Just take out the appendix

CT ruptured appendix with abscess

After placement of percutaneous drain

How to counsel families of children with ruptured appendicitis

If kids tolerate a diet pain resolves fever resolves oral antibiotics and home

Plan interval appy 6-8 weeks later

Non-operative management may not work and surgery may be needed (Failure rate 20)

At home they should watch for signs of persistentrecurrent appendicitis

High-anxiety time for patients and families

A 10-year-old boy comes to see you in the office after recent

hospitalization for perforated appendicitis He has 3 more days of

antibiotics left His mom is worried because appetite is poor and his

energy level is low On exam he has diffuse lower abdominal

tenderness You recommend

A Extending course of oral antibiotics

B CT scan to evaluate for persistent or

recurrent appendicitis

C CT scan to evaluate for intra-abdominal

abscess

D Follow-up with surgeon

Is interval appendectomy necessary

In adults many surgeons do not do this operation

In kids data are limited

ndash 2-year follow-up of 96 patients

ndash Perforated appendicitis treated non-operatively with antibiotics

ndash 6 became worse 41 had interval appendectomy

ndash 49 received no further treatment

ndash 57 no recurrence

ndash 43 had recurrence within one month to 2 years

ndash Presence of appendicolith 72 rate of recurrence vs 26 in those without appendicolith

Ein SH1 Langer JC Daneman A Nonoperative management of pediatric ruptured appendix

with inflammatory mass or abscess presence of an appendicolith predicts recurrent

appendicitis J Pediatr Surg 2005 Oct40(10)1612-5

Approach to interval appendectomy

More strongly recommended if fecalith present

Observation is a reasonable option

Best choice for an individual patient depends on their anxiety and parental anxiety

Immediate operation for ruptured appendicitis perhaps a better

option

Meta-analysis operative vs non-operative management of pediatric

ruptured appendicitis

2 RCTs identified

Total of 171 pediatric patients

Compared early vs interval appendectomy

Early appendectomy reduced incidence of adverse event

Ruptured appendicitis may have abscess or phlegmon

Early surgery was more strongly favored when there was no abscess at

time of presentation

Decreased antibiotic duration length of stay and total charges for

abscess and no abscess groups

You are seeing a 7 year old with constipation A

fecalith was seen on abdominal X ray during recent

ED visit How would you counsel the family

A They should be referred to surgery for appendectomy

B Their child is at higher risk for appendicitis so they should be aware of this in case he develops symptoms

C It is uncertain whether this child is at higher risk for appendicitis

D A course of Miralax may help wash out the fecalith

Take home points

Uncomplicated appendicitis

ndash Lap appy is still standard of care

ndash Non-operative management may be an acceptable option but not enough is known about long-term risk of recurrent appendicitis

Complicated appendicitis

ndash Can be managed with immediate operation delayed appendectomy or no appendectomy

ndash Immediate operation is probably more efficient and less stressful for patients and parents

Thank you very much

Questions

Page 6: New approaches for evaluation and treatment of appendicitis · Ultrasound or CT scan showing appendicitis with an appendix ≤ 1.1 centimeter thick and no evidence of abscess or fecalith

Objectives

Why do children get appendicitis

What are typical presenting features

How do we make the diagnosis

How do we decide WHETHER to operate

How do we decide WHEN to operate

What is the difference in how we approach perforated vs non-perforated appendicitis

Does every child with appendicitis actually need surgery

Why does appendicitis occur

Who is more likely to present with perforated

appendicitis

A Younger patients

B Patients with developmental delay

C Uninsured patients

D Minorities

E All of the above

More likely to present with perforated appendicitis

Perforation rates are reported from 20-80 in children

Younger kids

ndash 82 in children under 5

ndash Nearly 100 of 1-year-olds

Developmental delay

Uninsured

Minorities

Kokoska ER1 Bird TM Robbins JM Smith SD Corsi JM Campbell BT Racial disparities

in the management of pediatric appendicitis J Surg Res 2007 Jan137(1)83-8

Which of the following symptoms is LEAST likely to

be consistent with appendicitis

A Fever

B Nauseavomiting

C Diarrhea

D Peri-umbilical pain

E Anorexia

How do children present with appendicitis

Pain starts peri-umbilical and then migrates to RLQ

Pain starts insidiously is persistent and worsens over time

Pain is worse with movement

Fever nausea with or without vomiting and anorexia

Features suggesting alternative diagnoses

ndash Waxing and waning pain

ndash Diarrhea cough sore throat myalgias rhinorrhea and sick contacts

ndash (BUT there are patients with many of the above symptoms who DO have appendicitis)

Differential diagnosis

Virus

Pneumonia

Constipation

In adolescent females

ndash PID

ndash Ovarian cyst

ndash Mittelschmertz

Approach to physical exam in suspected appendicitis

Child does NOT want to move around

Focal tenderness

Percussion of the abdomen causes discomfort (this is better than rebound tenderness)

Psoas obturator Rovsingrsquos heel strike

Watch them walk around

ldquoDoes this child have appendicitisrdquo Meta-Analysis

Fever

ndash If present LR 34 (24-48)

ndash If absent LR 032 (016-064)

Rebound tenderness

ndash If present LR 30 (23-39)

ndash If absent LR 028 (014-055)

Migration of pain LR range 19-31

RLQ pain LR 12 (10-15)

WBC less than 10 LR 022 (017-030)

ANC less than 6750 LR 006 (003-016) Bundy DG Byerley JS Liles EA Perrin EM Katznelson J Rice HE

Does this child have appendicitis JAMA 2007 Jul 25298(4)438-51

Alvarado A (1986) A practical score for the early diagnosis

of acute appendicitis Ann Emerg Med 15557ndash564

You are seeing a child in your office with 1-day history of

RLQ pain You are suspicious for appendicitis Where would

you send the patient

A Emergency room

B Same day outpatient surgery clinic

C CT scan

D Ultrasound

E Other

What to do with suspected appendicitis

Obtain imaging vs send to ED

ndash If any suspicion for viral syndrome with dehydration to ED

ndash If VERY HIGH suspicion for appendicitis and good pediatric radiologist is available consider sending child for ultrasound

Operating Room

further wu per surgeon

Further workup and imaging in

consultation with

Surgery team

1 IV placed 20 cckg bolus CBC

2 Surgical consult within one hour

Strong suspicion on

HPE Equivocal history

physical exam

1 IV placed 20 cckg bolus CBC

2 Re-examine surgical consult within one

hour if still tender

Abdominal pain

ro appendicitis

What is the best way to image the appendix

A CT scan

B Ultrasound

C MRI

D Abdominal X ray

E Other

Ultrasound

CT scan

MRI

Summary of approach to imaging

Ultrasound is good if you have a high pretest probability good radiologist thin patient

MRI is good if you are in a center that uses it routinely to evaluate children with appendicitis

CT scan is best if above criteria are not met

Alternative to imaging

ndash OBSERVATION

Once diagnosis of appendicitis has been establishedhellip

Non-ruptured

ndash Short duration of symptoms no suggestion of rupture on imaging

ndash Start antibiotics

ndash Perform appendectomy

Ruptured

ndash Have radiology evaluate for drainable fluid collection

ndash Antibiotics

ndash Operation if above management fails

Once diagnosis of appendicitis has been establishedhellip

Non-ruptured

ndash Short duration of symptoms no suggestion of rupture on imaging

ndash Start antibiotics

ndash Perform appendectomy

Ruptured

ndash Have radiology evaluate for drainable fluid collection

ndash Antibiotics

ndash Operation if above management fails

When should appendectomy be performed in a

patient with non-ruptured acute appendicitis

A Immediately after diagnosis

B Within first 24 hours of hospital stay once appropriate antibiotics and IVF resuscitation have been provided

C After completion of a one-week course of antibiotics

D It is reasonable to treat these patients with antibiotics alone and not perform an appendectomy

Is the appendix a ticking time bomb

Traditional teaching appendicitis is an emergency and should be treated with surgery immediately upon diagnosis

Several studies have now shown that delays of 12 hours up to 48 hours do not increase risk of perforation

Current standard admit IVF resuscitation antibiotics appendectomy during daylight hours

Almstroumlm M1 Svensson JF Patkova B Svenningsson A Wester TAnn Surg 2016 Mar 8 In-hospital

Surgical Delay Does Not Increase the Risk for Perforated Appendicitis in Children A Single-center

Retrospective Cohort Study

Port placement

Lap appy with Endo-loops

Lap appy with stapler

Open appendectomy

Appendix with fecalith

Laparoscopic vs open appendectomy

Laparoscopy is standard of care

Lower complication rate

Less scar

Less pain

Ability to evaluate other intra-abdominal organs if the appendix looks normal

ndash Gallbladder

ndash Ovaries

ndash Inguinal canal

Sauerland S Jaschinski T Neugebauer EA Laparoscopic versus open surgery for

suspected appendicitis Cochrane Database Syst Rev 2010 Oct 6(10)

Post-op course

Most patients can go home on the day of surgery

Recovery is usually quick

Back to school within a week sports within 2 weeks

Very low risk surgery with good outcomes

Risk of infection is about 5 at port sites or in abdomen

You see a 7 year old male in your office with 1-day history of abdominal pain

now localized to RLQ with focal guarding and poor appetite You send him for

ultrasound and it shows appendicitis His mother has heard about some new

research and wants to know if you would recommend just treating with

antibiotics rather than surgery How would you counsel this mom

A This data is preliminary and surgery is still standard of

care

B This may be a reasonable option for her son and she

should discuss it with her surgeon

C If it were your son you wouldnrsquot let him have surgery

D Other

Do all patients with appendicitis require surgery

Pilot study at Nationwide Childrenrsquos Hospital

102 patients

ndash 7 to 17 years of age

ndash Uncomplicated appendicitis defined by

Abdominal pain le 48 hours

White blood cell count le 18000

Ultrasound or CT scan showing appendicitis with an appendix le 11 centimeter thick and no

evidence of abscess or fecalith

Patients and families chose to have appendectomy or antibiotics alone

Non-operative management at least 24 hours of in-hospital observation and IV antibiotics until symptoms improved followed by completion of 10 days of treatment with antibiotics by mouth

Minneci PC Mahida JB et al The effectiveness of patient choice in non-operative

versus surgical management of uncomplicated acute appendicitis JAMA Surgery

2015 Dec 16

Results of non-operative management

65 families chose appendectomy 37 families chose non-operative management

Success rate of non-operative management (defined as not undergoing an appendectomy) 89 at 30 days 76 at 1 year

The 24 who failed did NOT have a higher rate of ruptured appendicitis compared to the patients who had immediate appendectomy

1 year follow-up the children managed non-operatively compared with the surgery group had fewer disability days (8 vs 21 days) lower appendicitis-related health care costs (median $4219 vs $5029) and no difference in health-related quality of life

Non-Operative Treatment of Appendicitis Rationale

Appendectomy is invasive

Children may miss up to two weeks of schoolactivities

Caregivers miss work

Postop complications after appendectomy for uncomplicated appendicitis 5-10

Serious complications (reoperations or readmissions) 1-7

Adult data suggest one-year success rates of 63-85 no difference in rates of complicated appendicitis

bull Meta-analysis of 10 articles reporting 413 children receiving non

operative treatment (NOT) for appendicitis ndash all published in past 10

years

bull 5 comparative

bull 1 RCT

bull 4 case series of NOT

Interpreting meta-analyses Forest plot

Favors

antibiotics

Favors

appendectomy

Overall success of NOT

97 during initial episode

Incidence of recurrent appendicitis during follow-up period (range 2-51

months)

Long term efficacy of NOT (no appendectomy at end of follow-up

period)

LOS shortened by about 05 days in those undergoing appendectomy

compared to NOT

Increased rate of complications for appendectomy compared to NOT

Authorsrsquo Conclusions

ldquoCurrent data suggest that NOT is safe It appears effective as initial treatment in 97 of children with AUA and the rate of recurrent appendicitis is 14rdquo

The study highlights the lack of robust evidence comparing NOT with appendectomy in children

Confirms a position of equipoise between treatment approaches

We recommend that NOT of children with AUA be reserved for those participating in carefully designed research studies

Take-home point

Lap appy is still standard of care for uncomplicated appendicitis

Once diagnosis of appendicitis has been establishedhellip

Non-ruptured

ndash Short duration of symptoms no suggestion of rupture on imaging

ndash Start antibiotics

ndash Perform appendectomy

Ruptured

ndash Have radiology evaluate for drainable fluid collection

ndash Antibiotics

ndash Operation in acute setting if above management fails

ndash Interval appendectomy 6-8 weeks laterhellip

ndash ALTERNATIVELY Just take out the appendix

CT ruptured appendix with abscess

After placement of percutaneous drain

How to counsel families of children with ruptured appendicitis

If kids tolerate a diet pain resolves fever resolves oral antibiotics and home

Plan interval appy 6-8 weeks later

Non-operative management may not work and surgery may be needed (Failure rate 20)

At home they should watch for signs of persistentrecurrent appendicitis

High-anxiety time for patients and families

A 10-year-old boy comes to see you in the office after recent

hospitalization for perforated appendicitis He has 3 more days of

antibiotics left His mom is worried because appetite is poor and his

energy level is low On exam he has diffuse lower abdominal

tenderness You recommend

A Extending course of oral antibiotics

B CT scan to evaluate for persistent or

recurrent appendicitis

C CT scan to evaluate for intra-abdominal

abscess

D Follow-up with surgeon

Is interval appendectomy necessary

In adults many surgeons do not do this operation

In kids data are limited

ndash 2-year follow-up of 96 patients

ndash Perforated appendicitis treated non-operatively with antibiotics

ndash 6 became worse 41 had interval appendectomy

ndash 49 received no further treatment

ndash 57 no recurrence

ndash 43 had recurrence within one month to 2 years

ndash Presence of appendicolith 72 rate of recurrence vs 26 in those without appendicolith

Ein SH1 Langer JC Daneman A Nonoperative management of pediatric ruptured appendix

with inflammatory mass or abscess presence of an appendicolith predicts recurrent

appendicitis J Pediatr Surg 2005 Oct40(10)1612-5

Approach to interval appendectomy

More strongly recommended if fecalith present

Observation is a reasonable option

Best choice for an individual patient depends on their anxiety and parental anxiety

Immediate operation for ruptured appendicitis perhaps a better

option

Meta-analysis operative vs non-operative management of pediatric

ruptured appendicitis

2 RCTs identified

Total of 171 pediatric patients

Compared early vs interval appendectomy

Early appendectomy reduced incidence of adverse event

Ruptured appendicitis may have abscess or phlegmon

Early surgery was more strongly favored when there was no abscess at

time of presentation

Decreased antibiotic duration length of stay and total charges for

abscess and no abscess groups

You are seeing a 7 year old with constipation A

fecalith was seen on abdominal X ray during recent

ED visit How would you counsel the family

A They should be referred to surgery for appendectomy

B Their child is at higher risk for appendicitis so they should be aware of this in case he develops symptoms

C It is uncertain whether this child is at higher risk for appendicitis

D A course of Miralax may help wash out the fecalith

Take home points

Uncomplicated appendicitis

ndash Lap appy is still standard of care

ndash Non-operative management may be an acceptable option but not enough is known about long-term risk of recurrent appendicitis

Complicated appendicitis

ndash Can be managed with immediate operation delayed appendectomy or no appendectomy

ndash Immediate operation is probably more efficient and less stressful for patients and parents

Thank you very much

Questions

Page 7: New approaches for evaluation and treatment of appendicitis · Ultrasound or CT scan showing appendicitis with an appendix ≤ 1.1 centimeter thick and no evidence of abscess or fecalith

Why does appendicitis occur

Who is more likely to present with perforated

appendicitis

A Younger patients

B Patients with developmental delay

C Uninsured patients

D Minorities

E All of the above

More likely to present with perforated appendicitis

Perforation rates are reported from 20-80 in children

Younger kids

ndash 82 in children under 5

ndash Nearly 100 of 1-year-olds

Developmental delay

Uninsured

Minorities

Kokoska ER1 Bird TM Robbins JM Smith SD Corsi JM Campbell BT Racial disparities

in the management of pediatric appendicitis J Surg Res 2007 Jan137(1)83-8

Which of the following symptoms is LEAST likely to

be consistent with appendicitis

A Fever

B Nauseavomiting

C Diarrhea

D Peri-umbilical pain

E Anorexia

How do children present with appendicitis

Pain starts peri-umbilical and then migrates to RLQ

Pain starts insidiously is persistent and worsens over time

Pain is worse with movement

Fever nausea with or without vomiting and anorexia

Features suggesting alternative diagnoses

ndash Waxing and waning pain

ndash Diarrhea cough sore throat myalgias rhinorrhea and sick contacts

ndash (BUT there are patients with many of the above symptoms who DO have appendicitis)

Differential diagnosis

Virus

Pneumonia

Constipation

In adolescent females

ndash PID

ndash Ovarian cyst

ndash Mittelschmertz

Approach to physical exam in suspected appendicitis

Child does NOT want to move around

Focal tenderness

Percussion of the abdomen causes discomfort (this is better than rebound tenderness)

Psoas obturator Rovsingrsquos heel strike

Watch them walk around

ldquoDoes this child have appendicitisrdquo Meta-Analysis

Fever

ndash If present LR 34 (24-48)

ndash If absent LR 032 (016-064)

Rebound tenderness

ndash If present LR 30 (23-39)

ndash If absent LR 028 (014-055)

Migration of pain LR range 19-31

RLQ pain LR 12 (10-15)

WBC less than 10 LR 022 (017-030)

ANC less than 6750 LR 006 (003-016) Bundy DG Byerley JS Liles EA Perrin EM Katznelson J Rice HE

Does this child have appendicitis JAMA 2007 Jul 25298(4)438-51

Alvarado A (1986) A practical score for the early diagnosis

of acute appendicitis Ann Emerg Med 15557ndash564

You are seeing a child in your office with 1-day history of

RLQ pain You are suspicious for appendicitis Where would

you send the patient

A Emergency room

B Same day outpatient surgery clinic

C CT scan

D Ultrasound

E Other

What to do with suspected appendicitis

Obtain imaging vs send to ED

ndash If any suspicion for viral syndrome with dehydration to ED

ndash If VERY HIGH suspicion for appendicitis and good pediatric radiologist is available consider sending child for ultrasound

Operating Room

further wu per surgeon

Further workup and imaging in

consultation with

Surgery team

1 IV placed 20 cckg bolus CBC

2 Surgical consult within one hour

Strong suspicion on

HPE Equivocal history

physical exam

1 IV placed 20 cckg bolus CBC

2 Re-examine surgical consult within one

hour if still tender

Abdominal pain

ro appendicitis

What is the best way to image the appendix

A CT scan

B Ultrasound

C MRI

D Abdominal X ray

E Other

Ultrasound

CT scan

MRI

Summary of approach to imaging

Ultrasound is good if you have a high pretest probability good radiologist thin patient

MRI is good if you are in a center that uses it routinely to evaluate children with appendicitis

CT scan is best if above criteria are not met

Alternative to imaging

ndash OBSERVATION

Once diagnosis of appendicitis has been establishedhellip

Non-ruptured

ndash Short duration of symptoms no suggestion of rupture on imaging

ndash Start antibiotics

ndash Perform appendectomy

Ruptured

ndash Have radiology evaluate for drainable fluid collection

ndash Antibiotics

ndash Operation if above management fails

Once diagnosis of appendicitis has been establishedhellip

Non-ruptured

ndash Short duration of symptoms no suggestion of rupture on imaging

ndash Start antibiotics

ndash Perform appendectomy

Ruptured

ndash Have radiology evaluate for drainable fluid collection

ndash Antibiotics

ndash Operation if above management fails

When should appendectomy be performed in a

patient with non-ruptured acute appendicitis

A Immediately after diagnosis

B Within first 24 hours of hospital stay once appropriate antibiotics and IVF resuscitation have been provided

C After completion of a one-week course of antibiotics

D It is reasonable to treat these patients with antibiotics alone and not perform an appendectomy

Is the appendix a ticking time bomb

Traditional teaching appendicitis is an emergency and should be treated with surgery immediately upon diagnosis

Several studies have now shown that delays of 12 hours up to 48 hours do not increase risk of perforation

Current standard admit IVF resuscitation antibiotics appendectomy during daylight hours

Almstroumlm M1 Svensson JF Patkova B Svenningsson A Wester TAnn Surg 2016 Mar 8 In-hospital

Surgical Delay Does Not Increase the Risk for Perforated Appendicitis in Children A Single-center

Retrospective Cohort Study

Port placement

Lap appy with Endo-loops

Lap appy with stapler

Open appendectomy

Appendix with fecalith

Laparoscopic vs open appendectomy

Laparoscopy is standard of care

Lower complication rate

Less scar

Less pain

Ability to evaluate other intra-abdominal organs if the appendix looks normal

ndash Gallbladder

ndash Ovaries

ndash Inguinal canal

Sauerland S Jaschinski T Neugebauer EA Laparoscopic versus open surgery for

suspected appendicitis Cochrane Database Syst Rev 2010 Oct 6(10)

Post-op course

Most patients can go home on the day of surgery

Recovery is usually quick

Back to school within a week sports within 2 weeks

Very low risk surgery with good outcomes

Risk of infection is about 5 at port sites or in abdomen

You see a 7 year old male in your office with 1-day history of abdominal pain

now localized to RLQ with focal guarding and poor appetite You send him for

ultrasound and it shows appendicitis His mother has heard about some new

research and wants to know if you would recommend just treating with

antibiotics rather than surgery How would you counsel this mom

A This data is preliminary and surgery is still standard of

care

B This may be a reasonable option for her son and she

should discuss it with her surgeon

C If it were your son you wouldnrsquot let him have surgery

D Other

Do all patients with appendicitis require surgery

Pilot study at Nationwide Childrenrsquos Hospital

102 patients

ndash 7 to 17 years of age

ndash Uncomplicated appendicitis defined by

Abdominal pain le 48 hours

White blood cell count le 18000

Ultrasound or CT scan showing appendicitis with an appendix le 11 centimeter thick and no

evidence of abscess or fecalith

Patients and families chose to have appendectomy or antibiotics alone

Non-operative management at least 24 hours of in-hospital observation and IV antibiotics until symptoms improved followed by completion of 10 days of treatment with antibiotics by mouth

Minneci PC Mahida JB et al The effectiveness of patient choice in non-operative

versus surgical management of uncomplicated acute appendicitis JAMA Surgery

2015 Dec 16

Results of non-operative management

65 families chose appendectomy 37 families chose non-operative management

Success rate of non-operative management (defined as not undergoing an appendectomy) 89 at 30 days 76 at 1 year

The 24 who failed did NOT have a higher rate of ruptured appendicitis compared to the patients who had immediate appendectomy

1 year follow-up the children managed non-operatively compared with the surgery group had fewer disability days (8 vs 21 days) lower appendicitis-related health care costs (median $4219 vs $5029) and no difference in health-related quality of life

Non-Operative Treatment of Appendicitis Rationale

Appendectomy is invasive

Children may miss up to two weeks of schoolactivities

Caregivers miss work

Postop complications after appendectomy for uncomplicated appendicitis 5-10

Serious complications (reoperations or readmissions) 1-7

Adult data suggest one-year success rates of 63-85 no difference in rates of complicated appendicitis

bull Meta-analysis of 10 articles reporting 413 children receiving non

operative treatment (NOT) for appendicitis ndash all published in past 10

years

bull 5 comparative

bull 1 RCT

bull 4 case series of NOT

Interpreting meta-analyses Forest plot

Favors

antibiotics

Favors

appendectomy

Overall success of NOT

97 during initial episode

Incidence of recurrent appendicitis during follow-up period (range 2-51

months)

Long term efficacy of NOT (no appendectomy at end of follow-up

period)

LOS shortened by about 05 days in those undergoing appendectomy

compared to NOT

Increased rate of complications for appendectomy compared to NOT

Authorsrsquo Conclusions

ldquoCurrent data suggest that NOT is safe It appears effective as initial treatment in 97 of children with AUA and the rate of recurrent appendicitis is 14rdquo

The study highlights the lack of robust evidence comparing NOT with appendectomy in children

Confirms a position of equipoise between treatment approaches

We recommend that NOT of children with AUA be reserved for those participating in carefully designed research studies

Take-home point

Lap appy is still standard of care for uncomplicated appendicitis

Once diagnosis of appendicitis has been establishedhellip

Non-ruptured

ndash Short duration of symptoms no suggestion of rupture on imaging

ndash Start antibiotics

ndash Perform appendectomy

Ruptured

ndash Have radiology evaluate for drainable fluid collection

ndash Antibiotics

ndash Operation in acute setting if above management fails

ndash Interval appendectomy 6-8 weeks laterhellip

ndash ALTERNATIVELY Just take out the appendix

CT ruptured appendix with abscess

After placement of percutaneous drain

How to counsel families of children with ruptured appendicitis

If kids tolerate a diet pain resolves fever resolves oral antibiotics and home

Plan interval appy 6-8 weeks later

Non-operative management may not work and surgery may be needed (Failure rate 20)

At home they should watch for signs of persistentrecurrent appendicitis

High-anxiety time for patients and families

A 10-year-old boy comes to see you in the office after recent

hospitalization for perforated appendicitis He has 3 more days of

antibiotics left His mom is worried because appetite is poor and his

energy level is low On exam he has diffuse lower abdominal

tenderness You recommend

A Extending course of oral antibiotics

B CT scan to evaluate for persistent or

recurrent appendicitis

C CT scan to evaluate for intra-abdominal

abscess

D Follow-up with surgeon

Is interval appendectomy necessary

In adults many surgeons do not do this operation

In kids data are limited

ndash 2-year follow-up of 96 patients

ndash Perforated appendicitis treated non-operatively with antibiotics

ndash 6 became worse 41 had interval appendectomy

ndash 49 received no further treatment

ndash 57 no recurrence

ndash 43 had recurrence within one month to 2 years

ndash Presence of appendicolith 72 rate of recurrence vs 26 in those without appendicolith

Ein SH1 Langer JC Daneman A Nonoperative management of pediatric ruptured appendix

with inflammatory mass or abscess presence of an appendicolith predicts recurrent

appendicitis J Pediatr Surg 2005 Oct40(10)1612-5

Approach to interval appendectomy

More strongly recommended if fecalith present

Observation is a reasonable option

Best choice for an individual patient depends on their anxiety and parental anxiety

Immediate operation for ruptured appendicitis perhaps a better

option

Meta-analysis operative vs non-operative management of pediatric

ruptured appendicitis

2 RCTs identified

Total of 171 pediatric patients

Compared early vs interval appendectomy

Early appendectomy reduced incidence of adverse event

Ruptured appendicitis may have abscess or phlegmon

Early surgery was more strongly favored when there was no abscess at

time of presentation

Decreased antibiotic duration length of stay and total charges for

abscess and no abscess groups

You are seeing a 7 year old with constipation A

fecalith was seen on abdominal X ray during recent

ED visit How would you counsel the family

A They should be referred to surgery for appendectomy

B Their child is at higher risk for appendicitis so they should be aware of this in case he develops symptoms

C It is uncertain whether this child is at higher risk for appendicitis

D A course of Miralax may help wash out the fecalith

Take home points

Uncomplicated appendicitis

ndash Lap appy is still standard of care

ndash Non-operative management may be an acceptable option but not enough is known about long-term risk of recurrent appendicitis

Complicated appendicitis

ndash Can be managed with immediate operation delayed appendectomy or no appendectomy

ndash Immediate operation is probably more efficient and less stressful for patients and parents

Thank you very much

Questions

Page 8: New approaches for evaluation and treatment of appendicitis · Ultrasound or CT scan showing appendicitis with an appendix ≤ 1.1 centimeter thick and no evidence of abscess or fecalith

Who is more likely to present with perforated

appendicitis

A Younger patients

B Patients with developmental delay

C Uninsured patients

D Minorities

E All of the above

More likely to present with perforated appendicitis

Perforation rates are reported from 20-80 in children

Younger kids

ndash 82 in children under 5

ndash Nearly 100 of 1-year-olds

Developmental delay

Uninsured

Minorities

Kokoska ER1 Bird TM Robbins JM Smith SD Corsi JM Campbell BT Racial disparities

in the management of pediatric appendicitis J Surg Res 2007 Jan137(1)83-8

Which of the following symptoms is LEAST likely to

be consistent with appendicitis

A Fever

B Nauseavomiting

C Diarrhea

D Peri-umbilical pain

E Anorexia

How do children present with appendicitis

Pain starts peri-umbilical and then migrates to RLQ

Pain starts insidiously is persistent and worsens over time

Pain is worse with movement

Fever nausea with or without vomiting and anorexia

Features suggesting alternative diagnoses

ndash Waxing and waning pain

ndash Diarrhea cough sore throat myalgias rhinorrhea and sick contacts

ndash (BUT there are patients with many of the above symptoms who DO have appendicitis)

Differential diagnosis

Virus

Pneumonia

Constipation

In adolescent females

ndash PID

ndash Ovarian cyst

ndash Mittelschmertz

Approach to physical exam in suspected appendicitis

Child does NOT want to move around

Focal tenderness

Percussion of the abdomen causes discomfort (this is better than rebound tenderness)

Psoas obturator Rovsingrsquos heel strike

Watch them walk around

ldquoDoes this child have appendicitisrdquo Meta-Analysis

Fever

ndash If present LR 34 (24-48)

ndash If absent LR 032 (016-064)

Rebound tenderness

ndash If present LR 30 (23-39)

ndash If absent LR 028 (014-055)

Migration of pain LR range 19-31

RLQ pain LR 12 (10-15)

WBC less than 10 LR 022 (017-030)

ANC less than 6750 LR 006 (003-016) Bundy DG Byerley JS Liles EA Perrin EM Katznelson J Rice HE

Does this child have appendicitis JAMA 2007 Jul 25298(4)438-51

Alvarado A (1986) A practical score for the early diagnosis

of acute appendicitis Ann Emerg Med 15557ndash564

You are seeing a child in your office with 1-day history of

RLQ pain You are suspicious for appendicitis Where would

you send the patient

A Emergency room

B Same day outpatient surgery clinic

C CT scan

D Ultrasound

E Other

What to do with suspected appendicitis

Obtain imaging vs send to ED

ndash If any suspicion for viral syndrome with dehydration to ED

ndash If VERY HIGH suspicion for appendicitis and good pediatric radiologist is available consider sending child for ultrasound

Operating Room

further wu per surgeon

Further workup and imaging in

consultation with

Surgery team

1 IV placed 20 cckg bolus CBC

2 Surgical consult within one hour

Strong suspicion on

HPE Equivocal history

physical exam

1 IV placed 20 cckg bolus CBC

2 Re-examine surgical consult within one

hour if still tender

Abdominal pain

ro appendicitis

What is the best way to image the appendix

A CT scan

B Ultrasound

C MRI

D Abdominal X ray

E Other

Ultrasound

CT scan

MRI

Summary of approach to imaging

Ultrasound is good if you have a high pretest probability good radiologist thin patient

MRI is good if you are in a center that uses it routinely to evaluate children with appendicitis

CT scan is best if above criteria are not met

Alternative to imaging

ndash OBSERVATION

Once diagnosis of appendicitis has been establishedhellip

Non-ruptured

ndash Short duration of symptoms no suggestion of rupture on imaging

ndash Start antibiotics

ndash Perform appendectomy

Ruptured

ndash Have radiology evaluate for drainable fluid collection

ndash Antibiotics

ndash Operation if above management fails

Once diagnosis of appendicitis has been establishedhellip

Non-ruptured

ndash Short duration of symptoms no suggestion of rupture on imaging

ndash Start antibiotics

ndash Perform appendectomy

Ruptured

ndash Have radiology evaluate for drainable fluid collection

ndash Antibiotics

ndash Operation if above management fails

When should appendectomy be performed in a

patient with non-ruptured acute appendicitis

A Immediately after diagnosis

B Within first 24 hours of hospital stay once appropriate antibiotics and IVF resuscitation have been provided

C After completion of a one-week course of antibiotics

D It is reasonable to treat these patients with antibiotics alone and not perform an appendectomy

Is the appendix a ticking time bomb

Traditional teaching appendicitis is an emergency and should be treated with surgery immediately upon diagnosis

Several studies have now shown that delays of 12 hours up to 48 hours do not increase risk of perforation

Current standard admit IVF resuscitation antibiotics appendectomy during daylight hours

Almstroumlm M1 Svensson JF Patkova B Svenningsson A Wester TAnn Surg 2016 Mar 8 In-hospital

Surgical Delay Does Not Increase the Risk for Perforated Appendicitis in Children A Single-center

Retrospective Cohort Study

Port placement

Lap appy with Endo-loops

Lap appy with stapler

Open appendectomy

Appendix with fecalith

Laparoscopic vs open appendectomy

Laparoscopy is standard of care

Lower complication rate

Less scar

Less pain

Ability to evaluate other intra-abdominal organs if the appendix looks normal

ndash Gallbladder

ndash Ovaries

ndash Inguinal canal

Sauerland S Jaschinski T Neugebauer EA Laparoscopic versus open surgery for

suspected appendicitis Cochrane Database Syst Rev 2010 Oct 6(10)

Post-op course

Most patients can go home on the day of surgery

Recovery is usually quick

Back to school within a week sports within 2 weeks

Very low risk surgery with good outcomes

Risk of infection is about 5 at port sites or in abdomen

You see a 7 year old male in your office with 1-day history of abdominal pain

now localized to RLQ with focal guarding and poor appetite You send him for

ultrasound and it shows appendicitis His mother has heard about some new

research and wants to know if you would recommend just treating with

antibiotics rather than surgery How would you counsel this mom

A This data is preliminary and surgery is still standard of

care

B This may be a reasonable option for her son and she

should discuss it with her surgeon

C If it were your son you wouldnrsquot let him have surgery

D Other

Do all patients with appendicitis require surgery

Pilot study at Nationwide Childrenrsquos Hospital

102 patients

ndash 7 to 17 years of age

ndash Uncomplicated appendicitis defined by

Abdominal pain le 48 hours

White blood cell count le 18000

Ultrasound or CT scan showing appendicitis with an appendix le 11 centimeter thick and no

evidence of abscess or fecalith

Patients and families chose to have appendectomy or antibiotics alone

Non-operative management at least 24 hours of in-hospital observation and IV antibiotics until symptoms improved followed by completion of 10 days of treatment with antibiotics by mouth

Minneci PC Mahida JB et al The effectiveness of patient choice in non-operative

versus surgical management of uncomplicated acute appendicitis JAMA Surgery

2015 Dec 16

Results of non-operative management

65 families chose appendectomy 37 families chose non-operative management

Success rate of non-operative management (defined as not undergoing an appendectomy) 89 at 30 days 76 at 1 year

The 24 who failed did NOT have a higher rate of ruptured appendicitis compared to the patients who had immediate appendectomy

1 year follow-up the children managed non-operatively compared with the surgery group had fewer disability days (8 vs 21 days) lower appendicitis-related health care costs (median $4219 vs $5029) and no difference in health-related quality of life

Non-Operative Treatment of Appendicitis Rationale

Appendectomy is invasive

Children may miss up to two weeks of schoolactivities

Caregivers miss work

Postop complications after appendectomy for uncomplicated appendicitis 5-10

Serious complications (reoperations or readmissions) 1-7

Adult data suggest one-year success rates of 63-85 no difference in rates of complicated appendicitis

bull Meta-analysis of 10 articles reporting 413 children receiving non

operative treatment (NOT) for appendicitis ndash all published in past 10

years

bull 5 comparative

bull 1 RCT

bull 4 case series of NOT

Interpreting meta-analyses Forest plot

Favors

antibiotics

Favors

appendectomy

Overall success of NOT

97 during initial episode

Incidence of recurrent appendicitis during follow-up period (range 2-51

months)

Long term efficacy of NOT (no appendectomy at end of follow-up

period)

LOS shortened by about 05 days in those undergoing appendectomy

compared to NOT

Increased rate of complications for appendectomy compared to NOT

Authorsrsquo Conclusions

ldquoCurrent data suggest that NOT is safe It appears effective as initial treatment in 97 of children with AUA and the rate of recurrent appendicitis is 14rdquo

The study highlights the lack of robust evidence comparing NOT with appendectomy in children

Confirms a position of equipoise between treatment approaches

We recommend that NOT of children with AUA be reserved for those participating in carefully designed research studies

Take-home point

Lap appy is still standard of care for uncomplicated appendicitis

Once diagnosis of appendicitis has been establishedhellip

Non-ruptured

ndash Short duration of symptoms no suggestion of rupture on imaging

ndash Start antibiotics

ndash Perform appendectomy

Ruptured

ndash Have radiology evaluate for drainable fluid collection

ndash Antibiotics

ndash Operation in acute setting if above management fails

ndash Interval appendectomy 6-8 weeks laterhellip

ndash ALTERNATIVELY Just take out the appendix

CT ruptured appendix with abscess

After placement of percutaneous drain

How to counsel families of children with ruptured appendicitis

If kids tolerate a diet pain resolves fever resolves oral antibiotics and home

Plan interval appy 6-8 weeks later

Non-operative management may not work and surgery may be needed (Failure rate 20)

At home they should watch for signs of persistentrecurrent appendicitis

High-anxiety time for patients and families

A 10-year-old boy comes to see you in the office after recent

hospitalization for perforated appendicitis He has 3 more days of

antibiotics left His mom is worried because appetite is poor and his

energy level is low On exam he has diffuse lower abdominal

tenderness You recommend

A Extending course of oral antibiotics

B CT scan to evaluate for persistent or

recurrent appendicitis

C CT scan to evaluate for intra-abdominal

abscess

D Follow-up with surgeon

Is interval appendectomy necessary

In adults many surgeons do not do this operation

In kids data are limited

ndash 2-year follow-up of 96 patients

ndash Perforated appendicitis treated non-operatively with antibiotics

ndash 6 became worse 41 had interval appendectomy

ndash 49 received no further treatment

ndash 57 no recurrence

ndash 43 had recurrence within one month to 2 years

ndash Presence of appendicolith 72 rate of recurrence vs 26 in those without appendicolith

Ein SH1 Langer JC Daneman A Nonoperative management of pediatric ruptured appendix

with inflammatory mass or abscess presence of an appendicolith predicts recurrent

appendicitis J Pediatr Surg 2005 Oct40(10)1612-5

Approach to interval appendectomy

More strongly recommended if fecalith present

Observation is a reasonable option

Best choice for an individual patient depends on their anxiety and parental anxiety

Immediate operation for ruptured appendicitis perhaps a better

option

Meta-analysis operative vs non-operative management of pediatric

ruptured appendicitis

2 RCTs identified

Total of 171 pediatric patients

Compared early vs interval appendectomy

Early appendectomy reduced incidence of adverse event

Ruptured appendicitis may have abscess or phlegmon

Early surgery was more strongly favored when there was no abscess at

time of presentation

Decreased antibiotic duration length of stay and total charges for

abscess and no abscess groups

You are seeing a 7 year old with constipation A

fecalith was seen on abdominal X ray during recent

ED visit How would you counsel the family

A They should be referred to surgery for appendectomy

B Their child is at higher risk for appendicitis so they should be aware of this in case he develops symptoms

C It is uncertain whether this child is at higher risk for appendicitis

D A course of Miralax may help wash out the fecalith

Take home points

Uncomplicated appendicitis

ndash Lap appy is still standard of care

ndash Non-operative management may be an acceptable option but not enough is known about long-term risk of recurrent appendicitis

Complicated appendicitis

ndash Can be managed with immediate operation delayed appendectomy or no appendectomy

ndash Immediate operation is probably more efficient and less stressful for patients and parents

Thank you very much

Questions

Page 9: New approaches for evaluation and treatment of appendicitis · Ultrasound or CT scan showing appendicitis with an appendix ≤ 1.1 centimeter thick and no evidence of abscess or fecalith

More likely to present with perforated appendicitis

Perforation rates are reported from 20-80 in children

Younger kids

ndash 82 in children under 5

ndash Nearly 100 of 1-year-olds

Developmental delay

Uninsured

Minorities

Kokoska ER1 Bird TM Robbins JM Smith SD Corsi JM Campbell BT Racial disparities

in the management of pediatric appendicitis J Surg Res 2007 Jan137(1)83-8

Which of the following symptoms is LEAST likely to

be consistent with appendicitis

A Fever

B Nauseavomiting

C Diarrhea

D Peri-umbilical pain

E Anorexia

How do children present with appendicitis

Pain starts peri-umbilical and then migrates to RLQ

Pain starts insidiously is persistent and worsens over time

Pain is worse with movement

Fever nausea with or without vomiting and anorexia

Features suggesting alternative diagnoses

ndash Waxing and waning pain

ndash Diarrhea cough sore throat myalgias rhinorrhea and sick contacts

ndash (BUT there are patients with many of the above symptoms who DO have appendicitis)

Differential diagnosis

Virus

Pneumonia

Constipation

In adolescent females

ndash PID

ndash Ovarian cyst

ndash Mittelschmertz

Approach to physical exam in suspected appendicitis

Child does NOT want to move around

Focal tenderness

Percussion of the abdomen causes discomfort (this is better than rebound tenderness)

Psoas obturator Rovsingrsquos heel strike

Watch them walk around

ldquoDoes this child have appendicitisrdquo Meta-Analysis

Fever

ndash If present LR 34 (24-48)

ndash If absent LR 032 (016-064)

Rebound tenderness

ndash If present LR 30 (23-39)

ndash If absent LR 028 (014-055)

Migration of pain LR range 19-31

RLQ pain LR 12 (10-15)

WBC less than 10 LR 022 (017-030)

ANC less than 6750 LR 006 (003-016) Bundy DG Byerley JS Liles EA Perrin EM Katznelson J Rice HE

Does this child have appendicitis JAMA 2007 Jul 25298(4)438-51

Alvarado A (1986) A practical score for the early diagnosis

of acute appendicitis Ann Emerg Med 15557ndash564

You are seeing a child in your office with 1-day history of

RLQ pain You are suspicious for appendicitis Where would

you send the patient

A Emergency room

B Same day outpatient surgery clinic

C CT scan

D Ultrasound

E Other

What to do with suspected appendicitis

Obtain imaging vs send to ED

ndash If any suspicion for viral syndrome with dehydration to ED

ndash If VERY HIGH suspicion for appendicitis and good pediatric radiologist is available consider sending child for ultrasound

Operating Room

further wu per surgeon

Further workup and imaging in

consultation with

Surgery team

1 IV placed 20 cckg bolus CBC

2 Surgical consult within one hour

Strong suspicion on

HPE Equivocal history

physical exam

1 IV placed 20 cckg bolus CBC

2 Re-examine surgical consult within one

hour if still tender

Abdominal pain

ro appendicitis

What is the best way to image the appendix

A CT scan

B Ultrasound

C MRI

D Abdominal X ray

E Other

Ultrasound

CT scan

MRI

Summary of approach to imaging

Ultrasound is good if you have a high pretest probability good radiologist thin patient

MRI is good if you are in a center that uses it routinely to evaluate children with appendicitis

CT scan is best if above criteria are not met

Alternative to imaging

ndash OBSERVATION

Once diagnosis of appendicitis has been establishedhellip

Non-ruptured

ndash Short duration of symptoms no suggestion of rupture on imaging

ndash Start antibiotics

ndash Perform appendectomy

Ruptured

ndash Have radiology evaluate for drainable fluid collection

ndash Antibiotics

ndash Operation if above management fails

Once diagnosis of appendicitis has been establishedhellip

Non-ruptured

ndash Short duration of symptoms no suggestion of rupture on imaging

ndash Start antibiotics

ndash Perform appendectomy

Ruptured

ndash Have radiology evaluate for drainable fluid collection

ndash Antibiotics

ndash Operation if above management fails

When should appendectomy be performed in a

patient with non-ruptured acute appendicitis

A Immediately after diagnosis

B Within first 24 hours of hospital stay once appropriate antibiotics and IVF resuscitation have been provided

C After completion of a one-week course of antibiotics

D It is reasonable to treat these patients with antibiotics alone and not perform an appendectomy

Is the appendix a ticking time bomb

Traditional teaching appendicitis is an emergency and should be treated with surgery immediately upon diagnosis

Several studies have now shown that delays of 12 hours up to 48 hours do not increase risk of perforation

Current standard admit IVF resuscitation antibiotics appendectomy during daylight hours

Almstroumlm M1 Svensson JF Patkova B Svenningsson A Wester TAnn Surg 2016 Mar 8 In-hospital

Surgical Delay Does Not Increase the Risk for Perforated Appendicitis in Children A Single-center

Retrospective Cohort Study

Port placement

Lap appy with Endo-loops

Lap appy with stapler

Open appendectomy

Appendix with fecalith

Laparoscopic vs open appendectomy

Laparoscopy is standard of care

Lower complication rate

Less scar

Less pain

Ability to evaluate other intra-abdominal organs if the appendix looks normal

ndash Gallbladder

ndash Ovaries

ndash Inguinal canal

Sauerland S Jaschinski T Neugebauer EA Laparoscopic versus open surgery for

suspected appendicitis Cochrane Database Syst Rev 2010 Oct 6(10)

Post-op course

Most patients can go home on the day of surgery

Recovery is usually quick

Back to school within a week sports within 2 weeks

Very low risk surgery with good outcomes

Risk of infection is about 5 at port sites or in abdomen

You see a 7 year old male in your office with 1-day history of abdominal pain

now localized to RLQ with focal guarding and poor appetite You send him for

ultrasound and it shows appendicitis His mother has heard about some new

research and wants to know if you would recommend just treating with

antibiotics rather than surgery How would you counsel this mom

A This data is preliminary and surgery is still standard of

care

B This may be a reasonable option for her son and she

should discuss it with her surgeon

C If it were your son you wouldnrsquot let him have surgery

D Other

Do all patients with appendicitis require surgery

Pilot study at Nationwide Childrenrsquos Hospital

102 patients

ndash 7 to 17 years of age

ndash Uncomplicated appendicitis defined by

Abdominal pain le 48 hours

White blood cell count le 18000

Ultrasound or CT scan showing appendicitis with an appendix le 11 centimeter thick and no

evidence of abscess or fecalith

Patients and families chose to have appendectomy or antibiotics alone

Non-operative management at least 24 hours of in-hospital observation and IV antibiotics until symptoms improved followed by completion of 10 days of treatment with antibiotics by mouth

Minneci PC Mahida JB et al The effectiveness of patient choice in non-operative

versus surgical management of uncomplicated acute appendicitis JAMA Surgery

2015 Dec 16

Results of non-operative management

65 families chose appendectomy 37 families chose non-operative management

Success rate of non-operative management (defined as not undergoing an appendectomy) 89 at 30 days 76 at 1 year

The 24 who failed did NOT have a higher rate of ruptured appendicitis compared to the patients who had immediate appendectomy

1 year follow-up the children managed non-operatively compared with the surgery group had fewer disability days (8 vs 21 days) lower appendicitis-related health care costs (median $4219 vs $5029) and no difference in health-related quality of life

Non-Operative Treatment of Appendicitis Rationale

Appendectomy is invasive

Children may miss up to two weeks of schoolactivities

Caregivers miss work

Postop complications after appendectomy for uncomplicated appendicitis 5-10

Serious complications (reoperations or readmissions) 1-7

Adult data suggest one-year success rates of 63-85 no difference in rates of complicated appendicitis

bull Meta-analysis of 10 articles reporting 413 children receiving non

operative treatment (NOT) for appendicitis ndash all published in past 10

years

bull 5 comparative

bull 1 RCT

bull 4 case series of NOT

Interpreting meta-analyses Forest plot

Favors

antibiotics

Favors

appendectomy

Overall success of NOT

97 during initial episode

Incidence of recurrent appendicitis during follow-up period (range 2-51

months)

Long term efficacy of NOT (no appendectomy at end of follow-up

period)

LOS shortened by about 05 days in those undergoing appendectomy

compared to NOT

Increased rate of complications for appendectomy compared to NOT

Authorsrsquo Conclusions

ldquoCurrent data suggest that NOT is safe It appears effective as initial treatment in 97 of children with AUA and the rate of recurrent appendicitis is 14rdquo

The study highlights the lack of robust evidence comparing NOT with appendectomy in children

Confirms a position of equipoise between treatment approaches

We recommend that NOT of children with AUA be reserved for those participating in carefully designed research studies

Take-home point

Lap appy is still standard of care for uncomplicated appendicitis

Once diagnosis of appendicitis has been establishedhellip

Non-ruptured

ndash Short duration of symptoms no suggestion of rupture on imaging

ndash Start antibiotics

ndash Perform appendectomy

Ruptured

ndash Have radiology evaluate for drainable fluid collection

ndash Antibiotics

ndash Operation in acute setting if above management fails

ndash Interval appendectomy 6-8 weeks laterhellip

ndash ALTERNATIVELY Just take out the appendix

CT ruptured appendix with abscess

After placement of percutaneous drain

How to counsel families of children with ruptured appendicitis

If kids tolerate a diet pain resolves fever resolves oral antibiotics and home

Plan interval appy 6-8 weeks later

Non-operative management may not work and surgery may be needed (Failure rate 20)

At home they should watch for signs of persistentrecurrent appendicitis

High-anxiety time for patients and families

A 10-year-old boy comes to see you in the office after recent

hospitalization for perforated appendicitis He has 3 more days of

antibiotics left His mom is worried because appetite is poor and his

energy level is low On exam he has diffuse lower abdominal

tenderness You recommend

A Extending course of oral antibiotics

B CT scan to evaluate for persistent or

recurrent appendicitis

C CT scan to evaluate for intra-abdominal

abscess

D Follow-up with surgeon

Is interval appendectomy necessary

In adults many surgeons do not do this operation

In kids data are limited

ndash 2-year follow-up of 96 patients

ndash Perforated appendicitis treated non-operatively with antibiotics

ndash 6 became worse 41 had interval appendectomy

ndash 49 received no further treatment

ndash 57 no recurrence

ndash 43 had recurrence within one month to 2 years

ndash Presence of appendicolith 72 rate of recurrence vs 26 in those without appendicolith

Ein SH1 Langer JC Daneman A Nonoperative management of pediatric ruptured appendix

with inflammatory mass or abscess presence of an appendicolith predicts recurrent

appendicitis J Pediatr Surg 2005 Oct40(10)1612-5

Approach to interval appendectomy

More strongly recommended if fecalith present

Observation is a reasonable option

Best choice for an individual patient depends on their anxiety and parental anxiety

Immediate operation for ruptured appendicitis perhaps a better

option

Meta-analysis operative vs non-operative management of pediatric

ruptured appendicitis

2 RCTs identified

Total of 171 pediatric patients

Compared early vs interval appendectomy

Early appendectomy reduced incidence of adverse event

Ruptured appendicitis may have abscess or phlegmon

Early surgery was more strongly favored when there was no abscess at

time of presentation

Decreased antibiotic duration length of stay and total charges for

abscess and no abscess groups

You are seeing a 7 year old with constipation A

fecalith was seen on abdominal X ray during recent

ED visit How would you counsel the family

A They should be referred to surgery for appendectomy

B Their child is at higher risk for appendicitis so they should be aware of this in case he develops symptoms

C It is uncertain whether this child is at higher risk for appendicitis

D A course of Miralax may help wash out the fecalith

Take home points

Uncomplicated appendicitis

ndash Lap appy is still standard of care

ndash Non-operative management may be an acceptable option but not enough is known about long-term risk of recurrent appendicitis

Complicated appendicitis

ndash Can be managed with immediate operation delayed appendectomy or no appendectomy

ndash Immediate operation is probably more efficient and less stressful for patients and parents

Thank you very much

Questions

Page 10: New approaches for evaluation and treatment of appendicitis · Ultrasound or CT scan showing appendicitis with an appendix ≤ 1.1 centimeter thick and no evidence of abscess or fecalith

Which of the following symptoms is LEAST likely to

be consistent with appendicitis

A Fever

B Nauseavomiting

C Diarrhea

D Peri-umbilical pain

E Anorexia

How do children present with appendicitis

Pain starts peri-umbilical and then migrates to RLQ

Pain starts insidiously is persistent and worsens over time

Pain is worse with movement

Fever nausea with or without vomiting and anorexia

Features suggesting alternative diagnoses

ndash Waxing and waning pain

ndash Diarrhea cough sore throat myalgias rhinorrhea and sick contacts

ndash (BUT there are patients with many of the above symptoms who DO have appendicitis)

Differential diagnosis

Virus

Pneumonia

Constipation

In adolescent females

ndash PID

ndash Ovarian cyst

ndash Mittelschmertz

Approach to physical exam in suspected appendicitis

Child does NOT want to move around

Focal tenderness

Percussion of the abdomen causes discomfort (this is better than rebound tenderness)

Psoas obturator Rovsingrsquos heel strike

Watch them walk around

ldquoDoes this child have appendicitisrdquo Meta-Analysis

Fever

ndash If present LR 34 (24-48)

ndash If absent LR 032 (016-064)

Rebound tenderness

ndash If present LR 30 (23-39)

ndash If absent LR 028 (014-055)

Migration of pain LR range 19-31

RLQ pain LR 12 (10-15)

WBC less than 10 LR 022 (017-030)

ANC less than 6750 LR 006 (003-016) Bundy DG Byerley JS Liles EA Perrin EM Katznelson J Rice HE

Does this child have appendicitis JAMA 2007 Jul 25298(4)438-51

Alvarado A (1986) A practical score for the early diagnosis

of acute appendicitis Ann Emerg Med 15557ndash564

You are seeing a child in your office with 1-day history of

RLQ pain You are suspicious for appendicitis Where would

you send the patient

A Emergency room

B Same day outpatient surgery clinic

C CT scan

D Ultrasound

E Other

What to do with suspected appendicitis

Obtain imaging vs send to ED

ndash If any suspicion for viral syndrome with dehydration to ED

ndash If VERY HIGH suspicion for appendicitis and good pediatric radiologist is available consider sending child for ultrasound

Operating Room

further wu per surgeon

Further workup and imaging in

consultation with

Surgery team

1 IV placed 20 cckg bolus CBC

2 Surgical consult within one hour

Strong suspicion on

HPE Equivocal history

physical exam

1 IV placed 20 cckg bolus CBC

2 Re-examine surgical consult within one

hour if still tender

Abdominal pain

ro appendicitis

What is the best way to image the appendix

A CT scan

B Ultrasound

C MRI

D Abdominal X ray

E Other

Ultrasound

CT scan

MRI

Summary of approach to imaging

Ultrasound is good if you have a high pretest probability good radiologist thin patient

MRI is good if you are in a center that uses it routinely to evaluate children with appendicitis

CT scan is best if above criteria are not met

Alternative to imaging

ndash OBSERVATION

Once diagnosis of appendicitis has been establishedhellip

Non-ruptured

ndash Short duration of symptoms no suggestion of rupture on imaging

ndash Start antibiotics

ndash Perform appendectomy

Ruptured

ndash Have radiology evaluate for drainable fluid collection

ndash Antibiotics

ndash Operation if above management fails

Once diagnosis of appendicitis has been establishedhellip

Non-ruptured

ndash Short duration of symptoms no suggestion of rupture on imaging

ndash Start antibiotics

ndash Perform appendectomy

Ruptured

ndash Have radiology evaluate for drainable fluid collection

ndash Antibiotics

ndash Operation if above management fails

When should appendectomy be performed in a

patient with non-ruptured acute appendicitis

A Immediately after diagnosis

B Within first 24 hours of hospital stay once appropriate antibiotics and IVF resuscitation have been provided

C After completion of a one-week course of antibiotics

D It is reasonable to treat these patients with antibiotics alone and not perform an appendectomy

Is the appendix a ticking time bomb

Traditional teaching appendicitis is an emergency and should be treated with surgery immediately upon diagnosis

Several studies have now shown that delays of 12 hours up to 48 hours do not increase risk of perforation

Current standard admit IVF resuscitation antibiotics appendectomy during daylight hours

Almstroumlm M1 Svensson JF Patkova B Svenningsson A Wester TAnn Surg 2016 Mar 8 In-hospital

Surgical Delay Does Not Increase the Risk for Perforated Appendicitis in Children A Single-center

Retrospective Cohort Study

Port placement

Lap appy with Endo-loops

Lap appy with stapler

Open appendectomy

Appendix with fecalith

Laparoscopic vs open appendectomy

Laparoscopy is standard of care

Lower complication rate

Less scar

Less pain

Ability to evaluate other intra-abdominal organs if the appendix looks normal

ndash Gallbladder

ndash Ovaries

ndash Inguinal canal

Sauerland S Jaschinski T Neugebauer EA Laparoscopic versus open surgery for

suspected appendicitis Cochrane Database Syst Rev 2010 Oct 6(10)

Post-op course

Most patients can go home on the day of surgery

Recovery is usually quick

Back to school within a week sports within 2 weeks

Very low risk surgery with good outcomes

Risk of infection is about 5 at port sites or in abdomen

You see a 7 year old male in your office with 1-day history of abdominal pain

now localized to RLQ with focal guarding and poor appetite You send him for

ultrasound and it shows appendicitis His mother has heard about some new

research and wants to know if you would recommend just treating with

antibiotics rather than surgery How would you counsel this mom

A This data is preliminary and surgery is still standard of

care

B This may be a reasonable option for her son and she

should discuss it with her surgeon

C If it were your son you wouldnrsquot let him have surgery

D Other

Do all patients with appendicitis require surgery

Pilot study at Nationwide Childrenrsquos Hospital

102 patients

ndash 7 to 17 years of age

ndash Uncomplicated appendicitis defined by

Abdominal pain le 48 hours

White blood cell count le 18000

Ultrasound or CT scan showing appendicitis with an appendix le 11 centimeter thick and no

evidence of abscess or fecalith

Patients and families chose to have appendectomy or antibiotics alone

Non-operative management at least 24 hours of in-hospital observation and IV antibiotics until symptoms improved followed by completion of 10 days of treatment with antibiotics by mouth

Minneci PC Mahida JB et al The effectiveness of patient choice in non-operative

versus surgical management of uncomplicated acute appendicitis JAMA Surgery

2015 Dec 16

Results of non-operative management

65 families chose appendectomy 37 families chose non-operative management

Success rate of non-operative management (defined as not undergoing an appendectomy) 89 at 30 days 76 at 1 year

The 24 who failed did NOT have a higher rate of ruptured appendicitis compared to the patients who had immediate appendectomy

1 year follow-up the children managed non-operatively compared with the surgery group had fewer disability days (8 vs 21 days) lower appendicitis-related health care costs (median $4219 vs $5029) and no difference in health-related quality of life

Non-Operative Treatment of Appendicitis Rationale

Appendectomy is invasive

Children may miss up to two weeks of schoolactivities

Caregivers miss work

Postop complications after appendectomy for uncomplicated appendicitis 5-10

Serious complications (reoperations or readmissions) 1-7

Adult data suggest one-year success rates of 63-85 no difference in rates of complicated appendicitis

bull Meta-analysis of 10 articles reporting 413 children receiving non

operative treatment (NOT) for appendicitis ndash all published in past 10

years

bull 5 comparative

bull 1 RCT

bull 4 case series of NOT

Interpreting meta-analyses Forest plot

Favors

antibiotics

Favors

appendectomy

Overall success of NOT

97 during initial episode

Incidence of recurrent appendicitis during follow-up period (range 2-51

months)

Long term efficacy of NOT (no appendectomy at end of follow-up

period)

LOS shortened by about 05 days in those undergoing appendectomy

compared to NOT

Increased rate of complications for appendectomy compared to NOT

Authorsrsquo Conclusions

ldquoCurrent data suggest that NOT is safe It appears effective as initial treatment in 97 of children with AUA and the rate of recurrent appendicitis is 14rdquo

The study highlights the lack of robust evidence comparing NOT with appendectomy in children

Confirms a position of equipoise between treatment approaches

We recommend that NOT of children with AUA be reserved for those participating in carefully designed research studies

Take-home point

Lap appy is still standard of care for uncomplicated appendicitis

Once diagnosis of appendicitis has been establishedhellip

Non-ruptured

ndash Short duration of symptoms no suggestion of rupture on imaging

ndash Start antibiotics

ndash Perform appendectomy

Ruptured

ndash Have radiology evaluate for drainable fluid collection

ndash Antibiotics

ndash Operation in acute setting if above management fails

ndash Interval appendectomy 6-8 weeks laterhellip

ndash ALTERNATIVELY Just take out the appendix

CT ruptured appendix with abscess

After placement of percutaneous drain

How to counsel families of children with ruptured appendicitis

If kids tolerate a diet pain resolves fever resolves oral antibiotics and home

Plan interval appy 6-8 weeks later

Non-operative management may not work and surgery may be needed (Failure rate 20)

At home they should watch for signs of persistentrecurrent appendicitis

High-anxiety time for patients and families

A 10-year-old boy comes to see you in the office after recent

hospitalization for perforated appendicitis He has 3 more days of

antibiotics left His mom is worried because appetite is poor and his

energy level is low On exam he has diffuse lower abdominal

tenderness You recommend

A Extending course of oral antibiotics

B CT scan to evaluate for persistent or

recurrent appendicitis

C CT scan to evaluate for intra-abdominal

abscess

D Follow-up with surgeon

Is interval appendectomy necessary

In adults many surgeons do not do this operation

In kids data are limited

ndash 2-year follow-up of 96 patients

ndash Perforated appendicitis treated non-operatively with antibiotics

ndash 6 became worse 41 had interval appendectomy

ndash 49 received no further treatment

ndash 57 no recurrence

ndash 43 had recurrence within one month to 2 years

ndash Presence of appendicolith 72 rate of recurrence vs 26 in those without appendicolith

Ein SH1 Langer JC Daneman A Nonoperative management of pediatric ruptured appendix

with inflammatory mass or abscess presence of an appendicolith predicts recurrent

appendicitis J Pediatr Surg 2005 Oct40(10)1612-5

Approach to interval appendectomy

More strongly recommended if fecalith present

Observation is a reasonable option

Best choice for an individual patient depends on their anxiety and parental anxiety

Immediate operation for ruptured appendicitis perhaps a better

option

Meta-analysis operative vs non-operative management of pediatric

ruptured appendicitis

2 RCTs identified

Total of 171 pediatric patients

Compared early vs interval appendectomy

Early appendectomy reduced incidence of adverse event

Ruptured appendicitis may have abscess or phlegmon

Early surgery was more strongly favored when there was no abscess at

time of presentation

Decreased antibiotic duration length of stay and total charges for

abscess and no abscess groups

You are seeing a 7 year old with constipation A

fecalith was seen on abdominal X ray during recent

ED visit How would you counsel the family

A They should be referred to surgery for appendectomy

B Their child is at higher risk for appendicitis so they should be aware of this in case he develops symptoms

C It is uncertain whether this child is at higher risk for appendicitis

D A course of Miralax may help wash out the fecalith

Take home points

Uncomplicated appendicitis

ndash Lap appy is still standard of care

ndash Non-operative management may be an acceptable option but not enough is known about long-term risk of recurrent appendicitis

Complicated appendicitis

ndash Can be managed with immediate operation delayed appendectomy or no appendectomy

ndash Immediate operation is probably more efficient and less stressful for patients and parents

Thank you very much

Questions

Page 11: New approaches for evaluation and treatment of appendicitis · Ultrasound or CT scan showing appendicitis with an appendix ≤ 1.1 centimeter thick and no evidence of abscess or fecalith

How do children present with appendicitis

Pain starts peri-umbilical and then migrates to RLQ

Pain starts insidiously is persistent and worsens over time

Pain is worse with movement

Fever nausea with or without vomiting and anorexia

Features suggesting alternative diagnoses

ndash Waxing and waning pain

ndash Diarrhea cough sore throat myalgias rhinorrhea and sick contacts

ndash (BUT there are patients with many of the above symptoms who DO have appendicitis)

Differential diagnosis

Virus

Pneumonia

Constipation

In adolescent females

ndash PID

ndash Ovarian cyst

ndash Mittelschmertz

Approach to physical exam in suspected appendicitis

Child does NOT want to move around

Focal tenderness

Percussion of the abdomen causes discomfort (this is better than rebound tenderness)

Psoas obturator Rovsingrsquos heel strike

Watch them walk around

ldquoDoes this child have appendicitisrdquo Meta-Analysis

Fever

ndash If present LR 34 (24-48)

ndash If absent LR 032 (016-064)

Rebound tenderness

ndash If present LR 30 (23-39)

ndash If absent LR 028 (014-055)

Migration of pain LR range 19-31

RLQ pain LR 12 (10-15)

WBC less than 10 LR 022 (017-030)

ANC less than 6750 LR 006 (003-016) Bundy DG Byerley JS Liles EA Perrin EM Katznelson J Rice HE

Does this child have appendicitis JAMA 2007 Jul 25298(4)438-51

Alvarado A (1986) A practical score for the early diagnosis

of acute appendicitis Ann Emerg Med 15557ndash564

You are seeing a child in your office with 1-day history of

RLQ pain You are suspicious for appendicitis Where would

you send the patient

A Emergency room

B Same day outpatient surgery clinic

C CT scan

D Ultrasound

E Other

What to do with suspected appendicitis

Obtain imaging vs send to ED

ndash If any suspicion for viral syndrome with dehydration to ED

ndash If VERY HIGH suspicion for appendicitis and good pediatric radiologist is available consider sending child for ultrasound

Operating Room

further wu per surgeon

Further workup and imaging in

consultation with

Surgery team

1 IV placed 20 cckg bolus CBC

2 Surgical consult within one hour

Strong suspicion on

HPE Equivocal history

physical exam

1 IV placed 20 cckg bolus CBC

2 Re-examine surgical consult within one

hour if still tender

Abdominal pain

ro appendicitis

What is the best way to image the appendix

A CT scan

B Ultrasound

C MRI

D Abdominal X ray

E Other

Ultrasound

CT scan

MRI

Summary of approach to imaging

Ultrasound is good if you have a high pretest probability good radiologist thin patient

MRI is good if you are in a center that uses it routinely to evaluate children with appendicitis

CT scan is best if above criteria are not met

Alternative to imaging

ndash OBSERVATION

Once diagnosis of appendicitis has been establishedhellip

Non-ruptured

ndash Short duration of symptoms no suggestion of rupture on imaging

ndash Start antibiotics

ndash Perform appendectomy

Ruptured

ndash Have radiology evaluate for drainable fluid collection

ndash Antibiotics

ndash Operation if above management fails

Once diagnosis of appendicitis has been establishedhellip

Non-ruptured

ndash Short duration of symptoms no suggestion of rupture on imaging

ndash Start antibiotics

ndash Perform appendectomy

Ruptured

ndash Have radiology evaluate for drainable fluid collection

ndash Antibiotics

ndash Operation if above management fails

When should appendectomy be performed in a

patient with non-ruptured acute appendicitis

A Immediately after diagnosis

B Within first 24 hours of hospital stay once appropriate antibiotics and IVF resuscitation have been provided

C After completion of a one-week course of antibiotics

D It is reasonable to treat these patients with antibiotics alone and not perform an appendectomy

Is the appendix a ticking time bomb

Traditional teaching appendicitis is an emergency and should be treated with surgery immediately upon diagnosis

Several studies have now shown that delays of 12 hours up to 48 hours do not increase risk of perforation

Current standard admit IVF resuscitation antibiotics appendectomy during daylight hours

Almstroumlm M1 Svensson JF Patkova B Svenningsson A Wester TAnn Surg 2016 Mar 8 In-hospital

Surgical Delay Does Not Increase the Risk for Perforated Appendicitis in Children A Single-center

Retrospective Cohort Study

Port placement

Lap appy with Endo-loops

Lap appy with stapler

Open appendectomy

Appendix with fecalith

Laparoscopic vs open appendectomy

Laparoscopy is standard of care

Lower complication rate

Less scar

Less pain

Ability to evaluate other intra-abdominal organs if the appendix looks normal

ndash Gallbladder

ndash Ovaries

ndash Inguinal canal

Sauerland S Jaschinski T Neugebauer EA Laparoscopic versus open surgery for

suspected appendicitis Cochrane Database Syst Rev 2010 Oct 6(10)

Post-op course

Most patients can go home on the day of surgery

Recovery is usually quick

Back to school within a week sports within 2 weeks

Very low risk surgery with good outcomes

Risk of infection is about 5 at port sites or in abdomen

You see a 7 year old male in your office with 1-day history of abdominal pain

now localized to RLQ with focal guarding and poor appetite You send him for

ultrasound and it shows appendicitis His mother has heard about some new

research and wants to know if you would recommend just treating with

antibiotics rather than surgery How would you counsel this mom

A This data is preliminary and surgery is still standard of

care

B This may be a reasonable option for her son and she

should discuss it with her surgeon

C If it were your son you wouldnrsquot let him have surgery

D Other

Do all patients with appendicitis require surgery

Pilot study at Nationwide Childrenrsquos Hospital

102 patients

ndash 7 to 17 years of age

ndash Uncomplicated appendicitis defined by

Abdominal pain le 48 hours

White blood cell count le 18000

Ultrasound or CT scan showing appendicitis with an appendix le 11 centimeter thick and no

evidence of abscess or fecalith

Patients and families chose to have appendectomy or antibiotics alone

Non-operative management at least 24 hours of in-hospital observation and IV antibiotics until symptoms improved followed by completion of 10 days of treatment with antibiotics by mouth

Minneci PC Mahida JB et al The effectiveness of patient choice in non-operative

versus surgical management of uncomplicated acute appendicitis JAMA Surgery

2015 Dec 16

Results of non-operative management

65 families chose appendectomy 37 families chose non-operative management

Success rate of non-operative management (defined as not undergoing an appendectomy) 89 at 30 days 76 at 1 year

The 24 who failed did NOT have a higher rate of ruptured appendicitis compared to the patients who had immediate appendectomy

1 year follow-up the children managed non-operatively compared with the surgery group had fewer disability days (8 vs 21 days) lower appendicitis-related health care costs (median $4219 vs $5029) and no difference in health-related quality of life

Non-Operative Treatment of Appendicitis Rationale

Appendectomy is invasive

Children may miss up to two weeks of schoolactivities

Caregivers miss work

Postop complications after appendectomy for uncomplicated appendicitis 5-10

Serious complications (reoperations or readmissions) 1-7

Adult data suggest one-year success rates of 63-85 no difference in rates of complicated appendicitis

bull Meta-analysis of 10 articles reporting 413 children receiving non

operative treatment (NOT) for appendicitis ndash all published in past 10

years

bull 5 comparative

bull 1 RCT

bull 4 case series of NOT

Interpreting meta-analyses Forest plot

Favors

antibiotics

Favors

appendectomy

Overall success of NOT

97 during initial episode

Incidence of recurrent appendicitis during follow-up period (range 2-51

months)

Long term efficacy of NOT (no appendectomy at end of follow-up

period)

LOS shortened by about 05 days in those undergoing appendectomy

compared to NOT

Increased rate of complications for appendectomy compared to NOT

Authorsrsquo Conclusions

ldquoCurrent data suggest that NOT is safe It appears effective as initial treatment in 97 of children with AUA and the rate of recurrent appendicitis is 14rdquo

The study highlights the lack of robust evidence comparing NOT with appendectomy in children

Confirms a position of equipoise between treatment approaches

We recommend that NOT of children with AUA be reserved for those participating in carefully designed research studies

Take-home point

Lap appy is still standard of care for uncomplicated appendicitis

Once diagnosis of appendicitis has been establishedhellip

Non-ruptured

ndash Short duration of symptoms no suggestion of rupture on imaging

ndash Start antibiotics

ndash Perform appendectomy

Ruptured

ndash Have radiology evaluate for drainable fluid collection

ndash Antibiotics

ndash Operation in acute setting if above management fails

ndash Interval appendectomy 6-8 weeks laterhellip

ndash ALTERNATIVELY Just take out the appendix

CT ruptured appendix with abscess

After placement of percutaneous drain

How to counsel families of children with ruptured appendicitis

If kids tolerate a diet pain resolves fever resolves oral antibiotics and home

Plan interval appy 6-8 weeks later

Non-operative management may not work and surgery may be needed (Failure rate 20)

At home they should watch for signs of persistentrecurrent appendicitis

High-anxiety time for patients and families

A 10-year-old boy comes to see you in the office after recent

hospitalization for perforated appendicitis He has 3 more days of

antibiotics left His mom is worried because appetite is poor and his

energy level is low On exam he has diffuse lower abdominal

tenderness You recommend

A Extending course of oral antibiotics

B CT scan to evaluate for persistent or

recurrent appendicitis

C CT scan to evaluate for intra-abdominal

abscess

D Follow-up with surgeon

Is interval appendectomy necessary

In adults many surgeons do not do this operation

In kids data are limited

ndash 2-year follow-up of 96 patients

ndash Perforated appendicitis treated non-operatively with antibiotics

ndash 6 became worse 41 had interval appendectomy

ndash 49 received no further treatment

ndash 57 no recurrence

ndash 43 had recurrence within one month to 2 years

ndash Presence of appendicolith 72 rate of recurrence vs 26 in those without appendicolith

Ein SH1 Langer JC Daneman A Nonoperative management of pediatric ruptured appendix

with inflammatory mass or abscess presence of an appendicolith predicts recurrent

appendicitis J Pediatr Surg 2005 Oct40(10)1612-5

Approach to interval appendectomy

More strongly recommended if fecalith present

Observation is a reasonable option

Best choice for an individual patient depends on their anxiety and parental anxiety

Immediate operation for ruptured appendicitis perhaps a better

option

Meta-analysis operative vs non-operative management of pediatric

ruptured appendicitis

2 RCTs identified

Total of 171 pediatric patients

Compared early vs interval appendectomy

Early appendectomy reduced incidence of adverse event

Ruptured appendicitis may have abscess or phlegmon

Early surgery was more strongly favored when there was no abscess at

time of presentation

Decreased antibiotic duration length of stay and total charges for

abscess and no abscess groups

You are seeing a 7 year old with constipation A

fecalith was seen on abdominal X ray during recent

ED visit How would you counsel the family

A They should be referred to surgery for appendectomy

B Their child is at higher risk for appendicitis so they should be aware of this in case he develops symptoms

C It is uncertain whether this child is at higher risk for appendicitis

D A course of Miralax may help wash out the fecalith

Take home points

Uncomplicated appendicitis

ndash Lap appy is still standard of care

ndash Non-operative management may be an acceptable option but not enough is known about long-term risk of recurrent appendicitis

Complicated appendicitis

ndash Can be managed with immediate operation delayed appendectomy or no appendectomy

ndash Immediate operation is probably more efficient and less stressful for patients and parents

Thank you very much

Questions

Page 12: New approaches for evaluation and treatment of appendicitis · Ultrasound or CT scan showing appendicitis with an appendix ≤ 1.1 centimeter thick and no evidence of abscess or fecalith

Differential diagnosis

Virus

Pneumonia

Constipation

In adolescent females

ndash PID

ndash Ovarian cyst

ndash Mittelschmertz

Approach to physical exam in suspected appendicitis

Child does NOT want to move around

Focal tenderness

Percussion of the abdomen causes discomfort (this is better than rebound tenderness)

Psoas obturator Rovsingrsquos heel strike

Watch them walk around

ldquoDoes this child have appendicitisrdquo Meta-Analysis

Fever

ndash If present LR 34 (24-48)

ndash If absent LR 032 (016-064)

Rebound tenderness

ndash If present LR 30 (23-39)

ndash If absent LR 028 (014-055)

Migration of pain LR range 19-31

RLQ pain LR 12 (10-15)

WBC less than 10 LR 022 (017-030)

ANC less than 6750 LR 006 (003-016) Bundy DG Byerley JS Liles EA Perrin EM Katznelson J Rice HE

Does this child have appendicitis JAMA 2007 Jul 25298(4)438-51

Alvarado A (1986) A practical score for the early diagnosis

of acute appendicitis Ann Emerg Med 15557ndash564

You are seeing a child in your office with 1-day history of

RLQ pain You are suspicious for appendicitis Where would

you send the patient

A Emergency room

B Same day outpatient surgery clinic

C CT scan

D Ultrasound

E Other

What to do with suspected appendicitis

Obtain imaging vs send to ED

ndash If any suspicion for viral syndrome with dehydration to ED

ndash If VERY HIGH suspicion for appendicitis and good pediatric radiologist is available consider sending child for ultrasound

Operating Room

further wu per surgeon

Further workup and imaging in

consultation with

Surgery team

1 IV placed 20 cckg bolus CBC

2 Surgical consult within one hour

Strong suspicion on

HPE Equivocal history

physical exam

1 IV placed 20 cckg bolus CBC

2 Re-examine surgical consult within one

hour if still tender

Abdominal pain

ro appendicitis

What is the best way to image the appendix

A CT scan

B Ultrasound

C MRI

D Abdominal X ray

E Other

Ultrasound

CT scan

MRI

Summary of approach to imaging

Ultrasound is good if you have a high pretest probability good radiologist thin patient

MRI is good if you are in a center that uses it routinely to evaluate children with appendicitis

CT scan is best if above criteria are not met

Alternative to imaging

ndash OBSERVATION

Once diagnosis of appendicitis has been establishedhellip

Non-ruptured

ndash Short duration of symptoms no suggestion of rupture on imaging

ndash Start antibiotics

ndash Perform appendectomy

Ruptured

ndash Have radiology evaluate for drainable fluid collection

ndash Antibiotics

ndash Operation if above management fails

Once diagnosis of appendicitis has been establishedhellip

Non-ruptured

ndash Short duration of symptoms no suggestion of rupture on imaging

ndash Start antibiotics

ndash Perform appendectomy

Ruptured

ndash Have radiology evaluate for drainable fluid collection

ndash Antibiotics

ndash Operation if above management fails

When should appendectomy be performed in a

patient with non-ruptured acute appendicitis

A Immediately after diagnosis

B Within first 24 hours of hospital stay once appropriate antibiotics and IVF resuscitation have been provided

C After completion of a one-week course of antibiotics

D It is reasonable to treat these patients with antibiotics alone and not perform an appendectomy

Is the appendix a ticking time bomb

Traditional teaching appendicitis is an emergency and should be treated with surgery immediately upon diagnosis

Several studies have now shown that delays of 12 hours up to 48 hours do not increase risk of perforation

Current standard admit IVF resuscitation antibiotics appendectomy during daylight hours

Almstroumlm M1 Svensson JF Patkova B Svenningsson A Wester TAnn Surg 2016 Mar 8 In-hospital

Surgical Delay Does Not Increase the Risk for Perforated Appendicitis in Children A Single-center

Retrospective Cohort Study

Port placement

Lap appy with Endo-loops

Lap appy with stapler

Open appendectomy

Appendix with fecalith

Laparoscopic vs open appendectomy

Laparoscopy is standard of care

Lower complication rate

Less scar

Less pain

Ability to evaluate other intra-abdominal organs if the appendix looks normal

ndash Gallbladder

ndash Ovaries

ndash Inguinal canal

Sauerland S Jaschinski T Neugebauer EA Laparoscopic versus open surgery for

suspected appendicitis Cochrane Database Syst Rev 2010 Oct 6(10)

Post-op course

Most patients can go home on the day of surgery

Recovery is usually quick

Back to school within a week sports within 2 weeks

Very low risk surgery with good outcomes

Risk of infection is about 5 at port sites or in abdomen

You see a 7 year old male in your office with 1-day history of abdominal pain

now localized to RLQ with focal guarding and poor appetite You send him for

ultrasound and it shows appendicitis His mother has heard about some new

research and wants to know if you would recommend just treating with

antibiotics rather than surgery How would you counsel this mom

A This data is preliminary and surgery is still standard of

care

B This may be a reasonable option for her son and she

should discuss it with her surgeon

C If it were your son you wouldnrsquot let him have surgery

D Other

Do all patients with appendicitis require surgery

Pilot study at Nationwide Childrenrsquos Hospital

102 patients

ndash 7 to 17 years of age

ndash Uncomplicated appendicitis defined by

Abdominal pain le 48 hours

White blood cell count le 18000

Ultrasound or CT scan showing appendicitis with an appendix le 11 centimeter thick and no

evidence of abscess or fecalith

Patients and families chose to have appendectomy or antibiotics alone

Non-operative management at least 24 hours of in-hospital observation and IV antibiotics until symptoms improved followed by completion of 10 days of treatment with antibiotics by mouth

Minneci PC Mahida JB et al The effectiveness of patient choice in non-operative

versus surgical management of uncomplicated acute appendicitis JAMA Surgery

2015 Dec 16

Results of non-operative management

65 families chose appendectomy 37 families chose non-operative management

Success rate of non-operative management (defined as not undergoing an appendectomy) 89 at 30 days 76 at 1 year

The 24 who failed did NOT have a higher rate of ruptured appendicitis compared to the patients who had immediate appendectomy

1 year follow-up the children managed non-operatively compared with the surgery group had fewer disability days (8 vs 21 days) lower appendicitis-related health care costs (median $4219 vs $5029) and no difference in health-related quality of life

Non-Operative Treatment of Appendicitis Rationale

Appendectomy is invasive

Children may miss up to two weeks of schoolactivities

Caregivers miss work

Postop complications after appendectomy for uncomplicated appendicitis 5-10

Serious complications (reoperations or readmissions) 1-7

Adult data suggest one-year success rates of 63-85 no difference in rates of complicated appendicitis

bull Meta-analysis of 10 articles reporting 413 children receiving non

operative treatment (NOT) for appendicitis ndash all published in past 10

years

bull 5 comparative

bull 1 RCT

bull 4 case series of NOT

Interpreting meta-analyses Forest plot

Favors

antibiotics

Favors

appendectomy

Overall success of NOT

97 during initial episode

Incidence of recurrent appendicitis during follow-up period (range 2-51

months)

Long term efficacy of NOT (no appendectomy at end of follow-up

period)

LOS shortened by about 05 days in those undergoing appendectomy

compared to NOT

Increased rate of complications for appendectomy compared to NOT

Authorsrsquo Conclusions

ldquoCurrent data suggest that NOT is safe It appears effective as initial treatment in 97 of children with AUA and the rate of recurrent appendicitis is 14rdquo

The study highlights the lack of robust evidence comparing NOT with appendectomy in children

Confirms a position of equipoise between treatment approaches

We recommend that NOT of children with AUA be reserved for those participating in carefully designed research studies

Take-home point

Lap appy is still standard of care for uncomplicated appendicitis

Once diagnosis of appendicitis has been establishedhellip

Non-ruptured

ndash Short duration of symptoms no suggestion of rupture on imaging

ndash Start antibiotics

ndash Perform appendectomy

Ruptured

ndash Have radiology evaluate for drainable fluid collection

ndash Antibiotics

ndash Operation in acute setting if above management fails

ndash Interval appendectomy 6-8 weeks laterhellip

ndash ALTERNATIVELY Just take out the appendix

CT ruptured appendix with abscess

After placement of percutaneous drain

How to counsel families of children with ruptured appendicitis

If kids tolerate a diet pain resolves fever resolves oral antibiotics and home

Plan interval appy 6-8 weeks later

Non-operative management may not work and surgery may be needed (Failure rate 20)

At home they should watch for signs of persistentrecurrent appendicitis

High-anxiety time for patients and families

A 10-year-old boy comes to see you in the office after recent

hospitalization for perforated appendicitis He has 3 more days of

antibiotics left His mom is worried because appetite is poor and his

energy level is low On exam he has diffuse lower abdominal

tenderness You recommend

A Extending course of oral antibiotics

B CT scan to evaluate for persistent or

recurrent appendicitis

C CT scan to evaluate for intra-abdominal

abscess

D Follow-up with surgeon

Is interval appendectomy necessary

In adults many surgeons do not do this operation

In kids data are limited

ndash 2-year follow-up of 96 patients

ndash Perforated appendicitis treated non-operatively with antibiotics

ndash 6 became worse 41 had interval appendectomy

ndash 49 received no further treatment

ndash 57 no recurrence

ndash 43 had recurrence within one month to 2 years

ndash Presence of appendicolith 72 rate of recurrence vs 26 in those without appendicolith

Ein SH1 Langer JC Daneman A Nonoperative management of pediatric ruptured appendix

with inflammatory mass or abscess presence of an appendicolith predicts recurrent

appendicitis J Pediatr Surg 2005 Oct40(10)1612-5

Approach to interval appendectomy

More strongly recommended if fecalith present

Observation is a reasonable option

Best choice for an individual patient depends on their anxiety and parental anxiety

Immediate operation for ruptured appendicitis perhaps a better

option

Meta-analysis operative vs non-operative management of pediatric

ruptured appendicitis

2 RCTs identified

Total of 171 pediatric patients

Compared early vs interval appendectomy

Early appendectomy reduced incidence of adverse event

Ruptured appendicitis may have abscess or phlegmon

Early surgery was more strongly favored when there was no abscess at

time of presentation

Decreased antibiotic duration length of stay and total charges for

abscess and no abscess groups

You are seeing a 7 year old with constipation A

fecalith was seen on abdominal X ray during recent

ED visit How would you counsel the family

A They should be referred to surgery for appendectomy

B Their child is at higher risk for appendicitis so they should be aware of this in case he develops symptoms

C It is uncertain whether this child is at higher risk for appendicitis

D A course of Miralax may help wash out the fecalith

Take home points

Uncomplicated appendicitis

ndash Lap appy is still standard of care

ndash Non-operative management may be an acceptable option but not enough is known about long-term risk of recurrent appendicitis

Complicated appendicitis

ndash Can be managed with immediate operation delayed appendectomy or no appendectomy

ndash Immediate operation is probably more efficient and less stressful for patients and parents

Thank you very much

Questions

Page 13: New approaches for evaluation and treatment of appendicitis · Ultrasound or CT scan showing appendicitis with an appendix ≤ 1.1 centimeter thick and no evidence of abscess or fecalith

Approach to physical exam in suspected appendicitis

Child does NOT want to move around

Focal tenderness

Percussion of the abdomen causes discomfort (this is better than rebound tenderness)

Psoas obturator Rovsingrsquos heel strike

Watch them walk around

ldquoDoes this child have appendicitisrdquo Meta-Analysis

Fever

ndash If present LR 34 (24-48)

ndash If absent LR 032 (016-064)

Rebound tenderness

ndash If present LR 30 (23-39)

ndash If absent LR 028 (014-055)

Migration of pain LR range 19-31

RLQ pain LR 12 (10-15)

WBC less than 10 LR 022 (017-030)

ANC less than 6750 LR 006 (003-016) Bundy DG Byerley JS Liles EA Perrin EM Katznelson J Rice HE

Does this child have appendicitis JAMA 2007 Jul 25298(4)438-51

Alvarado A (1986) A practical score for the early diagnosis

of acute appendicitis Ann Emerg Med 15557ndash564

You are seeing a child in your office with 1-day history of

RLQ pain You are suspicious for appendicitis Where would

you send the patient

A Emergency room

B Same day outpatient surgery clinic

C CT scan

D Ultrasound

E Other

What to do with suspected appendicitis

Obtain imaging vs send to ED

ndash If any suspicion for viral syndrome with dehydration to ED

ndash If VERY HIGH suspicion for appendicitis and good pediatric radiologist is available consider sending child for ultrasound

Operating Room

further wu per surgeon

Further workup and imaging in

consultation with

Surgery team

1 IV placed 20 cckg bolus CBC

2 Surgical consult within one hour

Strong suspicion on

HPE Equivocal history

physical exam

1 IV placed 20 cckg bolus CBC

2 Re-examine surgical consult within one

hour if still tender

Abdominal pain

ro appendicitis

What is the best way to image the appendix

A CT scan

B Ultrasound

C MRI

D Abdominal X ray

E Other

Ultrasound

CT scan

MRI

Summary of approach to imaging

Ultrasound is good if you have a high pretest probability good radiologist thin patient

MRI is good if you are in a center that uses it routinely to evaluate children with appendicitis

CT scan is best if above criteria are not met

Alternative to imaging

ndash OBSERVATION

Once diagnosis of appendicitis has been establishedhellip

Non-ruptured

ndash Short duration of symptoms no suggestion of rupture on imaging

ndash Start antibiotics

ndash Perform appendectomy

Ruptured

ndash Have radiology evaluate for drainable fluid collection

ndash Antibiotics

ndash Operation if above management fails

Once diagnosis of appendicitis has been establishedhellip

Non-ruptured

ndash Short duration of symptoms no suggestion of rupture on imaging

ndash Start antibiotics

ndash Perform appendectomy

Ruptured

ndash Have radiology evaluate for drainable fluid collection

ndash Antibiotics

ndash Operation if above management fails

When should appendectomy be performed in a

patient with non-ruptured acute appendicitis

A Immediately after diagnosis

B Within first 24 hours of hospital stay once appropriate antibiotics and IVF resuscitation have been provided

C After completion of a one-week course of antibiotics

D It is reasonable to treat these patients with antibiotics alone and not perform an appendectomy

Is the appendix a ticking time bomb

Traditional teaching appendicitis is an emergency and should be treated with surgery immediately upon diagnosis

Several studies have now shown that delays of 12 hours up to 48 hours do not increase risk of perforation

Current standard admit IVF resuscitation antibiotics appendectomy during daylight hours

Almstroumlm M1 Svensson JF Patkova B Svenningsson A Wester TAnn Surg 2016 Mar 8 In-hospital

Surgical Delay Does Not Increase the Risk for Perforated Appendicitis in Children A Single-center

Retrospective Cohort Study

Port placement

Lap appy with Endo-loops

Lap appy with stapler

Open appendectomy

Appendix with fecalith

Laparoscopic vs open appendectomy

Laparoscopy is standard of care

Lower complication rate

Less scar

Less pain

Ability to evaluate other intra-abdominal organs if the appendix looks normal

ndash Gallbladder

ndash Ovaries

ndash Inguinal canal

Sauerland S Jaschinski T Neugebauer EA Laparoscopic versus open surgery for

suspected appendicitis Cochrane Database Syst Rev 2010 Oct 6(10)

Post-op course

Most patients can go home on the day of surgery

Recovery is usually quick

Back to school within a week sports within 2 weeks

Very low risk surgery with good outcomes

Risk of infection is about 5 at port sites or in abdomen

You see a 7 year old male in your office with 1-day history of abdominal pain

now localized to RLQ with focal guarding and poor appetite You send him for

ultrasound and it shows appendicitis His mother has heard about some new

research and wants to know if you would recommend just treating with

antibiotics rather than surgery How would you counsel this mom

A This data is preliminary and surgery is still standard of

care

B This may be a reasonable option for her son and she

should discuss it with her surgeon

C If it were your son you wouldnrsquot let him have surgery

D Other

Do all patients with appendicitis require surgery

Pilot study at Nationwide Childrenrsquos Hospital

102 patients

ndash 7 to 17 years of age

ndash Uncomplicated appendicitis defined by

Abdominal pain le 48 hours

White blood cell count le 18000

Ultrasound or CT scan showing appendicitis with an appendix le 11 centimeter thick and no

evidence of abscess or fecalith

Patients and families chose to have appendectomy or antibiotics alone

Non-operative management at least 24 hours of in-hospital observation and IV antibiotics until symptoms improved followed by completion of 10 days of treatment with antibiotics by mouth

Minneci PC Mahida JB et al The effectiveness of patient choice in non-operative

versus surgical management of uncomplicated acute appendicitis JAMA Surgery

2015 Dec 16

Results of non-operative management

65 families chose appendectomy 37 families chose non-operative management

Success rate of non-operative management (defined as not undergoing an appendectomy) 89 at 30 days 76 at 1 year

The 24 who failed did NOT have a higher rate of ruptured appendicitis compared to the patients who had immediate appendectomy

1 year follow-up the children managed non-operatively compared with the surgery group had fewer disability days (8 vs 21 days) lower appendicitis-related health care costs (median $4219 vs $5029) and no difference in health-related quality of life

Non-Operative Treatment of Appendicitis Rationale

Appendectomy is invasive

Children may miss up to two weeks of schoolactivities

Caregivers miss work

Postop complications after appendectomy for uncomplicated appendicitis 5-10

Serious complications (reoperations or readmissions) 1-7

Adult data suggest one-year success rates of 63-85 no difference in rates of complicated appendicitis

bull Meta-analysis of 10 articles reporting 413 children receiving non

operative treatment (NOT) for appendicitis ndash all published in past 10

years

bull 5 comparative

bull 1 RCT

bull 4 case series of NOT

Interpreting meta-analyses Forest plot

Favors

antibiotics

Favors

appendectomy

Overall success of NOT

97 during initial episode

Incidence of recurrent appendicitis during follow-up period (range 2-51

months)

Long term efficacy of NOT (no appendectomy at end of follow-up

period)

LOS shortened by about 05 days in those undergoing appendectomy

compared to NOT

Increased rate of complications for appendectomy compared to NOT

Authorsrsquo Conclusions

ldquoCurrent data suggest that NOT is safe It appears effective as initial treatment in 97 of children with AUA and the rate of recurrent appendicitis is 14rdquo

The study highlights the lack of robust evidence comparing NOT with appendectomy in children

Confirms a position of equipoise between treatment approaches

We recommend that NOT of children with AUA be reserved for those participating in carefully designed research studies

Take-home point

Lap appy is still standard of care for uncomplicated appendicitis

Once diagnosis of appendicitis has been establishedhellip

Non-ruptured

ndash Short duration of symptoms no suggestion of rupture on imaging

ndash Start antibiotics

ndash Perform appendectomy

Ruptured

ndash Have radiology evaluate for drainable fluid collection

ndash Antibiotics

ndash Operation in acute setting if above management fails

ndash Interval appendectomy 6-8 weeks laterhellip

ndash ALTERNATIVELY Just take out the appendix

CT ruptured appendix with abscess

After placement of percutaneous drain

How to counsel families of children with ruptured appendicitis

If kids tolerate a diet pain resolves fever resolves oral antibiotics and home

Plan interval appy 6-8 weeks later

Non-operative management may not work and surgery may be needed (Failure rate 20)

At home they should watch for signs of persistentrecurrent appendicitis

High-anxiety time for patients and families

A 10-year-old boy comes to see you in the office after recent

hospitalization for perforated appendicitis He has 3 more days of

antibiotics left His mom is worried because appetite is poor and his

energy level is low On exam he has diffuse lower abdominal

tenderness You recommend

A Extending course of oral antibiotics

B CT scan to evaluate for persistent or

recurrent appendicitis

C CT scan to evaluate for intra-abdominal

abscess

D Follow-up with surgeon

Is interval appendectomy necessary

In adults many surgeons do not do this operation

In kids data are limited

ndash 2-year follow-up of 96 patients

ndash Perforated appendicitis treated non-operatively with antibiotics

ndash 6 became worse 41 had interval appendectomy

ndash 49 received no further treatment

ndash 57 no recurrence

ndash 43 had recurrence within one month to 2 years

ndash Presence of appendicolith 72 rate of recurrence vs 26 in those without appendicolith

Ein SH1 Langer JC Daneman A Nonoperative management of pediatric ruptured appendix

with inflammatory mass or abscess presence of an appendicolith predicts recurrent

appendicitis J Pediatr Surg 2005 Oct40(10)1612-5

Approach to interval appendectomy

More strongly recommended if fecalith present

Observation is a reasonable option

Best choice for an individual patient depends on their anxiety and parental anxiety

Immediate operation for ruptured appendicitis perhaps a better

option

Meta-analysis operative vs non-operative management of pediatric

ruptured appendicitis

2 RCTs identified

Total of 171 pediatric patients

Compared early vs interval appendectomy

Early appendectomy reduced incidence of adverse event

Ruptured appendicitis may have abscess or phlegmon

Early surgery was more strongly favored when there was no abscess at

time of presentation

Decreased antibiotic duration length of stay and total charges for

abscess and no abscess groups

You are seeing a 7 year old with constipation A

fecalith was seen on abdominal X ray during recent

ED visit How would you counsel the family

A They should be referred to surgery for appendectomy

B Their child is at higher risk for appendicitis so they should be aware of this in case he develops symptoms

C It is uncertain whether this child is at higher risk for appendicitis

D A course of Miralax may help wash out the fecalith

Take home points

Uncomplicated appendicitis

ndash Lap appy is still standard of care

ndash Non-operative management may be an acceptable option but not enough is known about long-term risk of recurrent appendicitis

Complicated appendicitis

ndash Can be managed with immediate operation delayed appendectomy or no appendectomy

ndash Immediate operation is probably more efficient and less stressful for patients and parents

Thank you very much

Questions

Page 14: New approaches for evaluation and treatment of appendicitis · Ultrasound or CT scan showing appendicitis with an appendix ≤ 1.1 centimeter thick and no evidence of abscess or fecalith

ldquoDoes this child have appendicitisrdquo Meta-Analysis

Fever

ndash If present LR 34 (24-48)

ndash If absent LR 032 (016-064)

Rebound tenderness

ndash If present LR 30 (23-39)

ndash If absent LR 028 (014-055)

Migration of pain LR range 19-31

RLQ pain LR 12 (10-15)

WBC less than 10 LR 022 (017-030)

ANC less than 6750 LR 006 (003-016) Bundy DG Byerley JS Liles EA Perrin EM Katznelson J Rice HE

Does this child have appendicitis JAMA 2007 Jul 25298(4)438-51

Alvarado A (1986) A practical score for the early diagnosis

of acute appendicitis Ann Emerg Med 15557ndash564

You are seeing a child in your office with 1-day history of

RLQ pain You are suspicious for appendicitis Where would

you send the patient

A Emergency room

B Same day outpatient surgery clinic

C CT scan

D Ultrasound

E Other

What to do with suspected appendicitis

Obtain imaging vs send to ED

ndash If any suspicion for viral syndrome with dehydration to ED

ndash If VERY HIGH suspicion for appendicitis and good pediatric radiologist is available consider sending child for ultrasound

Operating Room

further wu per surgeon

Further workup and imaging in

consultation with

Surgery team

1 IV placed 20 cckg bolus CBC

2 Surgical consult within one hour

Strong suspicion on

HPE Equivocal history

physical exam

1 IV placed 20 cckg bolus CBC

2 Re-examine surgical consult within one

hour if still tender

Abdominal pain

ro appendicitis

What is the best way to image the appendix

A CT scan

B Ultrasound

C MRI

D Abdominal X ray

E Other

Ultrasound

CT scan

MRI

Summary of approach to imaging

Ultrasound is good if you have a high pretest probability good radiologist thin patient

MRI is good if you are in a center that uses it routinely to evaluate children with appendicitis

CT scan is best if above criteria are not met

Alternative to imaging

ndash OBSERVATION

Once diagnosis of appendicitis has been establishedhellip

Non-ruptured

ndash Short duration of symptoms no suggestion of rupture on imaging

ndash Start antibiotics

ndash Perform appendectomy

Ruptured

ndash Have radiology evaluate for drainable fluid collection

ndash Antibiotics

ndash Operation if above management fails

Once diagnosis of appendicitis has been establishedhellip

Non-ruptured

ndash Short duration of symptoms no suggestion of rupture on imaging

ndash Start antibiotics

ndash Perform appendectomy

Ruptured

ndash Have radiology evaluate for drainable fluid collection

ndash Antibiotics

ndash Operation if above management fails

When should appendectomy be performed in a

patient with non-ruptured acute appendicitis

A Immediately after diagnosis

B Within first 24 hours of hospital stay once appropriate antibiotics and IVF resuscitation have been provided

C After completion of a one-week course of antibiotics

D It is reasonable to treat these patients with antibiotics alone and not perform an appendectomy

Is the appendix a ticking time bomb

Traditional teaching appendicitis is an emergency and should be treated with surgery immediately upon diagnosis

Several studies have now shown that delays of 12 hours up to 48 hours do not increase risk of perforation

Current standard admit IVF resuscitation antibiotics appendectomy during daylight hours

Almstroumlm M1 Svensson JF Patkova B Svenningsson A Wester TAnn Surg 2016 Mar 8 In-hospital

Surgical Delay Does Not Increase the Risk for Perforated Appendicitis in Children A Single-center

Retrospective Cohort Study

Port placement

Lap appy with Endo-loops

Lap appy with stapler

Open appendectomy

Appendix with fecalith

Laparoscopic vs open appendectomy

Laparoscopy is standard of care

Lower complication rate

Less scar

Less pain

Ability to evaluate other intra-abdominal organs if the appendix looks normal

ndash Gallbladder

ndash Ovaries

ndash Inguinal canal

Sauerland S Jaschinski T Neugebauer EA Laparoscopic versus open surgery for

suspected appendicitis Cochrane Database Syst Rev 2010 Oct 6(10)

Post-op course

Most patients can go home on the day of surgery

Recovery is usually quick

Back to school within a week sports within 2 weeks

Very low risk surgery with good outcomes

Risk of infection is about 5 at port sites or in abdomen

You see a 7 year old male in your office with 1-day history of abdominal pain

now localized to RLQ with focal guarding and poor appetite You send him for

ultrasound and it shows appendicitis His mother has heard about some new

research and wants to know if you would recommend just treating with

antibiotics rather than surgery How would you counsel this mom

A This data is preliminary and surgery is still standard of

care

B This may be a reasonable option for her son and she

should discuss it with her surgeon

C If it were your son you wouldnrsquot let him have surgery

D Other

Do all patients with appendicitis require surgery

Pilot study at Nationwide Childrenrsquos Hospital

102 patients

ndash 7 to 17 years of age

ndash Uncomplicated appendicitis defined by

Abdominal pain le 48 hours

White blood cell count le 18000

Ultrasound or CT scan showing appendicitis with an appendix le 11 centimeter thick and no

evidence of abscess or fecalith

Patients and families chose to have appendectomy or antibiotics alone

Non-operative management at least 24 hours of in-hospital observation and IV antibiotics until symptoms improved followed by completion of 10 days of treatment with antibiotics by mouth

Minneci PC Mahida JB et al The effectiveness of patient choice in non-operative

versus surgical management of uncomplicated acute appendicitis JAMA Surgery

2015 Dec 16

Results of non-operative management

65 families chose appendectomy 37 families chose non-operative management

Success rate of non-operative management (defined as not undergoing an appendectomy) 89 at 30 days 76 at 1 year

The 24 who failed did NOT have a higher rate of ruptured appendicitis compared to the patients who had immediate appendectomy

1 year follow-up the children managed non-operatively compared with the surgery group had fewer disability days (8 vs 21 days) lower appendicitis-related health care costs (median $4219 vs $5029) and no difference in health-related quality of life

Non-Operative Treatment of Appendicitis Rationale

Appendectomy is invasive

Children may miss up to two weeks of schoolactivities

Caregivers miss work

Postop complications after appendectomy for uncomplicated appendicitis 5-10

Serious complications (reoperations or readmissions) 1-7

Adult data suggest one-year success rates of 63-85 no difference in rates of complicated appendicitis

bull Meta-analysis of 10 articles reporting 413 children receiving non

operative treatment (NOT) for appendicitis ndash all published in past 10

years

bull 5 comparative

bull 1 RCT

bull 4 case series of NOT

Interpreting meta-analyses Forest plot

Favors

antibiotics

Favors

appendectomy

Overall success of NOT

97 during initial episode

Incidence of recurrent appendicitis during follow-up period (range 2-51

months)

Long term efficacy of NOT (no appendectomy at end of follow-up

period)

LOS shortened by about 05 days in those undergoing appendectomy

compared to NOT

Increased rate of complications for appendectomy compared to NOT

Authorsrsquo Conclusions

ldquoCurrent data suggest that NOT is safe It appears effective as initial treatment in 97 of children with AUA and the rate of recurrent appendicitis is 14rdquo

The study highlights the lack of robust evidence comparing NOT with appendectomy in children

Confirms a position of equipoise between treatment approaches

We recommend that NOT of children with AUA be reserved for those participating in carefully designed research studies

Take-home point

Lap appy is still standard of care for uncomplicated appendicitis

Once diagnosis of appendicitis has been establishedhellip

Non-ruptured

ndash Short duration of symptoms no suggestion of rupture on imaging

ndash Start antibiotics

ndash Perform appendectomy

Ruptured

ndash Have radiology evaluate for drainable fluid collection

ndash Antibiotics

ndash Operation in acute setting if above management fails

ndash Interval appendectomy 6-8 weeks laterhellip

ndash ALTERNATIVELY Just take out the appendix

CT ruptured appendix with abscess

After placement of percutaneous drain

How to counsel families of children with ruptured appendicitis

If kids tolerate a diet pain resolves fever resolves oral antibiotics and home

Plan interval appy 6-8 weeks later

Non-operative management may not work and surgery may be needed (Failure rate 20)

At home they should watch for signs of persistentrecurrent appendicitis

High-anxiety time for patients and families

A 10-year-old boy comes to see you in the office after recent

hospitalization for perforated appendicitis He has 3 more days of

antibiotics left His mom is worried because appetite is poor and his

energy level is low On exam he has diffuse lower abdominal

tenderness You recommend

A Extending course of oral antibiotics

B CT scan to evaluate for persistent or

recurrent appendicitis

C CT scan to evaluate for intra-abdominal

abscess

D Follow-up with surgeon

Is interval appendectomy necessary

In adults many surgeons do not do this operation

In kids data are limited

ndash 2-year follow-up of 96 patients

ndash Perforated appendicitis treated non-operatively with antibiotics

ndash 6 became worse 41 had interval appendectomy

ndash 49 received no further treatment

ndash 57 no recurrence

ndash 43 had recurrence within one month to 2 years

ndash Presence of appendicolith 72 rate of recurrence vs 26 in those without appendicolith

Ein SH1 Langer JC Daneman A Nonoperative management of pediatric ruptured appendix

with inflammatory mass or abscess presence of an appendicolith predicts recurrent

appendicitis J Pediatr Surg 2005 Oct40(10)1612-5

Approach to interval appendectomy

More strongly recommended if fecalith present

Observation is a reasonable option

Best choice for an individual patient depends on their anxiety and parental anxiety

Immediate operation for ruptured appendicitis perhaps a better

option

Meta-analysis operative vs non-operative management of pediatric

ruptured appendicitis

2 RCTs identified

Total of 171 pediatric patients

Compared early vs interval appendectomy

Early appendectomy reduced incidence of adverse event

Ruptured appendicitis may have abscess or phlegmon

Early surgery was more strongly favored when there was no abscess at

time of presentation

Decreased antibiotic duration length of stay and total charges for

abscess and no abscess groups

You are seeing a 7 year old with constipation A

fecalith was seen on abdominal X ray during recent

ED visit How would you counsel the family

A They should be referred to surgery for appendectomy

B Their child is at higher risk for appendicitis so they should be aware of this in case he develops symptoms

C It is uncertain whether this child is at higher risk for appendicitis

D A course of Miralax may help wash out the fecalith

Take home points

Uncomplicated appendicitis

ndash Lap appy is still standard of care

ndash Non-operative management may be an acceptable option but not enough is known about long-term risk of recurrent appendicitis

Complicated appendicitis

ndash Can be managed with immediate operation delayed appendectomy or no appendectomy

ndash Immediate operation is probably more efficient and less stressful for patients and parents

Thank you very much

Questions

Page 15: New approaches for evaluation and treatment of appendicitis · Ultrasound or CT scan showing appendicitis with an appendix ≤ 1.1 centimeter thick and no evidence of abscess or fecalith

Alvarado A (1986) A practical score for the early diagnosis

of acute appendicitis Ann Emerg Med 15557ndash564

You are seeing a child in your office with 1-day history of

RLQ pain You are suspicious for appendicitis Where would

you send the patient

A Emergency room

B Same day outpatient surgery clinic

C CT scan

D Ultrasound

E Other

What to do with suspected appendicitis

Obtain imaging vs send to ED

ndash If any suspicion for viral syndrome with dehydration to ED

ndash If VERY HIGH suspicion for appendicitis and good pediatric radiologist is available consider sending child for ultrasound

Operating Room

further wu per surgeon

Further workup and imaging in

consultation with

Surgery team

1 IV placed 20 cckg bolus CBC

2 Surgical consult within one hour

Strong suspicion on

HPE Equivocal history

physical exam

1 IV placed 20 cckg bolus CBC

2 Re-examine surgical consult within one

hour if still tender

Abdominal pain

ro appendicitis

What is the best way to image the appendix

A CT scan

B Ultrasound

C MRI

D Abdominal X ray

E Other

Ultrasound

CT scan

MRI

Summary of approach to imaging

Ultrasound is good if you have a high pretest probability good radiologist thin patient

MRI is good if you are in a center that uses it routinely to evaluate children with appendicitis

CT scan is best if above criteria are not met

Alternative to imaging

ndash OBSERVATION

Once diagnosis of appendicitis has been establishedhellip

Non-ruptured

ndash Short duration of symptoms no suggestion of rupture on imaging

ndash Start antibiotics

ndash Perform appendectomy

Ruptured

ndash Have radiology evaluate for drainable fluid collection

ndash Antibiotics

ndash Operation if above management fails

Once diagnosis of appendicitis has been establishedhellip

Non-ruptured

ndash Short duration of symptoms no suggestion of rupture on imaging

ndash Start antibiotics

ndash Perform appendectomy

Ruptured

ndash Have radiology evaluate for drainable fluid collection

ndash Antibiotics

ndash Operation if above management fails

When should appendectomy be performed in a

patient with non-ruptured acute appendicitis

A Immediately after diagnosis

B Within first 24 hours of hospital stay once appropriate antibiotics and IVF resuscitation have been provided

C After completion of a one-week course of antibiotics

D It is reasonable to treat these patients with antibiotics alone and not perform an appendectomy

Is the appendix a ticking time bomb

Traditional teaching appendicitis is an emergency and should be treated with surgery immediately upon diagnosis

Several studies have now shown that delays of 12 hours up to 48 hours do not increase risk of perforation

Current standard admit IVF resuscitation antibiotics appendectomy during daylight hours

Almstroumlm M1 Svensson JF Patkova B Svenningsson A Wester TAnn Surg 2016 Mar 8 In-hospital

Surgical Delay Does Not Increase the Risk for Perforated Appendicitis in Children A Single-center

Retrospective Cohort Study

Port placement

Lap appy with Endo-loops

Lap appy with stapler

Open appendectomy

Appendix with fecalith

Laparoscopic vs open appendectomy

Laparoscopy is standard of care

Lower complication rate

Less scar

Less pain

Ability to evaluate other intra-abdominal organs if the appendix looks normal

ndash Gallbladder

ndash Ovaries

ndash Inguinal canal

Sauerland S Jaschinski T Neugebauer EA Laparoscopic versus open surgery for

suspected appendicitis Cochrane Database Syst Rev 2010 Oct 6(10)

Post-op course

Most patients can go home on the day of surgery

Recovery is usually quick

Back to school within a week sports within 2 weeks

Very low risk surgery with good outcomes

Risk of infection is about 5 at port sites or in abdomen

You see a 7 year old male in your office with 1-day history of abdominal pain

now localized to RLQ with focal guarding and poor appetite You send him for

ultrasound and it shows appendicitis His mother has heard about some new

research and wants to know if you would recommend just treating with

antibiotics rather than surgery How would you counsel this mom

A This data is preliminary and surgery is still standard of

care

B This may be a reasonable option for her son and she

should discuss it with her surgeon

C If it were your son you wouldnrsquot let him have surgery

D Other

Do all patients with appendicitis require surgery

Pilot study at Nationwide Childrenrsquos Hospital

102 patients

ndash 7 to 17 years of age

ndash Uncomplicated appendicitis defined by

Abdominal pain le 48 hours

White blood cell count le 18000

Ultrasound or CT scan showing appendicitis with an appendix le 11 centimeter thick and no

evidence of abscess or fecalith

Patients and families chose to have appendectomy or antibiotics alone

Non-operative management at least 24 hours of in-hospital observation and IV antibiotics until symptoms improved followed by completion of 10 days of treatment with antibiotics by mouth

Minneci PC Mahida JB et al The effectiveness of patient choice in non-operative

versus surgical management of uncomplicated acute appendicitis JAMA Surgery

2015 Dec 16

Results of non-operative management

65 families chose appendectomy 37 families chose non-operative management

Success rate of non-operative management (defined as not undergoing an appendectomy) 89 at 30 days 76 at 1 year

The 24 who failed did NOT have a higher rate of ruptured appendicitis compared to the patients who had immediate appendectomy

1 year follow-up the children managed non-operatively compared with the surgery group had fewer disability days (8 vs 21 days) lower appendicitis-related health care costs (median $4219 vs $5029) and no difference in health-related quality of life

Non-Operative Treatment of Appendicitis Rationale

Appendectomy is invasive

Children may miss up to two weeks of schoolactivities

Caregivers miss work

Postop complications after appendectomy for uncomplicated appendicitis 5-10

Serious complications (reoperations or readmissions) 1-7

Adult data suggest one-year success rates of 63-85 no difference in rates of complicated appendicitis

bull Meta-analysis of 10 articles reporting 413 children receiving non

operative treatment (NOT) for appendicitis ndash all published in past 10

years

bull 5 comparative

bull 1 RCT

bull 4 case series of NOT

Interpreting meta-analyses Forest plot

Favors

antibiotics

Favors

appendectomy

Overall success of NOT

97 during initial episode

Incidence of recurrent appendicitis during follow-up period (range 2-51

months)

Long term efficacy of NOT (no appendectomy at end of follow-up

period)

LOS shortened by about 05 days in those undergoing appendectomy

compared to NOT

Increased rate of complications for appendectomy compared to NOT

Authorsrsquo Conclusions

ldquoCurrent data suggest that NOT is safe It appears effective as initial treatment in 97 of children with AUA and the rate of recurrent appendicitis is 14rdquo

The study highlights the lack of robust evidence comparing NOT with appendectomy in children

Confirms a position of equipoise between treatment approaches

We recommend that NOT of children with AUA be reserved for those participating in carefully designed research studies

Take-home point

Lap appy is still standard of care for uncomplicated appendicitis

Once diagnosis of appendicitis has been establishedhellip

Non-ruptured

ndash Short duration of symptoms no suggestion of rupture on imaging

ndash Start antibiotics

ndash Perform appendectomy

Ruptured

ndash Have radiology evaluate for drainable fluid collection

ndash Antibiotics

ndash Operation in acute setting if above management fails

ndash Interval appendectomy 6-8 weeks laterhellip

ndash ALTERNATIVELY Just take out the appendix

CT ruptured appendix with abscess

After placement of percutaneous drain

How to counsel families of children with ruptured appendicitis

If kids tolerate a diet pain resolves fever resolves oral antibiotics and home

Plan interval appy 6-8 weeks later

Non-operative management may not work and surgery may be needed (Failure rate 20)

At home they should watch for signs of persistentrecurrent appendicitis

High-anxiety time for patients and families

A 10-year-old boy comes to see you in the office after recent

hospitalization for perforated appendicitis He has 3 more days of

antibiotics left His mom is worried because appetite is poor and his

energy level is low On exam he has diffuse lower abdominal

tenderness You recommend

A Extending course of oral antibiotics

B CT scan to evaluate for persistent or

recurrent appendicitis

C CT scan to evaluate for intra-abdominal

abscess

D Follow-up with surgeon

Is interval appendectomy necessary

In adults many surgeons do not do this operation

In kids data are limited

ndash 2-year follow-up of 96 patients

ndash Perforated appendicitis treated non-operatively with antibiotics

ndash 6 became worse 41 had interval appendectomy

ndash 49 received no further treatment

ndash 57 no recurrence

ndash 43 had recurrence within one month to 2 years

ndash Presence of appendicolith 72 rate of recurrence vs 26 in those without appendicolith

Ein SH1 Langer JC Daneman A Nonoperative management of pediatric ruptured appendix

with inflammatory mass or abscess presence of an appendicolith predicts recurrent

appendicitis J Pediatr Surg 2005 Oct40(10)1612-5

Approach to interval appendectomy

More strongly recommended if fecalith present

Observation is a reasonable option

Best choice for an individual patient depends on their anxiety and parental anxiety

Immediate operation for ruptured appendicitis perhaps a better

option

Meta-analysis operative vs non-operative management of pediatric

ruptured appendicitis

2 RCTs identified

Total of 171 pediatric patients

Compared early vs interval appendectomy

Early appendectomy reduced incidence of adverse event

Ruptured appendicitis may have abscess or phlegmon

Early surgery was more strongly favored when there was no abscess at

time of presentation

Decreased antibiotic duration length of stay and total charges for

abscess and no abscess groups

You are seeing a 7 year old with constipation A

fecalith was seen on abdominal X ray during recent

ED visit How would you counsel the family

A They should be referred to surgery for appendectomy

B Their child is at higher risk for appendicitis so they should be aware of this in case he develops symptoms

C It is uncertain whether this child is at higher risk for appendicitis

D A course of Miralax may help wash out the fecalith

Take home points

Uncomplicated appendicitis

ndash Lap appy is still standard of care

ndash Non-operative management may be an acceptable option but not enough is known about long-term risk of recurrent appendicitis

Complicated appendicitis

ndash Can be managed with immediate operation delayed appendectomy or no appendectomy

ndash Immediate operation is probably more efficient and less stressful for patients and parents

Thank you very much

Questions

Page 16: New approaches for evaluation and treatment of appendicitis · Ultrasound or CT scan showing appendicitis with an appendix ≤ 1.1 centimeter thick and no evidence of abscess or fecalith

You are seeing a child in your office with 1-day history of

RLQ pain You are suspicious for appendicitis Where would

you send the patient

A Emergency room

B Same day outpatient surgery clinic

C CT scan

D Ultrasound

E Other

What to do with suspected appendicitis

Obtain imaging vs send to ED

ndash If any suspicion for viral syndrome with dehydration to ED

ndash If VERY HIGH suspicion for appendicitis and good pediatric radiologist is available consider sending child for ultrasound

Operating Room

further wu per surgeon

Further workup and imaging in

consultation with

Surgery team

1 IV placed 20 cckg bolus CBC

2 Surgical consult within one hour

Strong suspicion on

HPE Equivocal history

physical exam

1 IV placed 20 cckg bolus CBC

2 Re-examine surgical consult within one

hour if still tender

Abdominal pain

ro appendicitis

What is the best way to image the appendix

A CT scan

B Ultrasound

C MRI

D Abdominal X ray

E Other

Ultrasound

CT scan

MRI

Summary of approach to imaging

Ultrasound is good if you have a high pretest probability good radiologist thin patient

MRI is good if you are in a center that uses it routinely to evaluate children with appendicitis

CT scan is best if above criteria are not met

Alternative to imaging

ndash OBSERVATION

Once diagnosis of appendicitis has been establishedhellip

Non-ruptured

ndash Short duration of symptoms no suggestion of rupture on imaging

ndash Start antibiotics

ndash Perform appendectomy

Ruptured

ndash Have radiology evaluate for drainable fluid collection

ndash Antibiotics

ndash Operation if above management fails

Once diagnosis of appendicitis has been establishedhellip

Non-ruptured

ndash Short duration of symptoms no suggestion of rupture on imaging

ndash Start antibiotics

ndash Perform appendectomy

Ruptured

ndash Have radiology evaluate for drainable fluid collection

ndash Antibiotics

ndash Operation if above management fails

When should appendectomy be performed in a

patient with non-ruptured acute appendicitis

A Immediately after diagnosis

B Within first 24 hours of hospital stay once appropriate antibiotics and IVF resuscitation have been provided

C After completion of a one-week course of antibiotics

D It is reasonable to treat these patients with antibiotics alone and not perform an appendectomy

Is the appendix a ticking time bomb

Traditional teaching appendicitis is an emergency and should be treated with surgery immediately upon diagnosis

Several studies have now shown that delays of 12 hours up to 48 hours do not increase risk of perforation

Current standard admit IVF resuscitation antibiotics appendectomy during daylight hours

Almstroumlm M1 Svensson JF Patkova B Svenningsson A Wester TAnn Surg 2016 Mar 8 In-hospital

Surgical Delay Does Not Increase the Risk for Perforated Appendicitis in Children A Single-center

Retrospective Cohort Study

Port placement

Lap appy with Endo-loops

Lap appy with stapler

Open appendectomy

Appendix with fecalith

Laparoscopic vs open appendectomy

Laparoscopy is standard of care

Lower complication rate

Less scar

Less pain

Ability to evaluate other intra-abdominal organs if the appendix looks normal

ndash Gallbladder

ndash Ovaries

ndash Inguinal canal

Sauerland S Jaschinski T Neugebauer EA Laparoscopic versus open surgery for

suspected appendicitis Cochrane Database Syst Rev 2010 Oct 6(10)

Post-op course

Most patients can go home on the day of surgery

Recovery is usually quick

Back to school within a week sports within 2 weeks

Very low risk surgery with good outcomes

Risk of infection is about 5 at port sites or in abdomen

You see a 7 year old male in your office with 1-day history of abdominal pain

now localized to RLQ with focal guarding and poor appetite You send him for

ultrasound and it shows appendicitis His mother has heard about some new

research and wants to know if you would recommend just treating with

antibiotics rather than surgery How would you counsel this mom

A This data is preliminary and surgery is still standard of

care

B This may be a reasonable option for her son and she

should discuss it with her surgeon

C If it were your son you wouldnrsquot let him have surgery

D Other

Do all patients with appendicitis require surgery

Pilot study at Nationwide Childrenrsquos Hospital

102 patients

ndash 7 to 17 years of age

ndash Uncomplicated appendicitis defined by

Abdominal pain le 48 hours

White blood cell count le 18000

Ultrasound or CT scan showing appendicitis with an appendix le 11 centimeter thick and no

evidence of abscess or fecalith

Patients and families chose to have appendectomy or antibiotics alone

Non-operative management at least 24 hours of in-hospital observation and IV antibiotics until symptoms improved followed by completion of 10 days of treatment with antibiotics by mouth

Minneci PC Mahida JB et al The effectiveness of patient choice in non-operative

versus surgical management of uncomplicated acute appendicitis JAMA Surgery

2015 Dec 16

Results of non-operative management

65 families chose appendectomy 37 families chose non-operative management

Success rate of non-operative management (defined as not undergoing an appendectomy) 89 at 30 days 76 at 1 year

The 24 who failed did NOT have a higher rate of ruptured appendicitis compared to the patients who had immediate appendectomy

1 year follow-up the children managed non-operatively compared with the surgery group had fewer disability days (8 vs 21 days) lower appendicitis-related health care costs (median $4219 vs $5029) and no difference in health-related quality of life

Non-Operative Treatment of Appendicitis Rationale

Appendectomy is invasive

Children may miss up to two weeks of schoolactivities

Caregivers miss work

Postop complications after appendectomy for uncomplicated appendicitis 5-10

Serious complications (reoperations or readmissions) 1-7

Adult data suggest one-year success rates of 63-85 no difference in rates of complicated appendicitis

bull Meta-analysis of 10 articles reporting 413 children receiving non

operative treatment (NOT) for appendicitis ndash all published in past 10

years

bull 5 comparative

bull 1 RCT

bull 4 case series of NOT

Interpreting meta-analyses Forest plot

Favors

antibiotics

Favors

appendectomy

Overall success of NOT

97 during initial episode

Incidence of recurrent appendicitis during follow-up period (range 2-51

months)

Long term efficacy of NOT (no appendectomy at end of follow-up

period)

LOS shortened by about 05 days in those undergoing appendectomy

compared to NOT

Increased rate of complications for appendectomy compared to NOT

Authorsrsquo Conclusions

ldquoCurrent data suggest that NOT is safe It appears effective as initial treatment in 97 of children with AUA and the rate of recurrent appendicitis is 14rdquo

The study highlights the lack of robust evidence comparing NOT with appendectomy in children

Confirms a position of equipoise between treatment approaches

We recommend that NOT of children with AUA be reserved for those participating in carefully designed research studies

Take-home point

Lap appy is still standard of care for uncomplicated appendicitis

Once diagnosis of appendicitis has been establishedhellip

Non-ruptured

ndash Short duration of symptoms no suggestion of rupture on imaging

ndash Start antibiotics

ndash Perform appendectomy

Ruptured

ndash Have radiology evaluate for drainable fluid collection

ndash Antibiotics

ndash Operation in acute setting if above management fails

ndash Interval appendectomy 6-8 weeks laterhellip

ndash ALTERNATIVELY Just take out the appendix

CT ruptured appendix with abscess

After placement of percutaneous drain

How to counsel families of children with ruptured appendicitis

If kids tolerate a diet pain resolves fever resolves oral antibiotics and home

Plan interval appy 6-8 weeks later

Non-operative management may not work and surgery may be needed (Failure rate 20)

At home they should watch for signs of persistentrecurrent appendicitis

High-anxiety time for patients and families

A 10-year-old boy comes to see you in the office after recent

hospitalization for perforated appendicitis He has 3 more days of

antibiotics left His mom is worried because appetite is poor and his

energy level is low On exam he has diffuse lower abdominal

tenderness You recommend

A Extending course of oral antibiotics

B CT scan to evaluate for persistent or

recurrent appendicitis

C CT scan to evaluate for intra-abdominal

abscess

D Follow-up with surgeon

Is interval appendectomy necessary

In adults many surgeons do not do this operation

In kids data are limited

ndash 2-year follow-up of 96 patients

ndash Perforated appendicitis treated non-operatively with antibiotics

ndash 6 became worse 41 had interval appendectomy

ndash 49 received no further treatment

ndash 57 no recurrence

ndash 43 had recurrence within one month to 2 years

ndash Presence of appendicolith 72 rate of recurrence vs 26 in those without appendicolith

Ein SH1 Langer JC Daneman A Nonoperative management of pediatric ruptured appendix

with inflammatory mass or abscess presence of an appendicolith predicts recurrent

appendicitis J Pediatr Surg 2005 Oct40(10)1612-5

Approach to interval appendectomy

More strongly recommended if fecalith present

Observation is a reasonable option

Best choice for an individual patient depends on their anxiety and parental anxiety

Immediate operation for ruptured appendicitis perhaps a better

option

Meta-analysis operative vs non-operative management of pediatric

ruptured appendicitis

2 RCTs identified

Total of 171 pediatric patients

Compared early vs interval appendectomy

Early appendectomy reduced incidence of adverse event

Ruptured appendicitis may have abscess or phlegmon

Early surgery was more strongly favored when there was no abscess at

time of presentation

Decreased antibiotic duration length of stay and total charges for

abscess and no abscess groups

You are seeing a 7 year old with constipation A

fecalith was seen on abdominal X ray during recent

ED visit How would you counsel the family

A They should be referred to surgery for appendectomy

B Their child is at higher risk for appendicitis so they should be aware of this in case he develops symptoms

C It is uncertain whether this child is at higher risk for appendicitis

D A course of Miralax may help wash out the fecalith

Take home points

Uncomplicated appendicitis

ndash Lap appy is still standard of care

ndash Non-operative management may be an acceptable option but not enough is known about long-term risk of recurrent appendicitis

Complicated appendicitis

ndash Can be managed with immediate operation delayed appendectomy or no appendectomy

ndash Immediate operation is probably more efficient and less stressful for patients and parents

Thank you very much

Questions

Page 17: New approaches for evaluation and treatment of appendicitis · Ultrasound or CT scan showing appendicitis with an appendix ≤ 1.1 centimeter thick and no evidence of abscess or fecalith

What to do with suspected appendicitis

Obtain imaging vs send to ED

ndash If any suspicion for viral syndrome with dehydration to ED

ndash If VERY HIGH suspicion for appendicitis and good pediatric radiologist is available consider sending child for ultrasound

Operating Room

further wu per surgeon

Further workup and imaging in

consultation with

Surgery team

1 IV placed 20 cckg bolus CBC

2 Surgical consult within one hour

Strong suspicion on

HPE Equivocal history

physical exam

1 IV placed 20 cckg bolus CBC

2 Re-examine surgical consult within one

hour if still tender

Abdominal pain

ro appendicitis

What is the best way to image the appendix

A CT scan

B Ultrasound

C MRI

D Abdominal X ray

E Other

Ultrasound

CT scan

MRI

Summary of approach to imaging

Ultrasound is good if you have a high pretest probability good radiologist thin patient

MRI is good if you are in a center that uses it routinely to evaluate children with appendicitis

CT scan is best if above criteria are not met

Alternative to imaging

ndash OBSERVATION

Once diagnosis of appendicitis has been establishedhellip

Non-ruptured

ndash Short duration of symptoms no suggestion of rupture on imaging

ndash Start antibiotics

ndash Perform appendectomy

Ruptured

ndash Have radiology evaluate for drainable fluid collection

ndash Antibiotics

ndash Operation if above management fails

Once diagnosis of appendicitis has been establishedhellip

Non-ruptured

ndash Short duration of symptoms no suggestion of rupture on imaging

ndash Start antibiotics

ndash Perform appendectomy

Ruptured

ndash Have radiology evaluate for drainable fluid collection

ndash Antibiotics

ndash Operation if above management fails

When should appendectomy be performed in a

patient with non-ruptured acute appendicitis

A Immediately after diagnosis

B Within first 24 hours of hospital stay once appropriate antibiotics and IVF resuscitation have been provided

C After completion of a one-week course of antibiotics

D It is reasonable to treat these patients with antibiotics alone and not perform an appendectomy

Is the appendix a ticking time bomb

Traditional teaching appendicitis is an emergency and should be treated with surgery immediately upon diagnosis

Several studies have now shown that delays of 12 hours up to 48 hours do not increase risk of perforation

Current standard admit IVF resuscitation antibiotics appendectomy during daylight hours

Almstroumlm M1 Svensson JF Patkova B Svenningsson A Wester TAnn Surg 2016 Mar 8 In-hospital

Surgical Delay Does Not Increase the Risk for Perforated Appendicitis in Children A Single-center

Retrospective Cohort Study

Port placement

Lap appy with Endo-loops

Lap appy with stapler

Open appendectomy

Appendix with fecalith

Laparoscopic vs open appendectomy

Laparoscopy is standard of care

Lower complication rate

Less scar

Less pain

Ability to evaluate other intra-abdominal organs if the appendix looks normal

ndash Gallbladder

ndash Ovaries

ndash Inguinal canal

Sauerland S Jaschinski T Neugebauer EA Laparoscopic versus open surgery for

suspected appendicitis Cochrane Database Syst Rev 2010 Oct 6(10)

Post-op course

Most patients can go home on the day of surgery

Recovery is usually quick

Back to school within a week sports within 2 weeks

Very low risk surgery with good outcomes

Risk of infection is about 5 at port sites or in abdomen

You see a 7 year old male in your office with 1-day history of abdominal pain

now localized to RLQ with focal guarding and poor appetite You send him for

ultrasound and it shows appendicitis His mother has heard about some new

research and wants to know if you would recommend just treating with

antibiotics rather than surgery How would you counsel this mom

A This data is preliminary and surgery is still standard of

care

B This may be a reasonable option for her son and she

should discuss it with her surgeon

C If it were your son you wouldnrsquot let him have surgery

D Other

Do all patients with appendicitis require surgery

Pilot study at Nationwide Childrenrsquos Hospital

102 patients

ndash 7 to 17 years of age

ndash Uncomplicated appendicitis defined by

Abdominal pain le 48 hours

White blood cell count le 18000

Ultrasound or CT scan showing appendicitis with an appendix le 11 centimeter thick and no

evidence of abscess or fecalith

Patients and families chose to have appendectomy or antibiotics alone

Non-operative management at least 24 hours of in-hospital observation and IV antibiotics until symptoms improved followed by completion of 10 days of treatment with antibiotics by mouth

Minneci PC Mahida JB et al The effectiveness of patient choice in non-operative

versus surgical management of uncomplicated acute appendicitis JAMA Surgery

2015 Dec 16

Results of non-operative management

65 families chose appendectomy 37 families chose non-operative management

Success rate of non-operative management (defined as not undergoing an appendectomy) 89 at 30 days 76 at 1 year

The 24 who failed did NOT have a higher rate of ruptured appendicitis compared to the patients who had immediate appendectomy

1 year follow-up the children managed non-operatively compared with the surgery group had fewer disability days (8 vs 21 days) lower appendicitis-related health care costs (median $4219 vs $5029) and no difference in health-related quality of life

Non-Operative Treatment of Appendicitis Rationale

Appendectomy is invasive

Children may miss up to two weeks of schoolactivities

Caregivers miss work

Postop complications after appendectomy for uncomplicated appendicitis 5-10

Serious complications (reoperations or readmissions) 1-7

Adult data suggest one-year success rates of 63-85 no difference in rates of complicated appendicitis

bull Meta-analysis of 10 articles reporting 413 children receiving non

operative treatment (NOT) for appendicitis ndash all published in past 10

years

bull 5 comparative

bull 1 RCT

bull 4 case series of NOT

Interpreting meta-analyses Forest plot

Favors

antibiotics

Favors

appendectomy

Overall success of NOT

97 during initial episode

Incidence of recurrent appendicitis during follow-up period (range 2-51

months)

Long term efficacy of NOT (no appendectomy at end of follow-up

period)

LOS shortened by about 05 days in those undergoing appendectomy

compared to NOT

Increased rate of complications for appendectomy compared to NOT

Authorsrsquo Conclusions

ldquoCurrent data suggest that NOT is safe It appears effective as initial treatment in 97 of children with AUA and the rate of recurrent appendicitis is 14rdquo

The study highlights the lack of robust evidence comparing NOT with appendectomy in children

Confirms a position of equipoise between treatment approaches

We recommend that NOT of children with AUA be reserved for those participating in carefully designed research studies

Take-home point

Lap appy is still standard of care for uncomplicated appendicitis

Once diagnosis of appendicitis has been establishedhellip

Non-ruptured

ndash Short duration of symptoms no suggestion of rupture on imaging

ndash Start antibiotics

ndash Perform appendectomy

Ruptured

ndash Have radiology evaluate for drainable fluid collection

ndash Antibiotics

ndash Operation in acute setting if above management fails

ndash Interval appendectomy 6-8 weeks laterhellip

ndash ALTERNATIVELY Just take out the appendix

CT ruptured appendix with abscess

After placement of percutaneous drain

How to counsel families of children with ruptured appendicitis

If kids tolerate a diet pain resolves fever resolves oral antibiotics and home

Plan interval appy 6-8 weeks later

Non-operative management may not work and surgery may be needed (Failure rate 20)

At home they should watch for signs of persistentrecurrent appendicitis

High-anxiety time for patients and families

A 10-year-old boy comes to see you in the office after recent

hospitalization for perforated appendicitis He has 3 more days of

antibiotics left His mom is worried because appetite is poor and his

energy level is low On exam he has diffuse lower abdominal

tenderness You recommend

A Extending course of oral antibiotics

B CT scan to evaluate for persistent or

recurrent appendicitis

C CT scan to evaluate for intra-abdominal

abscess

D Follow-up with surgeon

Is interval appendectomy necessary

In adults many surgeons do not do this operation

In kids data are limited

ndash 2-year follow-up of 96 patients

ndash Perforated appendicitis treated non-operatively with antibiotics

ndash 6 became worse 41 had interval appendectomy

ndash 49 received no further treatment

ndash 57 no recurrence

ndash 43 had recurrence within one month to 2 years

ndash Presence of appendicolith 72 rate of recurrence vs 26 in those without appendicolith

Ein SH1 Langer JC Daneman A Nonoperative management of pediatric ruptured appendix

with inflammatory mass or abscess presence of an appendicolith predicts recurrent

appendicitis J Pediatr Surg 2005 Oct40(10)1612-5

Approach to interval appendectomy

More strongly recommended if fecalith present

Observation is a reasonable option

Best choice for an individual patient depends on their anxiety and parental anxiety

Immediate operation for ruptured appendicitis perhaps a better

option

Meta-analysis operative vs non-operative management of pediatric

ruptured appendicitis

2 RCTs identified

Total of 171 pediatric patients

Compared early vs interval appendectomy

Early appendectomy reduced incidence of adverse event

Ruptured appendicitis may have abscess or phlegmon

Early surgery was more strongly favored when there was no abscess at

time of presentation

Decreased antibiotic duration length of stay and total charges for

abscess and no abscess groups

You are seeing a 7 year old with constipation A

fecalith was seen on abdominal X ray during recent

ED visit How would you counsel the family

A They should be referred to surgery for appendectomy

B Their child is at higher risk for appendicitis so they should be aware of this in case he develops symptoms

C It is uncertain whether this child is at higher risk for appendicitis

D A course of Miralax may help wash out the fecalith

Take home points

Uncomplicated appendicitis

ndash Lap appy is still standard of care

ndash Non-operative management may be an acceptable option but not enough is known about long-term risk of recurrent appendicitis

Complicated appendicitis

ndash Can be managed with immediate operation delayed appendectomy or no appendectomy

ndash Immediate operation is probably more efficient and less stressful for patients and parents

Thank you very much

Questions

Page 18: New approaches for evaluation and treatment of appendicitis · Ultrasound or CT scan showing appendicitis with an appendix ≤ 1.1 centimeter thick and no evidence of abscess or fecalith

Operating Room

further wu per surgeon

Further workup and imaging in

consultation with

Surgery team

1 IV placed 20 cckg bolus CBC

2 Surgical consult within one hour

Strong suspicion on

HPE Equivocal history

physical exam

1 IV placed 20 cckg bolus CBC

2 Re-examine surgical consult within one

hour if still tender

Abdominal pain

ro appendicitis

What is the best way to image the appendix

A CT scan

B Ultrasound

C MRI

D Abdominal X ray

E Other

Ultrasound

CT scan

MRI

Summary of approach to imaging

Ultrasound is good if you have a high pretest probability good radiologist thin patient

MRI is good if you are in a center that uses it routinely to evaluate children with appendicitis

CT scan is best if above criteria are not met

Alternative to imaging

ndash OBSERVATION

Once diagnosis of appendicitis has been establishedhellip

Non-ruptured

ndash Short duration of symptoms no suggestion of rupture on imaging

ndash Start antibiotics

ndash Perform appendectomy

Ruptured

ndash Have radiology evaluate for drainable fluid collection

ndash Antibiotics

ndash Operation if above management fails

Once diagnosis of appendicitis has been establishedhellip

Non-ruptured

ndash Short duration of symptoms no suggestion of rupture on imaging

ndash Start antibiotics

ndash Perform appendectomy

Ruptured

ndash Have radiology evaluate for drainable fluid collection

ndash Antibiotics

ndash Operation if above management fails

When should appendectomy be performed in a

patient with non-ruptured acute appendicitis

A Immediately after diagnosis

B Within first 24 hours of hospital stay once appropriate antibiotics and IVF resuscitation have been provided

C After completion of a one-week course of antibiotics

D It is reasonable to treat these patients with antibiotics alone and not perform an appendectomy

Is the appendix a ticking time bomb

Traditional teaching appendicitis is an emergency and should be treated with surgery immediately upon diagnosis

Several studies have now shown that delays of 12 hours up to 48 hours do not increase risk of perforation

Current standard admit IVF resuscitation antibiotics appendectomy during daylight hours

Almstroumlm M1 Svensson JF Patkova B Svenningsson A Wester TAnn Surg 2016 Mar 8 In-hospital

Surgical Delay Does Not Increase the Risk for Perforated Appendicitis in Children A Single-center

Retrospective Cohort Study

Port placement

Lap appy with Endo-loops

Lap appy with stapler

Open appendectomy

Appendix with fecalith

Laparoscopic vs open appendectomy

Laparoscopy is standard of care

Lower complication rate

Less scar

Less pain

Ability to evaluate other intra-abdominal organs if the appendix looks normal

ndash Gallbladder

ndash Ovaries

ndash Inguinal canal

Sauerland S Jaschinski T Neugebauer EA Laparoscopic versus open surgery for

suspected appendicitis Cochrane Database Syst Rev 2010 Oct 6(10)

Post-op course

Most patients can go home on the day of surgery

Recovery is usually quick

Back to school within a week sports within 2 weeks

Very low risk surgery with good outcomes

Risk of infection is about 5 at port sites or in abdomen

You see a 7 year old male in your office with 1-day history of abdominal pain

now localized to RLQ with focal guarding and poor appetite You send him for

ultrasound and it shows appendicitis His mother has heard about some new

research and wants to know if you would recommend just treating with

antibiotics rather than surgery How would you counsel this mom

A This data is preliminary and surgery is still standard of

care

B This may be a reasonable option for her son and she

should discuss it with her surgeon

C If it were your son you wouldnrsquot let him have surgery

D Other

Do all patients with appendicitis require surgery

Pilot study at Nationwide Childrenrsquos Hospital

102 patients

ndash 7 to 17 years of age

ndash Uncomplicated appendicitis defined by

Abdominal pain le 48 hours

White blood cell count le 18000

Ultrasound or CT scan showing appendicitis with an appendix le 11 centimeter thick and no

evidence of abscess or fecalith

Patients and families chose to have appendectomy or antibiotics alone

Non-operative management at least 24 hours of in-hospital observation and IV antibiotics until symptoms improved followed by completion of 10 days of treatment with antibiotics by mouth

Minneci PC Mahida JB et al The effectiveness of patient choice in non-operative

versus surgical management of uncomplicated acute appendicitis JAMA Surgery

2015 Dec 16

Results of non-operative management

65 families chose appendectomy 37 families chose non-operative management

Success rate of non-operative management (defined as not undergoing an appendectomy) 89 at 30 days 76 at 1 year

The 24 who failed did NOT have a higher rate of ruptured appendicitis compared to the patients who had immediate appendectomy

1 year follow-up the children managed non-operatively compared with the surgery group had fewer disability days (8 vs 21 days) lower appendicitis-related health care costs (median $4219 vs $5029) and no difference in health-related quality of life

Non-Operative Treatment of Appendicitis Rationale

Appendectomy is invasive

Children may miss up to two weeks of schoolactivities

Caregivers miss work

Postop complications after appendectomy for uncomplicated appendicitis 5-10

Serious complications (reoperations or readmissions) 1-7

Adult data suggest one-year success rates of 63-85 no difference in rates of complicated appendicitis

bull Meta-analysis of 10 articles reporting 413 children receiving non

operative treatment (NOT) for appendicitis ndash all published in past 10

years

bull 5 comparative

bull 1 RCT

bull 4 case series of NOT

Interpreting meta-analyses Forest plot

Favors

antibiotics

Favors

appendectomy

Overall success of NOT

97 during initial episode

Incidence of recurrent appendicitis during follow-up period (range 2-51

months)

Long term efficacy of NOT (no appendectomy at end of follow-up

period)

LOS shortened by about 05 days in those undergoing appendectomy

compared to NOT

Increased rate of complications for appendectomy compared to NOT

Authorsrsquo Conclusions

ldquoCurrent data suggest that NOT is safe It appears effective as initial treatment in 97 of children with AUA and the rate of recurrent appendicitis is 14rdquo

The study highlights the lack of robust evidence comparing NOT with appendectomy in children

Confirms a position of equipoise between treatment approaches

We recommend that NOT of children with AUA be reserved for those participating in carefully designed research studies

Take-home point

Lap appy is still standard of care for uncomplicated appendicitis

Once diagnosis of appendicitis has been establishedhellip

Non-ruptured

ndash Short duration of symptoms no suggestion of rupture on imaging

ndash Start antibiotics

ndash Perform appendectomy

Ruptured

ndash Have radiology evaluate for drainable fluid collection

ndash Antibiotics

ndash Operation in acute setting if above management fails

ndash Interval appendectomy 6-8 weeks laterhellip

ndash ALTERNATIVELY Just take out the appendix

CT ruptured appendix with abscess

After placement of percutaneous drain

How to counsel families of children with ruptured appendicitis

If kids tolerate a diet pain resolves fever resolves oral antibiotics and home

Plan interval appy 6-8 weeks later

Non-operative management may not work and surgery may be needed (Failure rate 20)

At home they should watch for signs of persistentrecurrent appendicitis

High-anxiety time for patients and families

A 10-year-old boy comes to see you in the office after recent

hospitalization for perforated appendicitis He has 3 more days of

antibiotics left His mom is worried because appetite is poor and his

energy level is low On exam he has diffuse lower abdominal

tenderness You recommend

A Extending course of oral antibiotics

B CT scan to evaluate for persistent or

recurrent appendicitis

C CT scan to evaluate for intra-abdominal

abscess

D Follow-up with surgeon

Is interval appendectomy necessary

In adults many surgeons do not do this operation

In kids data are limited

ndash 2-year follow-up of 96 patients

ndash Perforated appendicitis treated non-operatively with antibiotics

ndash 6 became worse 41 had interval appendectomy

ndash 49 received no further treatment

ndash 57 no recurrence

ndash 43 had recurrence within one month to 2 years

ndash Presence of appendicolith 72 rate of recurrence vs 26 in those without appendicolith

Ein SH1 Langer JC Daneman A Nonoperative management of pediatric ruptured appendix

with inflammatory mass or abscess presence of an appendicolith predicts recurrent

appendicitis J Pediatr Surg 2005 Oct40(10)1612-5

Approach to interval appendectomy

More strongly recommended if fecalith present

Observation is a reasonable option

Best choice for an individual patient depends on their anxiety and parental anxiety

Immediate operation for ruptured appendicitis perhaps a better

option

Meta-analysis operative vs non-operative management of pediatric

ruptured appendicitis

2 RCTs identified

Total of 171 pediatric patients

Compared early vs interval appendectomy

Early appendectomy reduced incidence of adverse event

Ruptured appendicitis may have abscess or phlegmon

Early surgery was more strongly favored when there was no abscess at

time of presentation

Decreased antibiotic duration length of stay and total charges for

abscess and no abscess groups

You are seeing a 7 year old with constipation A

fecalith was seen on abdominal X ray during recent

ED visit How would you counsel the family

A They should be referred to surgery for appendectomy

B Their child is at higher risk for appendicitis so they should be aware of this in case he develops symptoms

C It is uncertain whether this child is at higher risk for appendicitis

D A course of Miralax may help wash out the fecalith

Take home points

Uncomplicated appendicitis

ndash Lap appy is still standard of care

ndash Non-operative management may be an acceptable option but not enough is known about long-term risk of recurrent appendicitis

Complicated appendicitis

ndash Can be managed with immediate operation delayed appendectomy or no appendectomy

ndash Immediate operation is probably more efficient and less stressful for patients and parents

Thank you very much

Questions

Page 19: New approaches for evaluation and treatment of appendicitis · Ultrasound or CT scan showing appendicitis with an appendix ≤ 1.1 centimeter thick and no evidence of abscess or fecalith

What is the best way to image the appendix

A CT scan

B Ultrasound

C MRI

D Abdominal X ray

E Other

Ultrasound

CT scan

MRI

Summary of approach to imaging

Ultrasound is good if you have a high pretest probability good radiologist thin patient

MRI is good if you are in a center that uses it routinely to evaluate children with appendicitis

CT scan is best if above criteria are not met

Alternative to imaging

ndash OBSERVATION

Once diagnosis of appendicitis has been establishedhellip

Non-ruptured

ndash Short duration of symptoms no suggestion of rupture on imaging

ndash Start antibiotics

ndash Perform appendectomy

Ruptured

ndash Have radiology evaluate for drainable fluid collection

ndash Antibiotics

ndash Operation if above management fails

Once diagnosis of appendicitis has been establishedhellip

Non-ruptured

ndash Short duration of symptoms no suggestion of rupture on imaging

ndash Start antibiotics

ndash Perform appendectomy

Ruptured

ndash Have radiology evaluate for drainable fluid collection

ndash Antibiotics

ndash Operation if above management fails

When should appendectomy be performed in a

patient with non-ruptured acute appendicitis

A Immediately after diagnosis

B Within first 24 hours of hospital stay once appropriate antibiotics and IVF resuscitation have been provided

C After completion of a one-week course of antibiotics

D It is reasonable to treat these patients with antibiotics alone and not perform an appendectomy

Is the appendix a ticking time bomb

Traditional teaching appendicitis is an emergency and should be treated with surgery immediately upon diagnosis

Several studies have now shown that delays of 12 hours up to 48 hours do not increase risk of perforation

Current standard admit IVF resuscitation antibiotics appendectomy during daylight hours

Almstroumlm M1 Svensson JF Patkova B Svenningsson A Wester TAnn Surg 2016 Mar 8 In-hospital

Surgical Delay Does Not Increase the Risk for Perforated Appendicitis in Children A Single-center

Retrospective Cohort Study

Port placement

Lap appy with Endo-loops

Lap appy with stapler

Open appendectomy

Appendix with fecalith

Laparoscopic vs open appendectomy

Laparoscopy is standard of care

Lower complication rate

Less scar

Less pain

Ability to evaluate other intra-abdominal organs if the appendix looks normal

ndash Gallbladder

ndash Ovaries

ndash Inguinal canal

Sauerland S Jaschinski T Neugebauer EA Laparoscopic versus open surgery for

suspected appendicitis Cochrane Database Syst Rev 2010 Oct 6(10)

Post-op course

Most patients can go home on the day of surgery

Recovery is usually quick

Back to school within a week sports within 2 weeks

Very low risk surgery with good outcomes

Risk of infection is about 5 at port sites or in abdomen

You see a 7 year old male in your office with 1-day history of abdominal pain

now localized to RLQ with focal guarding and poor appetite You send him for

ultrasound and it shows appendicitis His mother has heard about some new

research and wants to know if you would recommend just treating with

antibiotics rather than surgery How would you counsel this mom

A This data is preliminary and surgery is still standard of

care

B This may be a reasonable option for her son and she

should discuss it with her surgeon

C If it were your son you wouldnrsquot let him have surgery

D Other

Do all patients with appendicitis require surgery

Pilot study at Nationwide Childrenrsquos Hospital

102 patients

ndash 7 to 17 years of age

ndash Uncomplicated appendicitis defined by

Abdominal pain le 48 hours

White blood cell count le 18000

Ultrasound or CT scan showing appendicitis with an appendix le 11 centimeter thick and no

evidence of abscess or fecalith

Patients and families chose to have appendectomy or antibiotics alone

Non-operative management at least 24 hours of in-hospital observation and IV antibiotics until symptoms improved followed by completion of 10 days of treatment with antibiotics by mouth

Minneci PC Mahida JB et al The effectiveness of patient choice in non-operative

versus surgical management of uncomplicated acute appendicitis JAMA Surgery

2015 Dec 16

Results of non-operative management

65 families chose appendectomy 37 families chose non-operative management

Success rate of non-operative management (defined as not undergoing an appendectomy) 89 at 30 days 76 at 1 year

The 24 who failed did NOT have a higher rate of ruptured appendicitis compared to the patients who had immediate appendectomy

1 year follow-up the children managed non-operatively compared with the surgery group had fewer disability days (8 vs 21 days) lower appendicitis-related health care costs (median $4219 vs $5029) and no difference in health-related quality of life

Non-Operative Treatment of Appendicitis Rationale

Appendectomy is invasive

Children may miss up to two weeks of schoolactivities

Caregivers miss work

Postop complications after appendectomy for uncomplicated appendicitis 5-10

Serious complications (reoperations or readmissions) 1-7

Adult data suggest one-year success rates of 63-85 no difference in rates of complicated appendicitis

bull Meta-analysis of 10 articles reporting 413 children receiving non

operative treatment (NOT) for appendicitis ndash all published in past 10

years

bull 5 comparative

bull 1 RCT

bull 4 case series of NOT

Interpreting meta-analyses Forest plot

Favors

antibiotics

Favors

appendectomy

Overall success of NOT

97 during initial episode

Incidence of recurrent appendicitis during follow-up period (range 2-51

months)

Long term efficacy of NOT (no appendectomy at end of follow-up

period)

LOS shortened by about 05 days in those undergoing appendectomy

compared to NOT

Increased rate of complications for appendectomy compared to NOT

Authorsrsquo Conclusions

ldquoCurrent data suggest that NOT is safe It appears effective as initial treatment in 97 of children with AUA and the rate of recurrent appendicitis is 14rdquo

The study highlights the lack of robust evidence comparing NOT with appendectomy in children

Confirms a position of equipoise between treatment approaches

We recommend that NOT of children with AUA be reserved for those participating in carefully designed research studies

Take-home point

Lap appy is still standard of care for uncomplicated appendicitis

Once diagnosis of appendicitis has been establishedhellip

Non-ruptured

ndash Short duration of symptoms no suggestion of rupture on imaging

ndash Start antibiotics

ndash Perform appendectomy

Ruptured

ndash Have radiology evaluate for drainable fluid collection

ndash Antibiotics

ndash Operation in acute setting if above management fails

ndash Interval appendectomy 6-8 weeks laterhellip

ndash ALTERNATIVELY Just take out the appendix

CT ruptured appendix with abscess

After placement of percutaneous drain

How to counsel families of children with ruptured appendicitis

If kids tolerate a diet pain resolves fever resolves oral antibiotics and home

Plan interval appy 6-8 weeks later

Non-operative management may not work and surgery may be needed (Failure rate 20)

At home they should watch for signs of persistentrecurrent appendicitis

High-anxiety time for patients and families

A 10-year-old boy comes to see you in the office after recent

hospitalization for perforated appendicitis He has 3 more days of

antibiotics left His mom is worried because appetite is poor and his

energy level is low On exam he has diffuse lower abdominal

tenderness You recommend

A Extending course of oral antibiotics

B CT scan to evaluate for persistent or

recurrent appendicitis

C CT scan to evaluate for intra-abdominal

abscess

D Follow-up with surgeon

Is interval appendectomy necessary

In adults many surgeons do not do this operation

In kids data are limited

ndash 2-year follow-up of 96 patients

ndash Perforated appendicitis treated non-operatively with antibiotics

ndash 6 became worse 41 had interval appendectomy

ndash 49 received no further treatment

ndash 57 no recurrence

ndash 43 had recurrence within one month to 2 years

ndash Presence of appendicolith 72 rate of recurrence vs 26 in those without appendicolith

Ein SH1 Langer JC Daneman A Nonoperative management of pediatric ruptured appendix

with inflammatory mass or abscess presence of an appendicolith predicts recurrent

appendicitis J Pediatr Surg 2005 Oct40(10)1612-5

Approach to interval appendectomy

More strongly recommended if fecalith present

Observation is a reasonable option

Best choice for an individual patient depends on their anxiety and parental anxiety

Immediate operation for ruptured appendicitis perhaps a better

option

Meta-analysis operative vs non-operative management of pediatric

ruptured appendicitis

2 RCTs identified

Total of 171 pediatric patients

Compared early vs interval appendectomy

Early appendectomy reduced incidence of adverse event

Ruptured appendicitis may have abscess or phlegmon

Early surgery was more strongly favored when there was no abscess at

time of presentation

Decreased antibiotic duration length of stay and total charges for

abscess and no abscess groups

You are seeing a 7 year old with constipation A

fecalith was seen on abdominal X ray during recent

ED visit How would you counsel the family

A They should be referred to surgery for appendectomy

B Their child is at higher risk for appendicitis so they should be aware of this in case he develops symptoms

C It is uncertain whether this child is at higher risk for appendicitis

D A course of Miralax may help wash out the fecalith

Take home points

Uncomplicated appendicitis

ndash Lap appy is still standard of care

ndash Non-operative management may be an acceptable option but not enough is known about long-term risk of recurrent appendicitis

Complicated appendicitis

ndash Can be managed with immediate operation delayed appendectomy or no appendectomy

ndash Immediate operation is probably more efficient and less stressful for patients and parents

Thank you very much

Questions

Page 20: New approaches for evaluation and treatment of appendicitis · Ultrasound or CT scan showing appendicitis with an appendix ≤ 1.1 centimeter thick and no evidence of abscess or fecalith

Ultrasound

CT scan

MRI

Summary of approach to imaging

Ultrasound is good if you have a high pretest probability good radiologist thin patient

MRI is good if you are in a center that uses it routinely to evaluate children with appendicitis

CT scan is best if above criteria are not met

Alternative to imaging

ndash OBSERVATION

Once diagnosis of appendicitis has been establishedhellip

Non-ruptured

ndash Short duration of symptoms no suggestion of rupture on imaging

ndash Start antibiotics

ndash Perform appendectomy

Ruptured

ndash Have radiology evaluate for drainable fluid collection

ndash Antibiotics

ndash Operation if above management fails

Once diagnosis of appendicitis has been establishedhellip

Non-ruptured

ndash Short duration of symptoms no suggestion of rupture on imaging

ndash Start antibiotics

ndash Perform appendectomy

Ruptured

ndash Have radiology evaluate for drainable fluid collection

ndash Antibiotics

ndash Operation if above management fails

When should appendectomy be performed in a

patient with non-ruptured acute appendicitis

A Immediately after diagnosis

B Within first 24 hours of hospital stay once appropriate antibiotics and IVF resuscitation have been provided

C After completion of a one-week course of antibiotics

D It is reasonable to treat these patients with antibiotics alone and not perform an appendectomy

Is the appendix a ticking time bomb

Traditional teaching appendicitis is an emergency and should be treated with surgery immediately upon diagnosis

Several studies have now shown that delays of 12 hours up to 48 hours do not increase risk of perforation

Current standard admit IVF resuscitation antibiotics appendectomy during daylight hours

Almstroumlm M1 Svensson JF Patkova B Svenningsson A Wester TAnn Surg 2016 Mar 8 In-hospital

Surgical Delay Does Not Increase the Risk for Perforated Appendicitis in Children A Single-center

Retrospective Cohort Study

Port placement

Lap appy with Endo-loops

Lap appy with stapler

Open appendectomy

Appendix with fecalith

Laparoscopic vs open appendectomy

Laparoscopy is standard of care

Lower complication rate

Less scar

Less pain

Ability to evaluate other intra-abdominal organs if the appendix looks normal

ndash Gallbladder

ndash Ovaries

ndash Inguinal canal

Sauerland S Jaschinski T Neugebauer EA Laparoscopic versus open surgery for

suspected appendicitis Cochrane Database Syst Rev 2010 Oct 6(10)

Post-op course

Most patients can go home on the day of surgery

Recovery is usually quick

Back to school within a week sports within 2 weeks

Very low risk surgery with good outcomes

Risk of infection is about 5 at port sites or in abdomen

You see a 7 year old male in your office with 1-day history of abdominal pain

now localized to RLQ with focal guarding and poor appetite You send him for

ultrasound and it shows appendicitis His mother has heard about some new

research and wants to know if you would recommend just treating with

antibiotics rather than surgery How would you counsel this mom

A This data is preliminary and surgery is still standard of

care

B This may be a reasonable option for her son and she

should discuss it with her surgeon

C If it were your son you wouldnrsquot let him have surgery

D Other

Do all patients with appendicitis require surgery

Pilot study at Nationwide Childrenrsquos Hospital

102 patients

ndash 7 to 17 years of age

ndash Uncomplicated appendicitis defined by

Abdominal pain le 48 hours

White blood cell count le 18000

Ultrasound or CT scan showing appendicitis with an appendix le 11 centimeter thick and no

evidence of abscess or fecalith

Patients and families chose to have appendectomy or antibiotics alone

Non-operative management at least 24 hours of in-hospital observation and IV antibiotics until symptoms improved followed by completion of 10 days of treatment with antibiotics by mouth

Minneci PC Mahida JB et al The effectiveness of patient choice in non-operative

versus surgical management of uncomplicated acute appendicitis JAMA Surgery

2015 Dec 16

Results of non-operative management

65 families chose appendectomy 37 families chose non-operative management

Success rate of non-operative management (defined as not undergoing an appendectomy) 89 at 30 days 76 at 1 year

The 24 who failed did NOT have a higher rate of ruptured appendicitis compared to the patients who had immediate appendectomy

1 year follow-up the children managed non-operatively compared with the surgery group had fewer disability days (8 vs 21 days) lower appendicitis-related health care costs (median $4219 vs $5029) and no difference in health-related quality of life

Non-Operative Treatment of Appendicitis Rationale

Appendectomy is invasive

Children may miss up to two weeks of schoolactivities

Caregivers miss work

Postop complications after appendectomy for uncomplicated appendicitis 5-10

Serious complications (reoperations or readmissions) 1-7

Adult data suggest one-year success rates of 63-85 no difference in rates of complicated appendicitis

bull Meta-analysis of 10 articles reporting 413 children receiving non

operative treatment (NOT) for appendicitis ndash all published in past 10

years

bull 5 comparative

bull 1 RCT

bull 4 case series of NOT

Interpreting meta-analyses Forest plot

Favors

antibiotics

Favors

appendectomy

Overall success of NOT

97 during initial episode

Incidence of recurrent appendicitis during follow-up period (range 2-51

months)

Long term efficacy of NOT (no appendectomy at end of follow-up

period)

LOS shortened by about 05 days in those undergoing appendectomy

compared to NOT

Increased rate of complications for appendectomy compared to NOT

Authorsrsquo Conclusions

ldquoCurrent data suggest that NOT is safe It appears effective as initial treatment in 97 of children with AUA and the rate of recurrent appendicitis is 14rdquo

The study highlights the lack of robust evidence comparing NOT with appendectomy in children

Confirms a position of equipoise between treatment approaches

We recommend that NOT of children with AUA be reserved for those participating in carefully designed research studies

Take-home point

Lap appy is still standard of care for uncomplicated appendicitis

Once diagnosis of appendicitis has been establishedhellip

Non-ruptured

ndash Short duration of symptoms no suggestion of rupture on imaging

ndash Start antibiotics

ndash Perform appendectomy

Ruptured

ndash Have radiology evaluate for drainable fluid collection

ndash Antibiotics

ndash Operation in acute setting if above management fails

ndash Interval appendectomy 6-8 weeks laterhellip

ndash ALTERNATIVELY Just take out the appendix

CT ruptured appendix with abscess

After placement of percutaneous drain

How to counsel families of children with ruptured appendicitis

If kids tolerate a diet pain resolves fever resolves oral antibiotics and home

Plan interval appy 6-8 weeks later

Non-operative management may not work and surgery may be needed (Failure rate 20)

At home they should watch for signs of persistentrecurrent appendicitis

High-anxiety time for patients and families

A 10-year-old boy comes to see you in the office after recent

hospitalization for perforated appendicitis He has 3 more days of

antibiotics left His mom is worried because appetite is poor and his

energy level is low On exam he has diffuse lower abdominal

tenderness You recommend

A Extending course of oral antibiotics

B CT scan to evaluate for persistent or

recurrent appendicitis

C CT scan to evaluate for intra-abdominal

abscess

D Follow-up with surgeon

Is interval appendectomy necessary

In adults many surgeons do not do this operation

In kids data are limited

ndash 2-year follow-up of 96 patients

ndash Perforated appendicitis treated non-operatively with antibiotics

ndash 6 became worse 41 had interval appendectomy

ndash 49 received no further treatment

ndash 57 no recurrence

ndash 43 had recurrence within one month to 2 years

ndash Presence of appendicolith 72 rate of recurrence vs 26 in those without appendicolith

Ein SH1 Langer JC Daneman A Nonoperative management of pediatric ruptured appendix

with inflammatory mass or abscess presence of an appendicolith predicts recurrent

appendicitis J Pediatr Surg 2005 Oct40(10)1612-5

Approach to interval appendectomy

More strongly recommended if fecalith present

Observation is a reasonable option

Best choice for an individual patient depends on their anxiety and parental anxiety

Immediate operation for ruptured appendicitis perhaps a better

option

Meta-analysis operative vs non-operative management of pediatric

ruptured appendicitis

2 RCTs identified

Total of 171 pediatric patients

Compared early vs interval appendectomy

Early appendectomy reduced incidence of adverse event

Ruptured appendicitis may have abscess or phlegmon

Early surgery was more strongly favored when there was no abscess at

time of presentation

Decreased antibiotic duration length of stay and total charges for

abscess and no abscess groups

You are seeing a 7 year old with constipation A

fecalith was seen on abdominal X ray during recent

ED visit How would you counsel the family

A They should be referred to surgery for appendectomy

B Their child is at higher risk for appendicitis so they should be aware of this in case he develops symptoms

C It is uncertain whether this child is at higher risk for appendicitis

D A course of Miralax may help wash out the fecalith

Take home points

Uncomplicated appendicitis

ndash Lap appy is still standard of care

ndash Non-operative management may be an acceptable option but not enough is known about long-term risk of recurrent appendicitis

Complicated appendicitis

ndash Can be managed with immediate operation delayed appendectomy or no appendectomy

ndash Immediate operation is probably more efficient and less stressful for patients and parents

Thank you very much

Questions

Page 21: New approaches for evaluation and treatment of appendicitis · Ultrasound or CT scan showing appendicitis with an appendix ≤ 1.1 centimeter thick and no evidence of abscess or fecalith

CT scan

MRI

Summary of approach to imaging

Ultrasound is good if you have a high pretest probability good radiologist thin patient

MRI is good if you are in a center that uses it routinely to evaluate children with appendicitis

CT scan is best if above criteria are not met

Alternative to imaging

ndash OBSERVATION

Once diagnosis of appendicitis has been establishedhellip

Non-ruptured

ndash Short duration of symptoms no suggestion of rupture on imaging

ndash Start antibiotics

ndash Perform appendectomy

Ruptured

ndash Have radiology evaluate for drainable fluid collection

ndash Antibiotics

ndash Operation if above management fails

Once diagnosis of appendicitis has been establishedhellip

Non-ruptured

ndash Short duration of symptoms no suggestion of rupture on imaging

ndash Start antibiotics

ndash Perform appendectomy

Ruptured

ndash Have radiology evaluate for drainable fluid collection

ndash Antibiotics

ndash Operation if above management fails

When should appendectomy be performed in a

patient with non-ruptured acute appendicitis

A Immediately after diagnosis

B Within first 24 hours of hospital stay once appropriate antibiotics and IVF resuscitation have been provided

C After completion of a one-week course of antibiotics

D It is reasonable to treat these patients with antibiotics alone and not perform an appendectomy

Is the appendix a ticking time bomb

Traditional teaching appendicitis is an emergency and should be treated with surgery immediately upon diagnosis

Several studies have now shown that delays of 12 hours up to 48 hours do not increase risk of perforation

Current standard admit IVF resuscitation antibiotics appendectomy during daylight hours

Almstroumlm M1 Svensson JF Patkova B Svenningsson A Wester TAnn Surg 2016 Mar 8 In-hospital

Surgical Delay Does Not Increase the Risk for Perforated Appendicitis in Children A Single-center

Retrospective Cohort Study

Port placement

Lap appy with Endo-loops

Lap appy with stapler

Open appendectomy

Appendix with fecalith

Laparoscopic vs open appendectomy

Laparoscopy is standard of care

Lower complication rate

Less scar

Less pain

Ability to evaluate other intra-abdominal organs if the appendix looks normal

ndash Gallbladder

ndash Ovaries

ndash Inguinal canal

Sauerland S Jaschinski T Neugebauer EA Laparoscopic versus open surgery for

suspected appendicitis Cochrane Database Syst Rev 2010 Oct 6(10)

Post-op course

Most patients can go home on the day of surgery

Recovery is usually quick

Back to school within a week sports within 2 weeks

Very low risk surgery with good outcomes

Risk of infection is about 5 at port sites or in abdomen

You see a 7 year old male in your office with 1-day history of abdominal pain

now localized to RLQ with focal guarding and poor appetite You send him for

ultrasound and it shows appendicitis His mother has heard about some new

research and wants to know if you would recommend just treating with

antibiotics rather than surgery How would you counsel this mom

A This data is preliminary and surgery is still standard of

care

B This may be a reasonable option for her son and she

should discuss it with her surgeon

C If it were your son you wouldnrsquot let him have surgery

D Other

Do all patients with appendicitis require surgery

Pilot study at Nationwide Childrenrsquos Hospital

102 patients

ndash 7 to 17 years of age

ndash Uncomplicated appendicitis defined by

Abdominal pain le 48 hours

White blood cell count le 18000

Ultrasound or CT scan showing appendicitis with an appendix le 11 centimeter thick and no

evidence of abscess or fecalith

Patients and families chose to have appendectomy or antibiotics alone

Non-operative management at least 24 hours of in-hospital observation and IV antibiotics until symptoms improved followed by completion of 10 days of treatment with antibiotics by mouth

Minneci PC Mahida JB et al The effectiveness of patient choice in non-operative

versus surgical management of uncomplicated acute appendicitis JAMA Surgery

2015 Dec 16

Results of non-operative management

65 families chose appendectomy 37 families chose non-operative management

Success rate of non-operative management (defined as not undergoing an appendectomy) 89 at 30 days 76 at 1 year

The 24 who failed did NOT have a higher rate of ruptured appendicitis compared to the patients who had immediate appendectomy

1 year follow-up the children managed non-operatively compared with the surgery group had fewer disability days (8 vs 21 days) lower appendicitis-related health care costs (median $4219 vs $5029) and no difference in health-related quality of life

Non-Operative Treatment of Appendicitis Rationale

Appendectomy is invasive

Children may miss up to two weeks of schoolactivities

Caregivers miss work

Postop complications after appendectomy for uncomplicated appendicitis 5-10

Serious complications (reoperations or readmissions) 1-7

Adult data suggest one-year success rates of 63-85 no difference in rates of complicated appendicitis

bull Meta-analysis of 10 articles reporting 413 children receiving non

operative treatment (NOT) for appendicitis ndash all published in past 10

years

bull 5 comparative

bull 1 RCT

bull 4 case series of NOT

Interpreting meta-analyses Forest plot

Favors

antibiotics

Favors

appendectomy

Overall success of NOT

97 during initial episode

Incidence of recurrent appendicitis during follow-up period (range 2-51

months)

Long term efficacy of NOT (no appendectomy at end of follow-up

period)

LOS shortened by about 05 days in those undergoing appendectomy

compared to NOT

Increased rate of complications for appendectomy compared to NOT

Authorsrsquo Conclusions

ldquoCurrent data suggest that NOT is safe It appears effective as initial treatment in 97 of children with AUA and the rate of recurrent appendicitis is 14rdquo

The study highlights the lack of robust evidence comparing NOT with appendectomy in children

Confirms a position of equipoise between treatment approaches

We recommend that NOT of children with AUA be reserved for those participating in carefully designed research studies

Take-home point

Lap appy is still standard of care for uncomplicated appendicitis

Once diagnosis of appendicitis has been establishedhellip

Non-ruptured

ndash Short duration of symptoms no suggestion of rupture on imaging

ndash Start antibiotics

ndash Perform appendectomy

Ruptured

ndash Have radiology evaluate for drainable fluid collection

ndash Antibiotics

ndash Operation in acute setting if above management fails

ndash Interval appendectomy 6-8 weeks laterhellip

ndash ALTERNATIVELY Just take out the appendix

CT ruptured appendix with abscess

After placement of percutaneous drain

How to counsel families of children with ruptured appendicitis

If kids tolerate a diet pain resolves fever resolves oral antibiotics and home

Plan interval appy 6-8 weeks later

Non-operative management may not work and surgery may be needed (Failure rate 20)

At home they should watch for signs of persistentrecurrent appendicitis

High-anxiety time for patients and families

A 10-year-old boy comes to see you in the office after recent

hospitalization for perforated appendicitis He has 3 more days of

antibiotics left His mom is worried because appetite is poor and his

energy level is low On exam he has diffuse lower abdominal

tenderness You recommend

A Extending course of oral antibiotics

B CT scan to evaluate for persistent or

recurrent appendicitis

C CT scan to evaluate for intra-abdominal

abscess

D Follow-up with surgeon

Is interval appendectomy necessary

In adults many surgeons do not do this operation

In kids data are limited

ndash 2-year follow-up of 96 patients

ndash Perforated appendicitis treated non-operatively with antibiotics

ndash 6 became worse 41 had interval appendectomy

ndash 49 received no further treatment

ndash 57 no recurrence

ndash 43 had recurrence within one month to 2 years

ndash Presence of appendicolith 72 rate of recurrence vs 26 in those without appendicolith

Ein SH1 Langer JC Daneman A Nonoperative management of pediatric ruptured appendix

with inflammatory mass or abscess presence of an appendicolith predicts recurrent

appendicitis J Pediatr Surg 2005 Oct40(10)1612-5

Approach to interval appendectomy

More strongly recommended if fecalith present

Observation is a reasonable option

Best choice for an individual patient depends on their anxiety and parental anxiety

Immediate operation for ruptured appendicitis perhaps a better

option

Meta-analysis operative vs non-operative management of pediatric

ruptured appendicitis

2 RCTs identified

Total of 171 pediatric patients

Compared early vs interval appendectomy

Early appendectomy reduced incidence of adverse event

Ruptured appendicitis may have abscess or phlegmon

Early surgery was more strongly favored when there was no abscess at

time of presentation

Decreased antibiotic duration length of stay and total charges for

abscess and no abscess groups

You are seeing a 7 year old with constipation A

fecalith was seen on abdominal X ray during recent

ED visit How would you counsel the family

A They should be referred to surgery for appendectomy

B Their child is at higher risk for appendicitis so they should be aware of this in case he develops symptoms

C It is uncertain whether this child is at higher risk for appendicitis

D A course of Miralax may help wash out the fecalith

Take home points

Uncomplicated appendicitis

ndash Lap appy is still standard of care

ndash Non-operative management may be an acceptable option but not enough is known about long-term risk of recurrent appendicitis

Complicated appendicitis

ndash Can be managed with immediate operation delayed appendectomy or no appendectomy

ndash Immediate operation is probably more efficient and less stressful for patients and parents

Thank you very much

Questions

Page 22: New approaches for evaluation and treatment of appendicitis · Ultrasound or CT scan showing appendicitis with an appendix ≤ 1.1 centimeter thick and no evidence of abscess or fecalith

MRI

Summary of approach to imaging

Ultrasound is good if you have a high pretest probability good radiologist thin patient

MRI is good if you are in a center that uses it routinely to evaluate children with appendicitis

CT scan is best if above criteria are not met

Alternative to imaging

ndash OBSERVATION

Once diagnosis of appendicitis has been establishedhellip

Non-ruptured

ndash Short duration of symptoms no suggestion of rupture on imaging

ndash Start antibiotics

ndash Perform appendectomy

Ruptured

ndash Have radiology evaluate for drainable fluid collection

ndash Antibiotics

ndash Operation if above management fails

Once diagnosis of appendicitis has been establishedhellip

Non-ruptured

ndash Short duration of symptoms no suggestion of rupture on imaging

ndash Start antibiotics

ndash Perform appendectomy

Ruptured

ndash Have radiology evaluate for drainable fluid collection

ndash Antibiotics

ndash Operation if above management fails

When should appendectomy be performed in a

patient with non-ruptured acute appendicitis

A Immediately after diagnosis

B Within first 24 hours of hospital stay once appropriate antibiotics and IVF resuscitation have been provided

C After completion of a one-week course of antibiotics

D It is reasonable to treat these patients with antibiotics alone and not perform an appendectomy

Is the appendix a ticking time bomb

Traditional teaching appendicitis is an emergency and should be treated with surgery immediately upon diagnosis

Several studies have now shown that delays of 12 hours up to 48 hours do not increase risk of perforation

Current standard admit IVF resuscitation antibiotics appendectomy during daylight hours

Almstroumlm M1 Svensson JF Patkova B Svenningsson A Wester TAnn Surg 2016 Mar 8 In-hospital

Surgical Delay Does Not Increase the Risk for Perforated Appendicitis in Children A Single-center

Retrospective Cohort Study

Port placement

Lap appy with Endo-loops

Lap appy with stapler

Open appendectomy

Appendix with fecalith

Laparoscopic vs open appendectomy

Laparoscopy is standard of care

Lower complication rate

Less scar

Less pain

Ability to evaluate other intra-abdominal organs if the appendix looks normal

ndash Gallbladder

ndash Ovaries

ndash Inguinal canal

Sauerland S Jaschinski T Neugebauer EA Laparoscopic versus open surgery for

suspected appendicitis Cochrane Database Syst Rev 2010 Oct 6(10)

Post-op course

Most patients can go home on the day of surgery

Recovery is usually quick

Back to school within a week sports within 2 weeks

Very low risk surgery with good outcomes

Risk of infection is about 5 at port sites or in abdomen

You see a 7 year old male in your office with 1-day history of abdominal pain

now localized to RLQ with focal guarding and poor appetite You send him for

ultrasound and it shows appendicitis His mother has heard about some new

research and wants to know if you would recommend just treating with

antibiotics rather than surgery How would you counsel this mom

A This data is preliminary and surgery is still standard of

care

B This may be a reasonable option for her son and she

should discuss it with her surgeon

C If it were your son you wouldnrsquot let him have surgery

D Other

Do all patients with appendicitis require surgery

Pilot study at Nationwide Childrenrsquos Hospital

102 patients

ndash 7 to 17 years of age

ndash Uncomplicated appendicitis defined by

Abdominal pain le 48 hours

White blood cell count le 18000

Ultrasound or CT scan showing appendicitis with an appendix le 11 centimeter thick and no

evidence of abscess or fecalith

Patients and families chose to have appendectomy or antibiotics alone

Non-operative management at least 24 hours of in-hospital observation and IV antibiotics until symptoms improved followed by completion of 10 days of treatment with antibiotics by mouth

Minneci PC Mahida JB et al The effectiveness of patient choice in non-operative

versus surgical management of uncomplicated acute appendicitis JAMA Surgery

2015 Dec 16

Results of non-operative management

65 families chose appendectomy 37 families chose non-operative management

Success rate of non-operative management (defined as not undergoing an appendectomy) 89 at 30 days 76 at 1 year

The 24 who failed did NOT have a higher rate of ruptured appendicitis compared to the patients who had immediate appendectomy

1 year follow-up the children managed non-operatively compared with the surgery group had fewer disability days (8 vs 21 days) lower appendicitis-related health care costs (median $4219 vs $5029) and no difference in health-related quality of life

Non-Operative Treatment of Appendicitis Rationale

Appendectomy is invasive

Children may miss up to two weeks of schoolactivities

Caregivers miss work

Postop complications after appendectomy for uncomplicated appendicitis 5-10

Serious complications (reoperations or readmissions) 1-7

Adult data suggest one-year success rates of 63-85 no difference in rates of complicated appendicitis

bull Meta-analysis of 10 articles reporting 413 children receiving non

operative treatment (NOT) for appendicitis ndash all published in past 10

years

bull 5 comparative

bull 1 RCT

bull 4 case series of NOT

Interpreting meta-analyses Forest plot

Favors

antibiotics

Favors

appendectomy

Overall success of NOT

97 during initial episode

Incidence of recurrent appendicitis during follow-up period (range 2-51

months)

Long term efficacy of NOT (no appendectomy at end of follow-up

period)

LOS shortened by about 05 days in those undergoing appendectomy

compared to NOT

Increased rate of complications for appendectomy compared to NOT

Authorsrsquo Conclusions

ldquoCurrent data suggest that NOT is safe It appears effective as initial treatment in 97 of children with AUA and the rate of recurrent appendicitis is 14rdquo

The study highlights the lack of robust evidence comparing NOT with appendectomy in children

Confirms a position of equipoise between treatment approaches

We recommend that NOT of children with AUA be reserved for those participating in carefully designed research studies

Take-home point

Lap appy is still standard of care for uncomplicated appendicitis

Once diagnosis of appendicitis has been establishedhellip

Non-ruptured

ndash Short duration of symptoms no suggestion of rupture on imaging

ndash Start antibiotics

ndash Perform appendectomy

Ruptured

ndash Have radiology evaluate for drainable fluid collection

ndash Antibiotics

ndash Operation in acute setting if above management fails

ndash Interval appendectomy 6-8 weeks laterhellip

ndash ALTERNATIVELY Just take out the appendix

CT ruptured appendix with abscess

After placement of percutaneous drain

How to counsel families of children with ruptured appendicitis

If kids tolerate a diet pain resolves fever resolves oral antibiotics and home

Plan interval appy 6-8 weeks later

Non-operative management may not work and surgery may be needed (Failure rate 20)

At home they should watch for signs of persistentrecurrent appendicitis

High-anxiety time for patients and families

A 10-year-old boy comes to see you in the office after recent

hospitalization for perforated appendicitis He has 3 more days of

antibiotics left His mom is worried because appetite is poor and his

energy level is low On exam he has diffuse lower abdominal

tenderness You recommend

A Extending course of oral antibiotics

B CT scan to evaluate for persistent or

recurrent appendicitis

C CT scan to evaluate for intra-abdominal

abscess

D Follow-up with surgeon

Is interval appendectomy necessary

In adults many surgeons do not do this operation

In kids data are limited

ndash 2-year follow-up of 96 patients

ndash Perforated appendicitis treated non-operatively with antibiotics

ndash 6 became worse 41 had interval appendectomy

ndash 49 received no further treatment

ndash 57 no recurrence

ndash 43 had recurrence within one month to 2 years

ndash Presence of appendicolith 72 rate of recurrence vs 26 in those without appendicolith

Ein SH1 Langer JC Daneman A Nonoperative management of pediatric ruptured appendix

with inflammatory mass or abscess presence of an appendicolith predicts recurrent

appendicitis J Pediatr Surg 2005 Oct40(10)1612-5

Approach to interval appendectomy

More strongly recommended if fecalith present

Observation is a reasonable option

Best choice for an individual patient depends on their anxiety and parental anxiety

Immediate operation for ruptured appendicitis perhaps a better

option

Meta-analysis operative vs non-operative management of pediatric

ruptured appendicitis

2 RCTs identified

Total of 171 pediatric patients

Compared early vs interval appendectomy

Early appendectomy reduced incidence of adverse event

Ruptured appendicitis may have abscess or phlegmon

Early surgery was more strongly favored when there was no abscess at

time of presentation

Decreased antibiotic duration length of stay and total charges for

abscess and no abscess groups

You are seeing a 7 year old with constipation A

fecalith was seen on abdominal X ray during recent

ED visit How would you counsel the family

A They should be referred to surgery for appendectomy

B Their child is at higher risk for appendicitis so they should be aware of this in case he develops symptoms

C It is uncertain whether this child is at higher risk for appendicitis

D A course of Miralax may help wash out the fecalith

Take home points

Uncomplicated appendicitis

ndash Lap appy is still standard of care

ndash Non-operative management may be an acceptable option but not enough is known about long-term risk of recurrent appendicitis

Complicated appendicitis

ndash Can be managed with immediate operation delayed appendectomy or no appendectomy

ndash Immediate operation is probably more efficient and less stressful for patients and parents

Thank you very much

Questions

Page 23: New approaches for evaluation and treatment of appendicitis · Ultrasound or CT scan showing appendicitis with an appendix ≤ 1.1 centimeter thick and no evidence of abscess or fecalith

Summary of approach to imaging

Ultrasound is good if you have a high pretest probability good radiologist thin patient

MRI is good if you are in a center that uses it routinely to evaluate children with appendicitis

CT scan is best if above criteria are not met

Alternative to imaging

ndash OBSERVATION

Once diagnosis of appendicitis has been establishedhellip

Non-ruptured

ndash Short duration of symptoms no suggestion of rupture on imaging

ndash Start antibiotics

ndash Perform appendectomy

Ruptured

ndash Have radiology evaluate for drainable fluid collection

ndash Antibiotics

ndash Operation if above management fails

Once diagnosis of appendicitis has been establishedhellip

Non-ruptured

ndash Short duration of symptoms no suggestion of rupture on imaging

ndash Start antibiotics

ndash Perform appendectomy

Ruptured

ndash Have radiology evaluate for drainable fluid collection

ndash Antibiotics

ndash Operation if above management fails

When should appendectomy be performed in a

patient with non-ruptured acute appendicitis

A Immediately after diagnosis

B Within first 24 hours of hospital stay once appropriate antibiotics and IVF resuscitation have been provided

C After completion of a one-week course of antibiotics

D It is reasonable to treat these patients with antibiotics alone and not perform an appendectomy

Is the appendix a ticking time bomb

Traditional teaching appendicitis is an emergency and should be treated with surgery immediately upon diagnosis

Several studies have now shown that delays of 12 hours up to 48 hours do not increase risk of perforation

Current standard admit IVF resuscitation antibiotics appendectomy during daylight hours

Almstroumlm M1 Svensson JF Patkova B Svenningsson A Wester TAnn Surg 2016 Mar 8 In-hospital

Surgical Delay Does Not Increase the Risk for Perforated Appendicitis in Children A Single-center

Retrospective Cohort Study

Port placement

Lap appy with Endo-loops

Lap appy with stapler

Open appendectomy

Appendix with fecalith

Laparoscopic vs open appendectomy

Laparoscopy is standard of care

Lower complication rate

Less scar

Less pain

Ability to evaluate other intra-abdominal organs if the appendix looks normal

ndash Gallbladder

ndash Ovaries

ndash Inguinal canal

Sauerland S Jaschinski T Neugebauer EA Laparoscopic versus open surgery for

suspected appendicitis Cochrane Database Syst Rev 2010 Oct 6(10)

Post-op course

Most patients can go home on the day of surgery

Recovery is usually quick

Back to school within a week sports within 2 weeks

Very low risk surgery with good outcomes

Risk of infection is about 5 at port sites or in abdomen

You see a 7 year old male in your office with 1-day history of abdominal pain

now localized to RLQ with focal guarding and poor appetite You send him for

ultrasound and it shows appendicitis His mother has heard about some new

research and wants to know if you would recommend just treating with

antibiotics rather than surgery How would you counsel this mom

A This data is preliminary and surgery is still standard of

care

B This may be a reasonable option for her son and she

should discuss it with her surgeon

C If it were your son you wouldnrsquot let him have surgery

D Other

Do all patients with appendicitis require surgery

Pilot study at Nationwide Childrenrsquos Hospital

102 patients

ndash 7 to 17 years of age

ndash Uncomplicated appendicitis defined by

Abdominal pain le 48 hours

White blood cell count le 18000

Ultrasound or CT scan showing appendicitis with an appendix le 11 centimeter thick and no

evidence of abscess or fecalith

Patients and families chose to have appendectomy or antibiotics alone

Non-operative management at least 24 hours of in-hospital observation and IV antibiotics until symptoms improved followed by completion of 10 days of treatment with antibiotics by mouth

Minneci PC Mahida JB et al The effectiveness of patient choice in non-operative

versus surgical management of uncomplicated acute appendicitis JAMA Surgery

2015 Dec 16

Results of non-operative management

65 families chose appendectomy 37 families chose non-operative management

Success rate of non-operative management (defined as not undergoing an appendectomy) 89 at 30 days 76 at 1 year

The 24 who failed did NOT have a higher rate of ruptured appendicitis compared to the patients who had immediate appendectomy

1 year follow-up the children managed non-operatively compared with the surgery group had fewer disability days (8 vs 21 days) lower appendicitis-related health care costs (median $4219 vs $5029) and no difference in health-related quality of life

Non-Operative Treatment of Appendicitis Rationale

Appendectomy is invasive

Children may miss up to two weeks of schoolactivities

Caregivers miss work

Postop complications after appendectomy for uncomplicated appendicitis 5-10

Serious complications (reoperations or readmissions) 1-7

Adult data suggest one-year success rates of 63-85 no difference in rates of complicated appendicitis

bull Meta-analysis of 10 articles reporting 413 children receiving non

operative treatment (NOT) for appendicitis ndash all published in past 10

years

bull 5 comparative

bull 1 RCT

bull 4 case series of NOT

Interpreting meta-analyses Forest plot

Favors

antibiotics

Favors

appendectomy

Overall success of NOT

97 during initial episode

Incidence of recurrent appendicitis during follow-up period (range 2-51

months)

Long term efficacy of NOT (no appendectomy at end of follow-up

period)

LOS shortened by about 05 days in those undergoing appendectomy

compared to NOT

Increased rate of complications for appendectomy compared to NOT

Authorsrsquo Conclusions

ldquoCurrent data suggest that NOT is safe It appears effective as initial treatment in 97 of children with AUA and the rate of recurrent appendicitis is 14rdquo

The study highlights the lack of robust evidence comparing NOT with appendectomy in children

Confirms a position of equipoise between treatment approaches

We recommend that NOT of children with AUA be reserved for those participating in carefully designed research studies

Take-home point

Lap appy is still standard of care for uncomplicated appendicitis

Once diagnosis of appendicitis has been establishedhellip

Non-ruptured

ndash Short duration of symptoms no suggestion of rupture on imaging

ndash Start antibiotics

ndash Perform appendectomy

Ruptured

ndash Have radiology evaluate for drainable fluid collection

ndash Antibiotics

ndash Operation in acute setting if above management fails

ndash Interval appendectomy 6-8 weeks laterhellip

ndash ALTERNATIVELY Just take out the appendix

CT ruptured appendix with abscess

After placement of percutaneous drain

How to counsel families of children with ruptured appendicitis

If kids tolerate a diet pain resolves fever resolves oral antibiotics and home

Plan interval appy 6-8 weeks later

Non-operative management may not work and surgery may be needed (Failure rate 20)

At home they should watch for signs of persistentrecurrent appendicitis

High-anxiety time for patients and families

A 10-year-old boy comes to see you in the office after recent

hospitalization for perforated appendicitis He has 3 more days of

antibiotics left His mom is worried because appetite is poor and his

energy level is low On exam he has diffuse lower abdominal

tenderness You recommend

A Extending course of oral antibiotics

B CT scan to evaluate for persistent or

recurrent appendicitis

C CT scan to evaluate for intra-abdominal

abscess

D Follow-up with surgeon

Is interval appendectomy necessary

In adults many surgeons do not do this operation

In kids data are limited

ndash 2-year follow-up of 96 patients

ndash Perforated appendicitis treated non-operatively with antibiotics

ndash 6 became worse 41 had interval appendectomy

ndash 49 received no further treatment

ndash 57 no recurrence

ndash 43 had recurrence within one month to 2 years

ndash Presence of appendicolith 72 rate of recurrence vs 26 in those without appendicolith

Ein SH1 Langer JC Daneman A Nonoperative management of pediatric ruptured appendix

with inflammatory mass or abscess presence of an appendicolith predicts recurrent

appendicitis J Pediatr Surg 2005 Oct40(10)1612-5

Approach to interval appendectomy

More strongly recommended if fecalith present

Observation is a reasonable option

Best choice for an individual patient depends on their anxiety and parental anxiety

Immediate operation for ruptured appendicitis perhaps a better

option

Meta-analysis operative vs non-operative management of pediatric

ruptured appendicitis

2 RCTs identified

Total of 171 pediatric patients

Compared early vs interval appendectomy

Early appendectomy reduced incidence of adverse event

Ruptured appendicitis may have abscess or phlegmon

Early surgery was more strongly favored when there was no abscess at

time of presentation

Decreased antibiotic duration length of stay and total charges for

abscess and no abscess groups

You are seeing a 7 year old with constipation A

fecalith was seen on abdominal X ray during recent

ED visit How would you counsel the family

A They should be referred to surgery for appendectomy

B Their child is at higher risk for appendicitis so they should be aware of this in case he develops symptoms

C It is uncertain whether this child is at higher risk for appendicitis

D A course of Miralax may help wash out the fecalith

Take home points

Uncomplicated appendicitis

ndash Lap appy is still standard of care

ndash Non-operative management may be an acceptable option but not enough is known about long-term risk of recurrent appendicitis

Complicated appendicitis

ndash Can be managed with immediate operation delayed appendectomy or no appendectomy

ndash Immediate operation is probably more efficient and less stressful for patients and parents

Thank you very much

Questions

Page 24: New approaches for evaluation and treatment of appendicitis · Ultrasound or CT scan showing appendicitis with an appendix ≤ 1.1 centimeter thick and no evidence of abscess or fecalith

Once diagnosis of appendicitis has been establishedhellip

Non-ruptured

ndash Short duration of symptoms no suggestion of rupture on imaging

ndash Start antibiotics

ndash Perform appendectomy

Ruptured

ndash Have radiology evaluate for drainable fluid collection

ndash Antibiotics

ndash Operation if above management fails

Once diagnosis of appendicitis has been establishedhellip

Non-ruptured

ndash Short duration of symptoms no suggestion of rupture on imaging

ndash Start antibiotics

ndash Perform appendectomy

Ruptured

ndash Have radiology evaluate for drainable fluid collection

ndash Antibiotics

ndash Operation if above management fails

When should appendectomy be performed in a

patient with non-ruptured acute appendicitis

A Immediately after diagnosis

B Within first 24 hours of hospital stay once appropriate antibiotics and IVF resuscitation have been provided

C After completion of a one-week course of antibiotics

D It is reasonable to treat these patients with antibiotics alone and not perform an appendectomy

Is the appendix a ticking time bomb

Traditional teaching appendicitis is an emergency and should be treated with surgery immediately upon diagnosis

Several studies have now shown that delays of 12 hours up to 48 hours do not increase risk of perforation

Current standard admit IVF resuscitation antibiotics appendectomy during daylight hours

Almstroumlm M1 Svensson JF Patkova B Svenningsson A Wester TAnn Surg 2016 Mar 8 In-hospital

Surgical Delay Does Not Increase the Risk for Perforated Appendicitis in Children A Single-center

Retrospective Cohort Study

Port placement

Lap appy with Endo-loops

Lap appy with stapler

Open appendectomy

Appendix with fecalith

Laparoscopic vs open appendectomy

Laparoscopy is standard of care

Lower complication rate

Less scar

Less pain

Ability to evaluate other intra-abdominal organs if the appendix looks normal

ndash Gallbladder

ndash Ovaries

ndash Inguinal canal

Sauerland S Jaschinski T Neugebauer EA Laparoscopic versus open surgery for

suspected appendicitis Cochrane Database Syst Rev 2010 Oct 6(10)

Post-op course

Most patients can go home on the day of surgery

Recovery is usually quick

Back to school within a week sports within 2 weeks

Very low risk surgery with good outcomes

Risk of infection is about 5 at port sites or in abdomen

You see a 7 year old male in your office with 1-day history of abdominal pain

now localized to RLQ with focal guarding and poor appetite You send him for

ultrasound and it shows appendicitis His mother has heard about some new

research and wants to know if you would recommend just treating with

antibiotics rather than surgery How would you counsel this mom

A This data is preliminary and surgery is still standard of

care

B This may be a reasonable option for her son and she

should discuss it with her surgeon

C If it were your son you wouldnrsquot let him have surgery

D Other

Do all patients with appendicitis require surgery

Pilot study at Nationwide Childrenrsquos Hospital

102 patients

ndash 7 to 17 years of age

ndash Uncomplicated appendicitis defined by

Abdominal pain le 48 hours

White blood cell count le 18000

Ultrasound or CT scan showing appendicitis with an appendix le 11 centimeter thick and no

evidence of abscess or fecalith

Patients and families chose to have appendectomy or antibiotics alone

Non-operative management at least 24 hours of in-hospital observation and IV antibiotics until symptoms improved followed by completion of 10 days of treatment with antibiotics by mouth

Minneci PC Mahida JB et al The effectiveness of patient choice in non-operative

versus surgical management of uncomplicated acute appendicitis JAMA Surgery

2015 Dec 16

Results of non-operative management

65 families chose appendectomy 37 families chose non-operative management

Success rate of non-operative management (defined as not undergoing an appendectomy) 89 at 30 days 76 at 1 year

The 24 who failed did NOT have a higher rate of ruptured appendicitis compared to the patients who had immediate appendectomy

1 year follow-up the children managed non-operatively compared with the surgery group had fewer disability days (8 vs 21 days) lower appendicitis-related health care costs (median $4219 vs $5029) and no difference in health-related quality of life

Non-Operative Treatment of Appendicitis Rationale

Appendectomy is invasive

Children may miss up to two weeks of schoolactivities

Caregivers miss work

Postop complications after appendectomy for uncomplicated appendicitis 5-10

Serious complications (reoperations or readmissions) 1-7

Adult data suggest one-year success rates of 63-85 no difference in rates of complicated appendicitis

bull Meta-analysis of 10 articles reporting 413 children receiving non

operative treatment (NOT) for appendicitis ndash all published in past 10

years

bull 5 comparative

bull 1 RCT

bull 4 case series of NOT

Interpreting meta-analyses Forest plot

Favors

antibiotics

Favors

appendectomy

Overall success of NOT

97 during initial episode

Incidence of recurrent appendicitis during follow-up period (range 2-51

months)

Long term efficacy of NOT (no appendectomy at end of follow-up

period)

LOS shortened by about 05 days in those undergoing appendectomy

compared to NOT

Increased rate of complications for appendectomy compared to NOT

Authorsrsquo Conclusions

ldquoCurrent data suggest that NOT is safe It appears effective as initial treatment in 97 of children with AUA and the rate of recurrent appendicitis is 14rdquo

The study highlights the lack of robust evidence comparing NOT with appendectomy in children

Confirms a position of equipoise between treatment approaches

We recommend that NOT of children with AUA be reserved for those participating in carefully designed research studies

Take-home point

Lap appy is still standard of care for uncomplicated appendicitis

Once diagnosis of appendicitis has been establishedhellip

Non-ruptured

ndash Short duration of symptoms no suggestion of rupture on imaging

ndash Start antibiotics

ndash Perform appendectomy

Ruptured

ndash Have radiology evaluate for drainable fluid collection

ndash Antibiotics

ndash Operation in acute setting if above management fails

ndash Interval appendectomy 6-8 weeks laterhellip

ndash ALTERNATIVELY Just take out the appendix

CT ruptured appendix with abscess

After placement of percutaneous drain

How to counsel families of children with ruptured appendicitis

If kids tolerate a diet pain resolves fever resolves oral antibiotics and home

Plan interval appy 6-8 weeks later

Non-operative management may not work and surgery may be needed (Failure rate 20)

At home they should watch for signs of persistentrecurrent appendicitis

High-anxiety time for patients and families

A 10-year-old boy comes to see you in the office after recent

hospitalization for perforated appendicitis He has 3 more days of

antibiotics left His mom is worried because appetite is poor and his

energy level is low On exam he has diffuse lower abdominal

tenderness You recommend

A Extending course of oral antibiotics

B CT scan to evaluate for persistent or

recurrent appendicitis

C CT scan to evaluate for intra-abdominal

abscess

D Follow-up with surgeon

Is interval appendectomy necessary

In adults many surgeons do not do this operation

In kids data are limited

ndash 2-year follow-up of 96 patients

ndash Perforated appendicitis treated non-operatively with antibiotics

ndash 6 became worse 41 had interval appendectomy

ndash 49 received no further treatment

ndash 57 no recurrence

ndash 43 had recurrence within one month to 2 years

ndash Presence of appendicolith 72 rate of recurrence vs 26 in those without appendicolith

Ein SH1 Langer JC Daneman A Nonoperative management of pediatric ruptured appendix

with inflammatory mass or abscess presence of an appendicolith predicts recurrent

appendicitis J Pediatr Surg 2005 Oct40(10)1612-5

Approach to interval appendectomy

More strongly recommended if fecalith present

Observation is a reasonable option

Best choice for an individual patient depends on their anxiety and parental anxiety

Immediate operation for ruptured appendicitis perhaps a better

option

Meta-analysis operative vs non-operative management of pediatric

ruptured appendicitis

2 RCTs identified

Total of 171 pediatric patients

Compared early vs interval appendectomy

Early appendectomy reduced incidence of adverse event

Ruptured appendicitis may have abscess or phlegmon

Early surgery was more strongly favored when there was no abscess at

time of presentation

Decreased antibiotic duration length of stay and total charges for

abscess and no abscess groups

You are seeing a 7 year old with constipation A

fecalith was seen on abdominal X ray during recent

ED visit How would you counsel the family

A They should be referred to surgery for appendectomy

B Their child is at higher risk for appendicitis so they should be aware of this in case he develops symptoms

C It is uncertain whether this child is at higher risk for appendicitis

D A course of Miralax may help wash out the fecalith

Take home points

Uncomplicated appendicitis

ndash Lap appy is still standard of care

ndash Non-operative management may be an acceptable option but not enough is known about long-term risk of recurrent appendicitis

Complicated appendicitis

ndash Can be managed with immediate operation delayed appendectomy or no appendectomy

ndash Immediate operation is probably more efficient and less stressful for patients and parents

Thank you very much

Questions

Page 25: New approaches for evaluation and treatment of appendicitis · Ultrasound or CT scan showing appendicitis with an appendix ≤ 1.1 centimeter thick and no evidence of abscess or fecalith

Once diagnosis of appendicitis has been establishedhellip

Non-ruptured

ndash Short duration of symptoms no suggestion of rupture on imaging

ndash Start antibiotics

ndash Perform appendectomy

Ruptured

ndash Have radiology evaluate for drainable fluid collection

ndash Antibiotics

ndash Operation if above management fails

When should appendectomy be performed in a

patient with non-ruptured acute appendicitis

A Immediately after diagnosis

B Within first 24 hours of hospital stay once appropriate antibiotics and IVF resuscitation have been provided

C After completion of a one-week course of antibiotics

D It is reasonable to treat these patients with antibiotics alone and not perform an appendectomy

Is the appendix a ticking time bomb

Traditional teaching appendicitis is an emergency and should be treated with surgery immediately upon diagnosis

Several studies have now shown that delays of 12 hours up to 48 hours do not increase risk of perforation

Current standard admit IVF resuscitation antibiotics appendectomy during daylight hours

Almstroumlm M1 Svensson JF Patkova B Svenningsson A Wester TAnn Surg 2016 Mar 8 In-hospital

Surgical Delay Does Not Increase the Risk for Perforated Appendicitis in Children A Single-center

Retrospective Cohort Study

Port placement

Lap appy with Endo-loops

Lap appy with stapler

Open appendectomy

Appendix with fecalith

Laparoscopic vs open appendectomy

Laparoscopy is standard of care

Lower complication rate

Less scar

Less pain

Ability to evaluate other intra-abdominal organs if the appendix looks normal

ndash Gallbladder

ndash Ovaries

ndash Inguinal canal

Sauerland S Jaschinski T Neugebauer EA Laparoscopic versus open surgery for

suspected appendicitis Cochrane Database Syst Rev 2010 Oct 6(10)

Post-op course

Most patients can go home on the day of surgery

Recovery is usually quick

Back to school within a week sports within 2 weeks

Very low risk surgery with good outcomes

Risk of infection is about 5 at port sites or in abdomen

You see a 7 year old male in your office with 1-day history of abdominal pain

now localized to RLQ with focal guarding and poor appetite You send him for

ultrasound and it shows appendicitis His mother has heard about some new

research and wants to know if you would recommend just treating with

antibiotics rather than surgery How would you counsel this mom

A This data is preliminary and surgery is still standard of

care

B This may be a reasonable option for her son and she

should discuss it with her surgeon

C If it were your son you wouldnrsquot let him have surgery

D Other

Do all patients with appendicitis require surgery

Pilot study at Nationwide Childrenrsquos Hospital

102 patients

ndash 7 to 17 years of age

ndash Uncomplicated appendicitis defined by

Abdominal pain le 48 hours

White blood cell count le 18000

Ultrasound or CT scan showing appendicitis with an appendix le 11 centimeter thick and no

evidence of abscess or fecalith

Patients and families chose to have appendectomy or antibiotics alone

Non-operative management at least 24 hours of in-hospital observation and IV antibiotics until symptoms improved followed by completion of 10 days of treatment with antibiotics by mouth

Minneci PC Mahida JB et al The effectiveness of patient choice in non-operative

versus surgical management of uncomplicated acute appendicitis JAMA Surgery

2015 Dec 16

Results of non-operative management

65 families chose appendectomy 37 families chose non-operative management

Success rate of non-operative management (defined as not undergoing an appendectomy) 89 at 30 days 76 at 1 year

The 24 who failed did NOT have a higher rate of ruptured appendicitis compared to the patients who had immediate appendectomy

1 year follow-up the children managed non-operatively compared with the surgery group had fewer disability days (8 vs 21 days) lower appendicitis-related health care costs (median $4219 vs $5029) and no difference in health-related quality of life

Non-Operative Treatment of Appendicitis Rationale

Appendectomy is invasive

Children may miss up to two weeks of schoolactivities

Caregivers miss work

Postop complications after appendectomy for uncomplicated appendicitis 5-10

Serious complications (reoperations or readmissions) 1-7

Adult data suggest one-year success rates of 63-85 no difference in rates of complicated appendicitis

bull Meta-analysis of 10 articles reporting 413 children receiving non

operative treatment (NOT) for appendicitis ndash all published in past 10

years

bull 5 comparative

bull 1 RCT

bull 4 case series of NOT

Interpreting meta-analyses Forest plot

Favors

antibiotics

Favors

appendectomy

Overall success of NOT

97 during initial episode

Incidence of recurrent appendicitis during follow-up period (range 2-51

months)

Long term efficacy of NOT (no appendectomy at end of follow-up

period)

LOS shortened by about 05 days in those undergoing appendectomy

compared to NOT

Increased rate of complications for appendectomy compared to NOT

Authorsrsquo Conclusions

ldquoCurrent data suggest that NOT is safe It appears effective as initial treatment in 97 of children with AUA and the rate of recurrent appendicitis is 14rdquo

The study highlights the lack of robust evidence comparing NOT with appendectomy in children

Confirms a position of equipoise between treatment approaches

We recommend that NOT of children with AUA be reserved for those participating in carefully designed research studies

Take-home point

Lap appy is still standard of care for uncomplicated appendicitis

Once diagnosis of appendicitis has been establishedhellip

Non-ruptured

ndash Short duration of symptoms no suggestion of rupture on imaging

ndash Start antibiotics

ndash Perform appendectomy

Ruptured

ndash Have radiology evaluate for drainable fluid collection

ndash Antibiotics

ndash Operation in acute setting if above management fails

ndash Interval appendectomy 6-8 weeks laterhellip

ndash ALTERNATIVELY Just take out the appendix

CT ruptured appendix with abscess

After placement of percutaneous drain

How to counsel families of children with ruptured appendicitis

If kids tolerate a diet pain resolves fever resolves oral antibiotics and home

Plan interval appy 6-8 weeks later

Non-operative management may not work and surgery may be needed (Failure rate 20)

At home they should watch for signs of persistentrecurrent appendicitis

High-anxiety time for patients and families

A 10-year-old boy comes to see you in the office after recent

hospitalization for perforated appendicitis He has 3 more days of

antibiotics left His mom is worried because appetite is poor and his

energy level is low On exam he has diffuse lower abdominal

tenderness You recommend

A Extending course of oral antibiotics

B CT scan to evaluate for persistent or

recurrent appendicitis

C CT scan to evaluate for intra-abdominal

abscess

D Follow-up with surgeon

Is interval appendectomy necessary

In adults many surgeons do not do this operation

In kids data are limited

ndash 2-year follow-up of 96 patients

ndash Perforated appendicitis treated non-operatively with antibiotics

ndash 6 became worse 41 had interval appendectomy

ndash 49 received no further treatment

ndash 57 no recurrence

ndash 43 had recurrence within one month to 2 years

ndash Presence of appendicolith 72 rate of recurrence vs 26 in those without appendicolith

Ein SH1 Langer JC Daneman A Nonoperative management of pediatric ruptured appendix

with inflammatory mass or abscess presence of an appendicolith predicts recurrent

appendicitis J Pediatr Surg 2005 Oct40(10)1612-5

Approach to interval appendectomy

More strongly recommended if fecalith present

Observation is a reasonable option

Best choice for an individual patient depends on their anxiety and parental anxiety

Immediate operation for ruptured appendicitis perhaps a better

option

Meta-analysis operative vs non-operative management of pediatric

ruptured appendicitis

2 RCTs identified

Total of 171 pediatric patients

Compared early vs interval appendectomy

Early appendectomy reduced incidence of adverse event

Ruptured appendicitis may have abscess or phlegmon

Early surgery was more strongly favored when there was no abscess at

time of presentation

Decreased antibiotic duration length of stay and total charges for

abscess and no abscess groups

You are seeing a 7 year old with constipation A

fecalith was seen on abdominal X ray during recent

ED visit How would you counsel the family

A They should be referred to surgery for appendectomy

B Their child is at higher risk for appendicitis so they should be aware of this in case he develops symptoms

C It is uncertain whether this child is at higher risk for appendicitis

D A course of Miralax may help wash out the fecalith

Take home points

Uncomplicated appendicitis

ndash Lap appy is still standard of care

ndash Non-operative management may be an acceptable option but not enough is known about long-term risk of recurrent appendicitis

Complicated appendicitis

ndash Can be managed with immediate operation delayed appendectomy or no appendectomy

ndash Immediate operation is probably more efficient and less stressful for patients and parents

Thank you very much

Questions

Page 26: New approaches for evaluation and treatment of appendicitis · Ultrasound or CT scan showing appendicitis with an appendix ≤ 1.1 centimeter thick and no evidence of abscess or fecalith

When should appendectomy be performed in a

patient with non-ruptured acute appendicitis

A Immediately after diagnosis

B Within first 24 hours of hospital stay once appropriate antibiotics and IVF resuscitation have been provided

C After completion of a one-week course of antibiotics

D It is reasonable to treat these patients with antibiotics alone and not perform an appendectomy

Is the appendix a ticking time bomb

Traditional teaching appendicitis is an emergency and should be treated with surgery immediately upon diagnosis

Several studies have now shown that delays of 12 hours up to 48 hours do not increase risk of perforation

Current standard admit IVF resuscitation antibiotics appendectomy during daylight hours

Almstroumlm M1 Svensson JF Patkova B Svenningsson A Wester TAnn Surg 2016 Mar 8 In-hospital

Surgical Delay Does Not Increase the Risk for Perforated Appendicitis in Children A Single-center

Retrospective Cohort Study

Port placement

Lap appy with Endo-loops

Lap appy with stapler

Open appendectomy

Appendix with fecalith

Laparoscopic vs open appendectomy

Laparoscopy is standard of care

Lower complication rate

Less scar

Less pain

Ability to evaluate other intra-abdominal organs if the appendix looks normal

ndash Gallbladder

ndash Ovaries

ndash Inguinal canal

Sauerland S Jaschinski T Neugebauer EA Laparoscopic versus open surgery for

suspected appendicitis Cochrane Database Syst Rev 2010 Oct 6(10)

Post-op course

Most patients can go home on the day of surgery

Recovery is usually quick

Back to school within a week sports within 2 weeks

Very low risk surgery with good outcomes

Risk of infection is about 5 at port sites or in abdomen

You see a 7 year old male in your office with 1-day history of abdominal pain

now localized to RLQ with focal guarding and poor appetite You send him for

ultrasound and it shows appendicitis His mother has heard about some new

research and wants to know if you would recommend just treating with

antibiotics rather than surgery How would you counsel this mom

A This data is preliminary and surgery is still standard of

care

B This may be a reasonable option for her son and she

should discuss it with her surgeon

C If it were your son you wouldnrsquot let him have surgery

D Other

Do all patients with appendicitis require surgery

Pilot study at Nationwide Childrenrsquos Hospital

102 patients

ndash 7 to 17 years of age

ndash Uncomplicated appendicitis defined by

Abdominal pain le 48 hours

White blood cell count le 18000

Ultrasound or CT scan showing appendicitis with an appendix le 11 centimeter thick and no

evidence of abscess or fecalith

Patients and families chose to have appendectomy or antibiotics alone

Non-operative management at least 24 hours of in-hospital observation and IV antibiotics until symptoms improved followed by completion of 10 days of treatment with antibiotics by mouth

Minneci PC Mahida JB et al The effectiveness of patient choice in non-operative

versus surgical management of uncomplicated acute appendicitis JAMA Surgery

2015 Dec 16

Results of non-operative management

65 families chose appendectomy 37 families chose non-operative management

Success rate of non-operative management (defined as not undergoing an appendectomy) 89 at 30 days 76 at 1 year

The 24 who failed did NOT have a higher rate of ruptured appendicitis compared to the patients who had immediate appendectomy

1 year follow-up the children managed non-operatively compared with the surgery group had fewer disability days (8 vs 21 days) lower appendicitis-related health care costs (median $4219 vs $5029) and no difference in health-related quality of life

Non-Operative Treatment of Appendicitis Rationale

Appendectomy is invasive

Children may miss up to two weeks of schoolactivities

Caregivers miss work

Postop complications after appendectomy for uncomplicated appendicitis 5-10

Serious complications (reoperations or readmissions) 1-7

Adult data suggest one-year success rates of 63-85 no difference in rates of complicated appendicitis

bull Meta-analysis of 10 articles reporting 413 children receiving non

operative treatment (NOT) for appendicitis ndash all published in past 10

years

bull 5 comparative

bull 1 RCT

bull 4 case series of NOT

Interpreting meta-analyses Forest plot

Favors

antibiotics

Favors

appendectomy

Overall success of NOT

97 during initial episode

Incidence of recurrent appendicitis during follow-up period (range 2-51

months)

Long term efficacy of NOT (no appendectomy at end of follow-up

period)

LOS shortened by about 05 days in those undergoing appendectomy

compared to NOT

Increased rate of complications for appendectomy compared to NOT

Authorsrsquo Conclusions

ldquoCurrent data suggest that NOT is safe It appears effective as initial treatment in 97 of children with AUA and the rate of recurrent appendicitis is 14rdquo

The study highlights the lack of robust evidence comparing NOT with appendectomy in children

Confirms a position of equipoise between treatment approaches

We recommend that NOT of children with AUA be reserved for those participating in carefully designed research studies

Take-home point

Lap appy is still standard of care for uncomplicated appendicitis

Once diagnosis of appendicitis has been establishedhellip

Non-ruptured

ndash Short duration of symptoms no suggestion of rupture on imaging

ndash Start antibiotics

ndash Perform appendectomy

Ruptured

ndash Have radiology evaluate for drainable fluid collection

ndash Antibiotics

ndash Operation in acute setting if above management fails

ndash Interval appendectomy 6-8 weeks laterhellip

ndash ALTERNATIVELY Just take out the appendix

CT ruptured appendix with abscess

After placement of percutaneous drain

How to counsel families of children with ruptured appendicitis

If kids tolerate a diet pain resolves fever resolves oral antibiotics and home

Plan interval appy 6-8 weeks later

Non-operative management may not work and surgery may be needed (Failure rate 20)

At home they should watch for signs of persistentrecurrent appendicitis

High-anxiety time for patients and families

A 10-year-old boy comes to see you in the office after recent

hospitalization for perforated appendicitis He has 3 more days of

antibiotics left His mom is worried because appetite is poor and his

energy level is low On exam he has diffuse lower abdominal

tenderness You recommend

A Extending course of oral antibiotics

B CT scan to evaluate for persistent or

recurrent appendicitis

C CT scan to evaluate for intra-abdominal

abscess

D Follow-up with surgeon

Is interval appendectomy necessary

In adults many surgeons do not do this operation

In kids data are limited

ndash 2-year follow-up of 96 patients

ndash Perforated appendicitis treated non-operatively with antibiotics

ndash 6 became worse 41 had interval appendectomy

ndash 49 received no further treatment

ndash 57 no recurrence

ndash 43 had recurrence within one month to 2 years

ndash Presence of appendicolith 72 rate of recurrence vs 26 in those without appendicolith

Ein SH1 Langer JC Daneman A Nonoperative management of pediatric ruptured appendix

with inflammatory mass or abscess presence of an appendicolith predicts recurrent

appendicitis J Pediatr Surg 2005 Oct40(10)1612-5

Approach to interval appendectomy

More strongly recommended if fecalith present

Observation is a reasonable option

Best choice for an individual patient depends on their anxiety and parental anxiety

Immediate operation for ruptured appendicitis perhaps a better

option

Meta-analysis operative vs non-operative management of pediatric

ruptured appendicitis

2 RCTs identified

Total of 171 pediatric patients

Compared early vs interval appendectomy

Early appendectomy reduced incidence of adverse event

Ruptured appendicitis may have abscess or phlegmon

Early surgery was more strongly favored when there was no abscess at

time of presentation

Decreased antibiotic duration length of stay and total charges for

abscess and no abscess groups

You are seeing a 7 year old with constipation A

fecalith was seen on abdominal X ray during recent

ED visit How would you counsel the family

A They should be referred to surgery for appendectomy

B Their child is at higher risk for appendicitis so they should be aware of this in case he develops symptoms

C It is uncertain whether this child is at higher risk for appendicitis

D A course of Miralax may help wash out the fecalith

Take home points

Uncomplicated appendicitis

ndash Lap appy is still standard of care

ndash Non-operative management may be an acceptable option but not enough is known about long-term risk of recurrent appendicitis

Complicated appendicitis

ndash Can be managed with immediate operation delayed appendectomy or no appendectomy

ndash Immediate operation is probably more efficient and less stressful for patients and parents

Thank you very much

Questions

Page 27: New approaches for evaluation and treatment of appendicitis · Ultrasound or CT scan showing appendicitis with an appendix ≤ 1.1 centimeter thick and no evidence of abscess or fecalith

Is the appendix a ticking time bomb

Traditional teaching appendicitis is an emergency and should be treated with surgery immediately upon diagnosis

Several studies have now shown that delays of 12 hours up to 48 hours do not increase risk of perforation

Current standard admit IVF resuscitation antibiotics appendectomy during daylight hours

Almstroumlm M1 Svensson JF Patkova B Svenningsson A Wester TAnn Surg 2016 Mar 8 In-hospital

Surgical Delay Does Not Increase the Risk for Perforated Appendicitis in Children A Single-center

Retrospective Cohort Study

Port placement

Lap appy with Endo-loops

Lap appy with stapler

Open appendectomy

Appendix with fecalith

Laparoscopic vs open appendectomy

Laparoscopy is standard of care

Lower complication rate

Less scar

Less pain

Ability to evaluate other intra-abdominal organs if the appendix looks normal

ndash Gallbladder

ndash Ovaries

ndash Inguinal canal

Sauerland S Jaschinski T Neugebauer EA Laparoscopic versus open surgery for

suspected appendicitis Cochrane Database Syst Rev 2010 Oct 6(10)

Post-op course

Most patients can go home on the day of surgery

Recovery is usually quick

Back to school within a week sports within 2 weeks

Very low risk surgery with good outcomes

Risk of infection is about 5 at port sites or in abdomen

You see a 7 year old male in your office with 1-day history of abdominal pain

now localized to RLQ with focal guarding and poor appetite You send him for

ultrasound and it shows appendicitis His mother has heard about some new

research and wants to know if you would recommend just treating with

antibiotics rather than surgery How would you counsel this mom

A This data is preliminary and surgery is still standard of

care

B This may be a reasonable option for her son and she

should discuss it with her surgeon

C If it were your son you wouldnrsquot let him have surgery

D Other

Do all patients with appendicitis require surgery

Pilot study at Nationwide Childrenrsquos Hospital

102 patients

ndash 7 to 17 years of age

ndash Uncomplicated appendicitis defined by

Abdominal pain le 48 hours

White blood cell count le 18000

Ultrasound or CT scan showing appendicitis with an appendix le 11 centimeter thick and no

evidence of abscess or fecalith

Patients and families chose to have appendectomy or antibiotics alone

Non-operative management at least 24 hours of in-hospital observation and IV antibiotics until symptoms improved followed by completion of 10 days of treatment with antibiotics by mouth

Minneci PC Mahida JB et al The effectiveness of patient choice in non-operative

versus surgical management of uncomplicated acute appendicitis JAMA Surgery

2015 Dec 16

Results of non-operative management

65 families chose appendectomy 37 families chose non-operative management

Success rate of non-operative management (defined as not undergoing an appendectomy) 89 at 30 days 76 at 1 year

The 24 who failed did NOT have a higher rate of ruptured appendicitis compared to the patients who had immediate appendectomy

1 year follow-up the children managed non-operatively compared with the surgery group had fewer disability days (8 vs 21 days) lower appendicitis-related health care costs (median $4219 vs $5029) and no difference in health-related quality of life

Non-Operative Treatment of Appendicitis Rationale

Appendectomy is invasive

Children may miss up to two weeks of schoolactivities

Caregivers miss work

Postop complications after appendectomy for uncomplicated appendicitis 5-10

Serious complications (reoperations or readmissions) 1-7

Adult data suggest one-year success rates of 63-85 no difference in rates of complicated appendicitis

bull Meta-analysis of 10 articles reporting 413 children receiving non

operative treatment (NOT) for appendicitis ndash all published in past 10

years

bull 5 comparative

bull 1 RCT

bull 4 case series of NOT

Interpreting meta-analyses Forest plot

Favors

antibiotics

Favors

appendectomy

Overall success of NOT

97 during initial episode

Incidence of recurrent appendicitis during follow-up period (range 2-51

months)

Long term efficacy of NOT (no appendectomy at end of follow-up

period)

LOS shortened by about 05 days in those undergoing appendectomy

compared to NOT

Increased rate of complications for appendectomy compared to NOT

Authorsrsquo Conclusions

ldquoCurrent data suggest that NOT is safe It appears effective as initial treatment in 97 of children with AUA and the rate of recurrent appendicitis is 14rdquo

The study highlights the lack of robust evidence comparing NOT with appendectomy in children

Confirms a position of equipoise between treatment approaches

We recommend that NOT of children with AUA be reserved for those participating in carefully designed research studies

Take-home point

Lap appy is still standard of care for uncomplicated appendicitis

Once diagnosis of appendicitis has been establishedhellip

Non-ruptured

ndash Short duration of symptoms no suggestion of rupture on imaging

ndash Start antibiotics

ndash Perform appendectomy

Ruptured

ndash Have radiology evaluate for drainable fluid collection

ndash Antibiotics

ndash Operation in acute setting if above management fails

ndash Interval appendectomy 6-8 weeks laterhellip

ndash ALTERNATIVELY Just take out the appendix

CT ruptured appendix with abscess

After placement of percutaneous drain

How to counsel families of children with ruptured appendicitis

If kids tolerate a diet pain resolves fever resolves oral antibiotics and home

Plan interval appy 6-8 weeks later

Non-operative management may not work and surgery may be needed (Failure rate 20)

At home they should watch for signs of persistentrecurrent appendicitis

High-anxiety time for patients and families

A 10-year-old boy comes to see you in the office after recent

hospitalization for perforated appendicitis He has 3 more days of

antibiotics left His mom is worried because appetite is poor and his

energy level is low On exam he has diffuse lower abdominal

tenderness You recommend

A Extending course of oral antibiotics

B CT scan to evaluate for persistent or

recurrent appendicitis

C CT scan to evaluate for intra-abdominal

abscess

D Follow-up with surgeon

Is interval appendectomy necessary

In adults many surgeons do not do this operation

In kids data are limited

ndash 2-year follow-up of 96 patients

ndash Perforated appendicitis treated non-operatively with antibiotics

ndash 6 became worse 41 had interval appendectomy

ndash 49 received no further treatment

ndash 57 no recurrence

ndash 43 had recurrence within one month to 2 years

ndash Presence of appendicolith 72 rate of recurrence vs 26 in those without appendicolith

Ein SH1 Langer JC Daneman A Nonoperative management of pediatric ruptured appendix

with inflammatory mass or abscess presence of an appendicolith predicts recurrent

appendicitis J Pediatr Surg 2005 Oct40(10)1612-5

Approach to interval appendectomy

More strongly recommended if fecalith present

Observation is a reasonable option

Best choice for an individual patient depends on their anxiety and parental anxiety

Immediate operation for ruptured appendicitis perhaps a better

option

Meta-analysis operative vs non-operative management of pediatric

ruptured appendicitis

2 RCTs identified

Total of 171 pediatric patients

Compared early vs interval appendectomy

Early appendectomy reduced incidence of adverse event

Ruptured appendicitis may have abscess or phlegmon

Early surgery was more strongly favored when there was no abscess at

time of presentation

Decreased antibiotic duration length of stay and total charges for

abscess and no abscess groups

You are seeing a 7 year old with constipation A

fecalith was seen on abdominal X ray during recent

ED visit How would you counsel the family

A They should be referred to surgery for appendectomy

B Their child is at higher risk for appendicitis so they should be aware of this in case he develops symptoms

C It is uncertain whether this child is at higher risk for appendicitis

D A course of Miralax may help wash out the fecalith

Take home points

Uncomplicated appendicitis

ndash Lap appy is still standard of care

ndash Non-operative management may be an acceptable option but not enough is known about long-term risk of recurrent appendicitis

Complicated appendicitis

ndash Can be managed with immediate operation delayed appendectomy or no appendectomy

ndash Immediate operation is probably more efficient and less stressful for patients and parents

Thank you very much

Questions

Page 28: New approaches for evaluation and treatment of appendicitis · Ultrasound or CT scan showing appendicitis with an appendix ≤ 1.1 centimeter thick and no evidence of abscess or fecalith

Port placement

Lap appy with Endo-loops

Lap appy with stapler

Open appendectomy

Appendix with fecalith

Laparoscopic vs open appendectomy

Laparoscopy is standard of care

Lower complication rate

Less scar

Less pain

Ability to evaluate other intra-abdominal organs if the appendix looks normal

ndash Gallbladder

ndash Ovaries

ndash Inguinal canal

Sauerland S Jaschinski T Neugebauer EA Laparoscopic versus open surgery for

suspected appendicitis Cochrane Database Syst Rev 2010 Oct 6(10)

Post-op course

Most patients can go home on the day of surgery

Recovery is usually quick

Back to school within a week sports within 2 weeks

Very low risk surgery with good outcomes

Risk of infection is about 5 at port sites or in abdomen

You see a 7 year old male in your office with 1-day history of abdominal pain

now localized to RLQ with focal guarding and poor appetite You send him for

ultrasound and it shows appendicitis His mother has heard about some new

research and wants to know if you would recommend just treating with

antibiotics rather than surgery How would you counsel this mom

A This data is preliminary and surgery is still standard of

care

B This may be a reasonable option for her son and she

should discuss it with her surgeon

C If it were your son you wouldnrsquot let him have surgery

D Other

Do all patients with appendicitis require surgery

Pilot study at Nationwide Childrenrsquos Hospital

102 patients

ndash 7 to 17 years of age

ndash Uncomplicated appendicitis defined by

Abdominal pain le 48 hours

White blood cell count le 18000

Ultrasound or CT scan showing appendicitis with an appendix le 11 centimeter thick and no

evidence of abscess or fecalith

Patients and families chose to have appendectomy or antibiotics alone

Non-operative management at least 24 hours of in-hospital observation and IV antibiotics until symptoms improved followed by completion of 10 days of treatment with antibiotics by mouth

Minneci PC Mahida JB et al The effectiveness of patient choice in non-operative

versus surgical management of uncomplicated acute appendicitis JAMA Surgery

2015 Dec 16

Results of non-operative management

65 families chose appendectomy 37 families chose non-operative management

Success rate of non-operative management (defined as not undergoing an appendectomy) 89 at 30 days 76 at 1 year

The 24 who failed did NOT have a higher rate of ruptured appendicitis compared to the patients who had immediate appendectomy

1 year follow-up the children managed non-operatively compared with the surgery group had fewer disability days (8 vs 21 days) lower appendicitis-related health care costs (median $4219 vs $5029) and no difference in health-related quality of life

Non-Operative Treatment of Appendicitis Rationale

Appendectomy is invasive

Children may miss up to two weeks of schoolactivities

Caregivers miss work

Postop complications after appendectomy for uncomplicated appendicitis 5-10

Serious complications (reoperations or readmissions) 1-7

Adult data suggest one-year success rates of 63-85 no difference in rates of complicated appendicitis

bull Meta-analysis of 10 articles reporting 413 children receiving non

operative treatment (NOT) for appendicitis ndash all published in past 10

years

bull 5 comparative

bull 1 RCT

bull 4 case series of NOT

Interpreting meta-analyses Forest plot

Favors

antibiotics

Favors

appendectomy

Overall success of NOT

97 during initial episode

Incidence of recurrent appendicitis during follow-up period (range 2-51

months)

Long term efficacy of NOT (no appendectomy at end of follow-up

period)

LOS shortened by about 05 days in those undergoing appendectomy

compared to NOT

Increased rate of complications for appendectomy compared to NOT

Authorsrsquo Conclusions

ldquoCurrent data suggest that NOT is safe It appears effective as initial treatment in 97 of children with AUA and the rate of recurrent appendicitis is 14rdquo

The study highlights the lack of robust evidence comparing NOT with appendectomy in children

Confirms a position of equipoise between treatment approaches

We recommend that NOT of children with AUA be reserved for those participating in carefully designed research studies

Take-home point

Lap appy is still standard of care for uncomplicated appendicitis

Once diagnosis of appendicitis has been establishedhellip

Non-ruptured

ndash Short duration of symptoms no suggestion of rupture on imaging

ndash Start antibiotics

ndash Perform appendectomy

Ruptured

ndash Have radiology evaluate for drainable fluid collection

ndash Antibiotics

ndash Operation in acute setting if above management fails

ndash Interval appendectomy 6-8 weeks laterhellip

ndash ALTERNATIVELY Just take out the appendix

CT ruptured appendix with abscess

After placement of percutaneous drain

How to counsel families of children with ruptured appendicitis

If kids tolerate a diet pain resolves fever resolves oral antibiotics and home

Plan interval appy 6-8 weeks later

Non-operative management may not work and surgery may be needed (Failure rate 20)

At home they should watch for signs of persistentrecurrent appendicitis

High-anxiety time for patients and families

A 10-year-old boy comes to see you in the office after recent

hospitalization for perforated appendicitis He has 3 more days of

antibiotics left His mom is worried because appetite is poor and his

energy level is low On exam he has diffuse lower abdominal

tenderness You recommend

A Extending course of oral antibiotics

B CT scan to evaluate for persistent or

recurrent appendicitis

C CT scan to evaluate for intra-abdominal

abscess

D Follow-up with surgeon

Is interval appendectomy necessary

In adults many surgeons do not do this operation

In kids data are limited

ndash 2-year follow-up of 96 patients

ndash Perforated appendicitis treated non-operatively with antibiotics

ndash 6 became worse 41 had interval appendectomy

ndash 49 received no further treatment

ndash 57 no recurrence

ndash 43 had recurrence within one month to 2 years

ndash Presence of appendicolith 72 rate of recurrence vs 26 in those without appendicolith

Ein SH1 Langer JC Daneman A Nonoperative management of pediatric ruptured appendix

with inflammatory mass or abscess presence of an appendicolith predicts recurrent

appendicitis J Pediatr Surg 2005 Oct40(10)1612-5

Approach to interval appendectomy

More strongly recommended if fecalith present

Observation is a reasonable option

Best choice for an individual patient depends on their anxiety and parental anxiety

Immediate operation for ruptured appendicitis perhaps a better

option

Meta-analysis operative vs non-operative management of pediatric

ruptured appendicitis

2 RCTs identified

Total of 171 pediatric patients

Compared early vs interval appendectomy

Early appendectomy reduced incidence of adverse event

Ruptured appendicitis may have abscess or phlegmon

Early surgery was more strongly favored when there was no abscess at

time of presentation

Decreased antibiotic duration length of stay and total charges for

abscess and no abscess groups

You are seeing a 7 year old with constipation A

fecalith was seen on abdominal X ray during recent

ED visit How would you counsel the family

A They should be referred to surgery for appendectomy

B Their child is at higher risk for appendicitis so they should be aware of this in case he develops symptoms

C It is uncertain whether this child is at higher risk for appendicitis

D A course of Miralax may help wash out the fecalith

Take home points

Uncomplicated appendicitis

ndash Lap appy is still standard of care

ndash Non-operative management may be an acceptable option but not enough is known about long-term risk of recurrent appendicitis

Complicated appendicitis

ndash Can be managed with immediate operation delayed appendectomy or no appendectomy

ndash Immediate operation is probably more efficient and less stressful for patients and parents

Thank you very much

Questions

Page 29: New approaches for evaluation and treatment of appendicitis · Ultrasound or CT scan showing appendicitis with an appendix ≤ 1.1 centimeter thick and no evidence of abscess or fecalith

Lap appy with Endo-loops

Lap appy with stapler

Open appendectomy

Appendix with fecalith

Laparoscopic vs open appendectomy

Laparoscopy is standard of care

Lower complication rate

Less scar

Less pain

Ability to evaluate other intra-abdominal organs if the appendix looks normal

ndash Gallbladder

ndash Ovaries

ndash Inguinal canal

Sauerland S Jaschinski T Neugebauer EA Laparoscopic versus open surgery for

suspected appendicitis Cochrane Database Syst Rev 2010 Oct 6(10)

Post-op course

Most patients can go home on the day of surgery

Recovery is usually quick

Back to school within a week sports within 2 weeks

Very low risk surgery with good outcomes

Risk of infection is about 5 at port sites or in abdomen

You see a 7 year old male in your office with 1-day history of abdominal pain

now localized to RLQ with focal guarding and poor appetite You send him for

ultrasound and it shows appendicitis His mother has heard about some new

research and wants to know if you would recommend just treating with

antibiotics rather than surgery How would you counsel this mom

A This data is preliminary and surgery is still standard of

care

B This may be a reasonable option for her son and she

should discuss it with her surgeon

C If it were your son you wouldnrsquot let him have surgery

D Other

Do all patients with appendicitis require surgery

Pilot study at Nationwide Childrenrsquos Hospital

102 patients

ndash 7 to 17 years of age

ndash Uncomplicated appendicitis defined by

Abdominal pain le 48 hours

White blood cell count le 18000

Ultrasound or CT scan showing appendicitis with an appendix le 11 centimeter thick and no

evidence of abscess or fecalith

Patients and families chose to have appendectomy or antibiotics alone

Non-operative management at least 24 hours of in-hospital observation and IV antibiotics until symptoms improved followed by completion of 10 days of treatment with antibiotics by mouth

Minneci PC Mahida JB et al The effectiveness of patient choice in non-operative

versus surgical management of uncomplicated acute appendicitis JAMA Surgery

2015 Dec 16

Results of non-operative management

65 families chose appendectomy 37 families chose non-operative management

Success rate of non-operative management (defined as not undergoing an appendectomy) 89 at 30 days 76 at 1 year

The 24 who failed did NOT have a higher rate of ruptured appendicitis compared to the patients who had immediate appendectomy

1 year follow-up the children managed non-operatively compared with the surgery group had fewer disability days (8 vs 21 days) lower appendicitis-related health care costs (median $4219 vs $5029) and no difference in health-related quality of life

Non-Operative Treatment of Appendicitis Rationale

Appendectomy is invasive

Children may miss up to two weeks of schoolactivities

Caregivers miss work

Postop complications after appendectomy for uncomplicated appendicitis 5-10

Serious complications (reoperations or readmissions) 1-7

Adult data suggest one-year success rates of 63-85 no difference in rates of complicated appendicitis

bull Meta-analysis of 10 articles reporting 413 children receiving non

operative treatment (NOT) for appendicitis ndash all published in past 10

years

bull 5 comparative

bull 1 RCT

bull 4 case series of NOT

Interpreting meta-analyses Forest plot

Favors

antibiotics

Favors

appendectomy

Overall success of NOT

97 during initial episode

Incidence of recurrent appendicitis during follow-up period (range 2-51

months)

Long term efficacy of NOT (no appendectomy at end of follow-up

period)

LOS shortened by about 05 days in those undergoing appendectomy

compared to NOT

Increased rate of complications for appendectomy compared to NOT

Authorsrsquo Conclusions

ldquoCurrent data suggest that NOT is safe It appears effective as initial treatment in 97 of children with AUA and the rate of recurrent appendicitis is 14rdquo

The study highlights the lack of robust evidence comparing NOT with appendectomy in children

Confirms a position of equipoise between treatment approaches

We recommend that NOT of children with AUA be reserved for those participating in carefully designed research studies

Take-home point

Lap appy is still standard of care for uncomplicated appendicitis

Once diagnosis of appendicitis has been establishedhellip

Non-ruptured

ndash Short duration of symptoms no suggestion of rupture on imaging

ndash Start antibiotics

ndash Perform appendectomy

Ruptured

ndash Have radiology evaluate for drainable fluid collection

ndash Antibiotics

ndash Operation in acute setting if above management fails

ndash Interval appendectomy 6-8 weeks laterhellip

ndash ALTERNATIVELY Just take out the appendix

CT ruptured appendix with abscess

After placement of percutaneous drain

How to counsel families of children with ruptured appendicitis

If kids tolerate a diet pain resolves fever resolves oral antibiotics and home

Plan interval appy 6-8 weeks later

Non-operative management may not work and surgery may be needed (Failure rate 20)

At home they should watch for signs of persistentrecurrent appendicitis

High-anxiety time for patients and families

A 10-year-old boy comes to see you in the office after recent

hospitalization for perforated appendicitis He has 3 more days of

antibiotics left His mom is worried because appetite is poor and his

energy level is low On exam he has diffuse lower abdominal

tenderness You recommend

A Extending course of oral antibiotics

B CT scan to evaluate for persistent or

recurrent appendicitis

C CT scan to evaluate for intra-abdominal

abscess

D Follow-up with surgeon

Is interval appendectomy necessary

In adults many surgeons do not do this operation

In kids data are limited

ndash 2-year follow-up of 96 patients

ndash Perforated appendicitis treated non-operatively with antibiotics

ndash 6 became worse 41 had interval appendectomy

ndash 49 received no further treatment

ndash 57 no recurrence

ndash 43 had recurrence within one month to 2 years

ndash Presence of appendicolith 72 rate of recurrence vs 26 in those without appendicolith

Ein SH1 Langer JC Daneman A Nonoperative management of pediatric ruptured appendix

with inflammatory mass or abscess presence of an appendicolith predicts recurrent

appendicitis J Pediatr Surg 2005 Oct40(10)1612-5

Approach to interval appendectomy

More strongly recommended if fecalith present

Observation is a reasonable option

Best choice for an individual patient depends on their anxiety and parental anxiety

Immediate operation for ruptured appendicitis perhaps a better

option

Meta-analysis operative vs non-operative management of pediatric

ruptured appendicitis

2 RCTs identified

Total of 171 pediatric patients

Compared early vs interval appendectomy

Early appendectomy reduced incidence of adverse event

Ruptured appendicitis may have abscess or phlegmon

Early surgery was more strongly favored when there was no abscess at

time of presentation

Decreased antibiotic duration length of stay and total charges for

abscess and no abscess groups

You are seeing a 7 year old with constipation A

fecalith was seen on abdominal X ray during recent

ED visit How would you counsel the family

A They should be referred to surgery for appendectomy

B Their child is at higher risk for appendicitis so they should be aware of this in case he develops symptoms

C It is uncertain whether this child is at higher risk for appendicitis

D A course of Miralax may help wash out the fecalith

Take home points

Uncomplicated appendicitis

ndash Lap appy is still standard of care

ndash Non-operative management may be an acceptable option but not enough is known about long-term risk of recurrent appendicitis

Complicated appendicitis

ndash Can be managed with immediate operation delayed appendectomy or no appendectomy

ndash Immediate operation is probably more efficient and less stressful for patients and parents

Thank you very much

Questions

Page 30: New approaches for evaluation and treatment of appendicitis · Ultrasound or CT scan showing appendicitis with an appendix ≤ 1.1 centimeter thick and no evidence of abscess or fecalith

Lap appy with stapler

Open appendectomy

Appendix with fecalith

Laparoscopic vs open appendectomy

Laparoscopy is standard of care

Lower complication rate

Less scar

Less pain

Ability to evaluate other intra-abdominal organs if the appendix looks normal

ndash Gallbladder

ndash Ovaries

ndash Inguinal canal

Sauerland S Jaschinski T Neugebauer EA Laparoscopic versus open surgery for

suspected appendicitis Cochrane Database Syst Rev 2010 Oct 6(10)

Post-op course

Most patients can go home on the day of surgery

Recovery is usually quick

Back to school within a week sports within 2 weeks

Very low risk surgery with good outcomes

Risk of infection is about 5 at port sites or in abdomen

You see a 7 year old male in your office with 1-day history of abdominal pain

now localized to RLQ with focal guarding and poor appetite You send him for

ultrasound and it shows appendicitis His mother has heard about some new

research and wants to know if you would recommend just treating with

antibiotics rather than surgery How would you counsel this mom

A This data is preliminary and surgery is still standard of

care

B This may be a reasonable option for her son and she

should discuss it with her surgeon

C If it were your son you wouldnrsquot let him have surgery

D Other

Do all patients with appendicitis require surgery

Pilot study at Nationwide Childrenrsquos Hospital

102 patients

ndash 7 to 17 years of age

ndash Uncomplicated appendicitis defined by

Abdominal pain le 48 hours

White blood cell count le 18000

Ultrasound or CT scan showing appendicitis with an appendix le 11 centimeter thick and no

evidence of abscess or fecalith

Patients and families chose to have appendectomy or antibiotics alone

Non-operative management at least 24 hours of in-hospital observation and IV antibiotics until symptoms improved followed by completion of 10 days of treatment with antibiotics by mouth

Minneci PC Mahida JB et al The effectiveness of patient choice in non-operative

versus surgical management of uncomplicated acute appendicitis JAMA Surgery

2015 Dec 16

Results of non-operative management

65 families chose appendectomy 37 families chose non-operative management

Success rate of non-operative management (defined as not undergoing an appendectomy) 89 at 30 days 76 at 1 year

The 24 who failed did NOT have a higher rate of ruptured appendicitis compared to the patients who had immediate appendectomy

1 year follow-up the children managed non-operatively compared with the surgery group had fewer disability days (8 vs 21 days) lower appendicitis-related health care costs (median $4219 vs $5029) and no difference in health-related quality of life

Non-Operative Treatment of Appendicitis Rationale

Appendectomy is invasive

Children may miss up to two weeks of schoolactivities

Caregivers miss work

Postop complications after appendectomy for uncomplicated appendicitis 5-10

Serious complications (reoperations or readmissions) 1-7

Adult data suggest one-year success rates of 63-85 no difference in rates of complicated appendicitis

bull Meta-analysis of 10 articles reporting 413 children receiving non

operative treatment (NOT) for appendicitis ndash all published in past 10

years

bull 5 comparative

bull 1 RCT

bull 4 case series of NOT

Interpreting meta-analyses Forest plot

Favors

antibiotics

Favors

appendectomy

Overall success of NOT

97 during initial episode

Incidence of recurrent appendicitis during follow-up period (range 2-51

months)

Long term efficacy of NOT (no appendectomy at end of follow-up

period)

LOS shortened by about 05 days in those undergoing appendectomy

compared to NOT

Increased rate of complications for appendectomy compared to NOT

Authorsrsquo Conclusions

ldquoCurrent data suggest that NOT is safe It appears effective as initial treatment in 97 of children with AUA and the rate of recurrent appendicitis is 14rdquo

The study highlights the lack of robust evidence comparing NOT with appendectomy in children

Confirms a position of equipoise between treatment approaches

We recommend that NOT of children with AUA be reserved for those participating in carefully designed research studies

Take-home point

Lap appy is still standard of care for uncomplicated appendicitis

Once diagnosis of appendicitis has been establishedhellip

Non-ruptured

ndash Short duration of symptoms no suggestion of rupture on imaging

ndash Start antibiotics

ndash Perform appendectomy

Ruptured

ndash Have radiology evaluate for drainable fluid collection

ndash Antibiotics

ndash Operation in acute setting if above management fails

ndash Interval appendectomy 6-8 weeks laterhellip

ndash ALTERNATIVELY Just take out the appendix

CT ruptured appendix with abscess

After placement of percutaneous drain

How to counsel families of children with ruptured appendicitis

If kids tolerate a diet pain resolves fever resolves oral antibiotics and home

Plan interval appy 6-8 weeks later

Non-operative management may not work and surgery may be needed (Failure rate 20)

At home they should watch for signs of persistentrecurrent appendicitis

High-anxiety time for patients and families

A 10-year-old boy comes to see you in the office after recent

hospitalization for perforated appendicitis He has 3 more days of

antibiotics left His mom is worried because appetite is poor and his

energy level is low On exam he has diffuse lower abdominal

tenderness You recommend

A Extending course of oral antibiotics

B CT scan to evaluate for persistent or

recurrent appendicitis

C CT scan to evaluate for intra-abdominal

abscess

D Follow-up with surgeon

Is interval appendectomy necessary

In adults many surgeons do not do this operation

In kids data are limited

ndash 2-year follow-up of 96 patients

ndash Perforated appendicitis treated non-operatively with antibiotics

ndash 6 became worse 41 had interval appendectomy

ndash 49 received no further treatment

ndash 57 no recurrence

ndash 43 had recurrence within one month to 2 years

ndash Presence of appendicolith 72 rate of recurrence vs 26 in those without appendicolith

Ein SH1 Langer JC Daneman A Nonoperative management of pediatric ruptured appendix

with inflammatory mass or abscess presence of an appendicolith predicts recurrent

appendicitis J Pediatr Surg 2005 Oct40(10)1612-5

Approach to interval appendectomy

More strongly recommended if fecalith present

Observation is a reasonable option

Best choice for an individual patient depends on their anxiety and parental anxiety

Immediate operation for ruptured appendicitis perhaps a better

option

Meta-analysis operative vs non-operative management of pediatric

ruptured appendicitis

2 RCTs identified

Total of 171 pediatric patients

Compared early vs interval appendectomy

Early appendectomy reduced incidence of adverse event

Ruptured appendicitis may have abscess or phlegmon

Early surgery was more strongly favored when there was no abscess at

time of presentation

Decreased antibiotic duration length of stay and total charges for

abscess and no abscess groups

You are seeing a 7 year old with constipation A

fecalith was seen on abdominal X ray during recent

ED visit How would you counsel the family

A They should be referred to surgery for appendectomy

B Their child is at higher risk for appendicitis so they should be aware of this in case he develops symptoms

C It is uncertain whether this child is at higher risk for appendicitis

D A course of Miralax may help wash out the fecalith

Take home points

Uncomplicated appendicitis

ndash Lap appy is still standard of care

ndash Non-operative management may be an acceptable option but not enough is known about long-term risk of recurrent appendicitis

Complicated appendicitis

ndash Can be managed with immediate operation delayed appendectomy or no appendectomy

ndash Immediate operation is probably more efficient and less stressful for patients and parents

Thank you very much

Questions

Page 31: New approaches for evaluation and treatment of appendicitis · Ultrasound or CT scan showing appendicitis with an appendix ≤ 1.1 centimeter thick and no evidence of abscess or fecalith

Open appendectomy

Appendix with fecalith

Laparoscopic vs open appendectomy

Laparoscopy is standard of care

Lower complication rate

Less scar

Less pain

Ability to evaluate other intra-abdominal organs if the appendix looks normal

ndash Gallbladder

ndash Ovaries

ndash Inguinal canal

Sauerland S Jaschinski T Neugebauer EA Laparoscopic versus open surgery for

suspected appendicitis Cochrane Database Syst Rev 2010 Oct 6(10)

Post-op course

Most patients can go home on the day of surgery

Recovery is usually quick

Back to school within a week sports within 2 weeks

Very low risk surgery with good outcomes

Risk of infection is about 5 at port sites or in abdomen

You see a 7 year old male in your office with 1-day history of abdominal pain

now localized to RLQ with focal guarding and poor appetite You send him for

ultrasound and it shows appendicitis His mother has heard about some new

research and wants to know if you would recommend just treating with

antibiotics rather than surgery How would you counsel this mom

A This data is preliminary and surgery is still standard of

care

B This may be a reasonable option for her son and she

should discuss it with her surgeon

C If it were your son you wouldnrsquot let him have surgery

D Other

Do all patients with appendicitis require surgery

Pilot study at Nationwide Childrenrsquos Hospital

102 patients

ndash 7 to 17 years of age

ndash Uncomplicated appendicitis defined by

Abdominal pain le 48 hours

White blood cell count le 18000

Ultrasound or CT scan showing appendicitis with an appendix le 11 centimeter thick and no

evidence of abscess or fecalith

Patients and families chose to have appendectomy or antibiotics alone

Non-operative management at least 24 hours of in-hospital observation and IV antibiotics until symptoms improved followed by completion of 10 days of treatment with antibiotics by mouth

Minneci PC Mahida JB et al The effectiveness of patient choice in non-operative

versus surgical management of uncomplicated acute appendicitis JAMA Surgery

2015 Dec 16

Results of non-operative management

65 families chose appendectomy 37 families chose non-operative management

Success rate of non-operative management (defined as not undergoing an appendectomy) 89 at 30 days 76 at 1 year

The 24 who failed did NOT have a higher rate of ruptured appendicitis compared to the patients who had immediate appendectomy

1 year follow-up the children managed non-operatively compared with the surgery group had fewer disability days (8 vs 21 days) lower appendicitis-related health care costs (median $4219 vs $5029) and no difference in health-related quality of life

Non-Operative Treatment of Appendicitis Rationale

Appendectomy is invasive

Children may miss up to two weeks of schoolactivities

Caregivers miss work

Postop complications after appendectomy for uncomplicated appendicitis 5-10

Serious complications (reoperations or readmissions) 1-7

Adult data suggest one-year success rates of 63-85 no difference in rates of complicated appendicitis

bull Meta-analysis of 10 articles reporting 413 children receiving non

operative treatment (NOT) for appendicitis ndash all published in past 10

years

bull 5 comparative

bull 1 RCT

bull 4 case series of NOT

Interpreting meta-analyses Forest plot

Favors

antibiotics

Favors

appendectomy

Overall success of NOT

97 during initial episode

Incidence of recurrent appendicitis during follow-up period (range 2-51

months)

Long term efficacy of NOT (no appendectomy at end of follow-up

period)

LOS shortened by about 05 days in those undergoing appendectomy

compared to NOT

Increased rate of complications for appendectomy compared to NOT

Authorsrsquo Conclusions

ldquoCurrent data suggest that NOT is safe It appears effective as initial treatment in 97 of children with AUA and the rate of recurrent appendicitis is 14rdquo

The study highlights the lack of robust evidence comparing NOT with appendectomy in children

Confirms a position of equipoise between treatment approaches

We recommend that NOT of children with AUA be reserved for those participating in carefully designed research studies

Take-home point

Lap appy is still standard of care for uncomplicated appendicitis

Once diagnosis of appendicitis has been establishedhellip

Non-ruptured

ndash Short duration of symptoms no suggestion of rupture on imaging

ndash Start antibiotics

ndash Perform appendectomy

Ruptured

ndash Have radiology evaluate for drainable fluid collection

ndash Antibiotics

ndash Operation in acute setting if above management fails

ndash Interval appendectomy 6-8 weeks laterhellip

ndash ALTERNATIVELY Just take out the appendix

CT ruptured appendix with abscess

After placement of percutaneous drain

How to counsel families of children with ruptured appendicitis

If kids tolerate a diet pain resolves fever resolves oral antibiotics and home

Plan interval appy 6-8 weeks later

Non-operative management may not work and surgery may be needed (Failure rate 20)

At home they should watch for signs of persistentrecurrent appendicitis

High-anxiety time for patients and families

A 10-year-old boy comes to see you in the office after recent

hospitalization for perforated appendicitis He has 3 more days of

antibiotics left His mom is worried because appetite is poor and his

energy level is low On exam he has diffuse lower abdominal

tenderness You recommend

A Extending course of oral antibiotics

B CT scan to evaluate for persistent or

recurrent appendicitis

C CT scan to evaluate for intra-abdominal

abscess

D Follow-up with surgeon

Is interval appendectomy necessary

In adults many surgeons do not do this operation

In kids data are limited

ndash 2-year follow-up of 96 patients

ndash Perforated appendicitis treated non-operatively with antibiotics

ndash 6 became worse 41 had interval appendectomy

ndash 49 received no further treatment

ndash 57 no recurrence

ndash 43 had recurrence within one month to 2 years

ndash Presence of appendicolith 72 rate of recurrence vs 26 in those without appendicolith

Ein SH1 Langer JC Daneman A Nonoperative management of pediatric ruptured appendix

with inflammatory mass or abscess presence of an appendicolith predicts recurrent

appendicitis J Pediatr Surg 2005 Oct40(10)1612-5

Approach to interval appendectomy

More strongly recommended if fecalith present

Observation is a reasonable option

Best choice for an individual patient depends on their anxiety and parental anxiety

Immediate operation for ruptured appendicitis perhaps a better

option

Meta-analysis operative vs non-operative management of pediatric

ruptured appendicitis

2 RCTs identified

Total of 171 pediatric patients

Compared early vs interval appendectomy

Early appendectomy reduced incidence of adverse event

Ruptured appendicitis may have abscess or phlegmon

Early surgery was more strongly favored when there was no abscess at

time of presentation

Decreased antibiotic duration length of stay and total charges for

abscess and no abscess groups

You are seeing a 7 year old with constipation A

fecalith was seen on abdominal X ray during recent

ED visit How would you counsel the family

A They should be referred to surgery for appendectomy

B Their child is at higher risk for appendicitis so they should be aware of this in case he develops symptoms

C It is uncertain whether this child is at higher risk for appendicitis

D A course of Miralax may help wash out the fecalith

Take home points

Uncomplicated appendicitis

ndash Lap appy is still standard of care

ndash Non-operative management may be an acceptable option but not enough is known about long-term risk of recurrent appendicitis

Complicated appendicitis

ndash Can be managed with immediate operation delayed appendectomy or no appendectomy

ndash Immediate operation is probably more efficient and less stressful for patients and parents

Thank you very much

Questions

Page 32: New approaches for evaluation and treatment of appendicitis · Ultrasound or CT scan showing appendicitis with an appendix ≤ 1.1 centimeter thick and no evidence of abscess or fecalith

Appendix with fecalith

Laparoscopic vs open appendectomy

Laparoscopy is standard of care

Lower complication rate

Less scar

Less pain

Ability to evaluate other intra-abdominal organs if the appendix looks normal

ndash Gallbladder

ndash Ovaries

ndash Inguinal canal

Sauerland S Jaschinski T Neugebauer EA Laparoscopic versus open surgery for

suspected appendicitis Cochrane Database Syst Rev 2010 Oct 6(10)

Post-op course

Most patients can go home on the day of surgery

Recovery is usually quick

Back to school within a week sports within 2 weeks

Very low risk surgery with good outcomes

Risk of infection is about 5 at port sites or in abdomen

You see a 7 year old male in your office with 1-day history of abdominal pain

now localized to RLQ with focal guarding and poor appetite You send him for

ultrasound and it shows appendicitis His mother has heard about some new

research and wants to know if you would recommend just treating with

antibiotics rather than surgery How would you counsel this mom

A This data is preliminary and surgery is still standard of

care

B This may be a reasonable option for her son and she

should discuss it with her surgeon

C If it were your son you wouldnrsquot let him have surgery

D Other

Do all patients with appendicitis require surgery

Pilot study at Nationwide Childrenrsquos Hospital

102 patients

ndash 7 to 17 years of age

ndash Uncomplicated appendicitis defined by

Abdominal pain le 48 hours

White blood cell count le 18000

Ultrasound or CT scan showing appendicitis with an appendix le 11 centimeter thick and no

evidence of abscess or fecalith

Patients and families chose to have appendectomy or antibiotics alone

Non-operative management at least 24 hours of in-hospital observation and IV antibiotics until symptoms improved followed by completion of 10 days of treatment with antibiotics by mouth

Minneci PC Mahida JB et al The effectiveness of patient choice in non-operative

versus surgical management of uncomplicated acute appendicitis JAMA Surgery

2015 Dec 16

Results of non-operative management

65 families chose appendectomy 37 families chose non-operative management

Success rate of non-operative management (defined as not undergoing an appendectomy) 89 at 30 days 76 at 1 year

The 24 who failed did NOT have a higher rate of ruptured appendicitis compared to the patients who had immediate appendectomy

1 year follow-up the children managed non-operatively compared with the surgery group had fewer disability days (8 vs 21 days) lower appendicitis-related health care costs (median $4219 vs $5029) and no difference in health-related quality of life

Non-Operative Treatment of Appendicitis Rationale

Appendectomy is invasive

Children may miss up to two weeks of schoolactivities

Caregivers miss work

Postop complications after appendectomy for uncomplicated appendicitis 5-10

Serious complications (reoperations or readmissions) 1-7

Adult data suggest one-year success rates of 63-85 no difference in rates of complicated appendicitis

bull Meta-analysis of 10 articles reporting 413 children receiving non

operative treatment (NOT) for appendicitis ndash all published in past 10

years

bull 5 comparative

bull 1 RCT

bull 4 case series of NOT

Interpreting meta-analyses Forest plot

Favors

antibiotics

Favors

appendectomy

Overall success of NOT

97 during initial episode

Incidence of recurrent appendicitis during follow-up period (range 2-51

months)

Long term efficacy of NOT (no appendectomy at end of follow-up

period)

LOS shortened by about 05 days in those undergoing appendectomy

compared to NOT

Increased rate of complications for appendectomy compared to NOT

Authorsrsquo Conclusions

ldquoCurrent data suggest that NOT is safe It appears effective as initial treatment in 97 of children with AUA and the rate of recurrent appendicitis is 14rdquo

The study highlights the lack of robust evidence comparing NOT with appendectomy in children

Confirms a position of equipoise between treatment approaches

We recommend that NOT of children with AUA be reserved for those participating in carefully designed research studies

Take-home point

Lap appy is still standard of care for uncomplicated appendicitis

Once diagnosis of appendicitis has been establishedhellip

Non-ruptured

ndash Short duration of symptoms no suggestion of rupture on imaging

ndash Start antibiotics

ndash Perform appendectomy

Ruptured

ndash Have radiology evaluate for drainable fluid collection

ndash Antibiotics

ndash Operation in acute setting if above management fails

ndash Interval appendectomy 6-8 weeks laterhellip

ndash ALTERNATIVELY Just take out the appendix

CT ruptured appendix with abscess

After placement of percutaneous drain

How to counsel families of children with ruptured appendicitis

If kids tolerate a diet pain resolves fever resolves oral antibiotics and home

Plan interval appy 6-8 weeks later

Non-operative management may not work and surgery may be needed (Failure rate 20)

At home they should watch for signs of persistentrecurrent appendicitis

High-anxiety time for patients and families

A 10-year-old boy comes to see you in the office after recent

hospitalization for perforated appendicitis He has 3 more days of

antibiotics left His mom is worried because appetite is poor and his

energy level is low On exam he has diffuse lower abdominal

tenderness You recommend

A Extending course of oral antibiotics

B CT scan to evaluate for persistent or

recurrent appendicitis

C CT scan to evaluate for intra-abdominal

abscess

D Follow-up with surgeon

Is interval appendectomy necessary

In adults many surgeons do not do this operation

In kids data are limited

ndash 2-year follow-up of 96 patients

ndash Perforated appendicitis treated non-operatively with antibiotics

ndash 6 became worse 41 had interval appendectomy

ndash 49 received no further treatment

ndash 57 no recurrence

ndash 43 had recurrence within one month to 2 years

ndash Presence of appendicolith 72 rate of recurrence vs 26 in those without appendicolith

Ein SH1 Langer JC Daneman A Nonoperative management of pediatric ruptured appendix

with inflammatory mass or abscess presence of an appendicolith predicts recurrent

appendicitis J Pediatr Surg 2005 Oct40(10)1612-5

Approach to interval appendectomy

More strongly recommended if fecalith present

Observation is a reasonable option

Best choice for an individual patient depends on their anxiety and parental anxiety

Immediate operation for ruptured appendicitis perhaps a better

option

Meta-analysis operative vs non-operative management of pediatric

ruptured appendicitis

2 RCTs identified

Total of 171 pediatric patients

Compared early vs interval appendectomy

Early appendectomy reduced incidence of adverse event

Ruptured appendicitis may have abscess or phlegmon

Early surgery was more strongly favored when there was no abscess at

time of presentation

Decreased antibiotic duration length of stay and total charges for

abscess and no abscess groups

You are seeing a 7 year old with constipation A

fecalith was seen on abdominal X ray during recent

ED visit How would you counsel the family

A They should be referred to surgery for appendectomy

B Their child is at higher risk for appendicitis so they should be aware of this in case he develops symptoms

C It is uncertain whether this child is at higher risk for appendicitis

D A course of Miralax may help wash out the fecalith

Take home points

Uncomplicated appendicitis

ndash Lap appy is still standard of care

ndash Non-operative management may be an acceptable option but not enough is known about long-term risk of recurrent appendicitis

Complicated appendicitis

ndash Can be managed with immediate operation delayed appendectomy or no appendectomy

ndash Immediate operation is probably more efficient and less stressful for patients and parents

Thank you very much

Questions

Page 33: New approaches for evaluation and treatment of appendicitis · Ultrasound or CT scan showing appendicitis with an appendix ≤ 1.1 centimeter thick and no evidence of abscess or fecalith

Laparoscopic vs open appendectomy

Laparoscopy is standard of care

Lower complication rate

Less scar

Less pain

Ability to evaluate other intra-abdominal organs if the appendix looks normal

ndash Gallbladder

ndash Ovaries

ndash Inguinal canal

Sauerland S Jaschinski T Neugebauer EA Laparoscopic versus open surgery for

suspected appendicitis Cochrane Database Syst Rev 2010 Oct 6(10)

Post-op course

Most patients can go home on the day of surgery

Recovery is usually quick

Back to school within a week sports within 2 weeks

Very low risk surgery with good outcomes

Risk of infection is about 5 at port sites or in abdomen

You see a 7 year old male in your office with 1-day history of abdominal pain

now localized to RLQ with focal guarding and poor appetite You send him for

ultrasound and it shows appendicitis His mother has heard about some new

research and wants to know if you would recommend just treating with

antibiotics rather than surgery How would you counsel this mom

A This data is preliminary and surgery is still standard of

care

B This may be a reasonable option for her son and she

should discuss it with her surgeon

C If it were your son you wouldnrsquot let him have surgery

D Other

Do all patients with appendicitis require surgery

Pilot study at Nationwide Childrenrsquos Hospital

102 patients

ndash 7 to 17 years of age

ndash Uncomplicated appendicitis defined by

Abdominal pain le 48 hours

White blood cell count le 18000

Ultrasound or CT scan showing appendicitis with an appendix le 11 centimeter thick and no

evidence of abscess or fecalith

Patients and families chose to have appendectomy or antibiotics alone

Non-operative management at least 24 hours of in-hospital observation and IV antibiotics until symptoms improved followed by completion of 10 days of treatment with antibiotics by mouth

Minneci PC Mahida JB et al The effectiveness of patient choice in non-operative

versus surgical management of uncomplicated acute appendicitis JAMA Surgery

2015 Dec 16

Results of non-operative management

65 families chose appendectomy 37 families chose non-operative management

Success rate of non-operative management (defined as not undergoing an appendectomy) 89 at 30 days 76 at 1 year

The 24 who failed did NOT have a higher rate of ruptured appendicitis compared to the patients who had immediate appendectomy

1 year follow-up the children managed non-operatively compared with the surgery group had fewer disability days (8 vs 21 days) lower appendicitis-related health care costs (median $4219 vs $5029) and no difference in health-related quality of life

Non-Operative Treatment of Appendicitis Rationale

Appendectomy is invasive

Children may miss up to two weeks of schoolactivities

Caregivers miss work

Postop complications after appendectomy for uncomplicated appendicitis 5-10

Serious complications (reoperations or readmissions) 1-7

Adult data suggest one-year success rates of 63-85 no difference in rates of complicated appendicitis

bull Meta-analysis of 10 articles reporting 413 children receiving non

operative treatment (NOT) for appendicitis ndash all published in past 10

years

bull 5 comparative

bull 1 RCT

bull 4 case series of NOT

Interpreting meta-analyses Forest plot

Favors

antibiotics

Favors

appendectomy

Overall success of NOT

97 during initial episode

Incidence of recurrent appendicitis during follow-up period (range 2-51

months)

Long term efficacy of NOT (no appendectomy at end of follow-up

period)

LOS shortened by about 05 days in those undergoing appendectomy

compared to NOT

Increased rate of complications for appendectomy compared to NOT

Authorsrsquo Conclusions

ldquoCurrent data suggest that NOT is safe It appears effective as initial treatment in 97 of children with AUA and the rate of recurrent appendicitis is 14rdquo

The study highlights the lack of robust evidence comparing NOT with appendectomy in children

Confirms a position of equipoise between treatment approaches

We recommend that NOT of children with AUA be reserved for those participating in carefully designed research studies

Take-home point

Lap appy is still standard of care for uncomplicated appendicitis

Once diagnosis of appendicitis has been establishedhellip

Non-ruptured

ndash Short duration of symptoms no suggestion of rupture on imaging

ndash Start antibiotics

ndash Perform appendectomy

Ruptured

ndash Have radiology evaluate for drainable fluid collection

ndash Antibiotics

ndash Operation in acute setting if above management fails

ndash Interval appendectomy 6-8 weeks laterhellip

ndash ALTERNATIVELY Just take out the appendix

CT ruptured appendix with abscess

After placement of percutaneous drain

How to counsel families of children with ruptured appendicitis

If kids tolerate a diet pain resolves fever resolves oral antibiotics and home

Plan interval appy 6-8 weeks later

Non-operative management may not work and surgery may be needed (Failure rate 20)

At home they should watch for signs of persistentrecurrent appendicitis

High-anxiety time for patients and families

A 10-year-old boy comes to see you in the office after recent

hospitalization for perforated appendicitis He has 3 more days of

antibiotics left His mom is worried because appetite is poor and his

energy level is low On exam he has diffuse lower abdominal

tenderness You recommend

A Extending course of oral antibiotics

B CT scan to evaluate for persistent or

recurrent appendicitis

C CT scan to evaluate for intra-abdominal

abscess

D Follow-up with surgeon

Is interval appendectomy necessary

In adults many surgeons do not do this operation

In kids data are limited

ndash 2-year follow-up of 96 patients

ndash Perforated appendicitis treated non-operatively with antibiotics

ndash 6 became worse 41 had interval appendectomy

ndash 49 received no further treatment

ndash 57 no recurrence

ndash 43 had recurrence within one month to 2 years

ndash Presence of appendicolith 72 rate of recurrence vs 26 in those without appendicolith

Ein SH1 Langer JC Daneman A Nonoperative management of pediatric ruptured appendix

with inflammatory mass or abscess presence of an appendicolith predicts recurrent

appendicitis J Pediatr Surg 2005 Oct40(10)1612-5

Approach to interval appendectomy

More strongly recommended if fecalith present

Observation is a reasonable option

Best choice for an individual patient depends on their anxiety and parental anxiety

Immediate operation for ruptured appendicitis perhaps a better

option

Meta-analysis operative vs non-operative management of pediatric

ruptured appendicitis

2 RCTs identified

Total of 171 pediatric patients

Compared early vs interval appendectomy

Early appendectomy reduced incidence of adverse event

Ruptured appendicitis may have abscess or phlegmon

Early surgery was more strongly favored when there was no abscess at

time of presentation

Decreased antibiotic duration length of stay and total charges for

abscess and no abscess groups

You are seeing a 7 year old with constipation A

fecalith was seen on abdominal X ray during recent

ED visit How would you counsel the family

A They should be referred to surgery for appendectomy

B Their child is at higher risk for appendicitis so they should be aware of this in case he develops symptoms

C It is uncertain whether this child is at higher risk for appendicitis

D A course of Miralax may help wash out the fecalith

Take home points

Uncomplicated appendicitis

ndash Lap appy is still standard of care

ndash Non-operative management may be an acceptable option but not enough is known about long-term risk of recurrent appendicitis

Complicated appendicitis

ndash Can be managed with immediate operation delayed appendectomy or no appendectomy

ndash Immediate operation is probably more efficient and less stressful for patients and parents

Thank you very much

Questions

Page 34: New approaches for evaluation and treatment of appendicitis · Ultrasound or CT scan showing appendicitis with an appendix ≤ 1.1 centimeter thick and no evidence of abscess or fecalith

Post-op course

Most patients can go home on the day of surgery

Recovery is usually quick

Back to school within a week sports within 2 weeks

Very low risk surgery with good outcomes

Risk of infection is about 5 at port sites or in abdomen

You see a 7 year old male in your office with 1-day history of abdominal pain

now localized to RLQ with focal guarding and poor appetite You send him for

ultrasound and it shows appendicitis His mother has heard about some new

research and wants to know if you would recommend just treating with

antibiotics rather than surgery How would you counsel this mom

A This data is preliminary and surgery is still standard of

care

B This may be a reasonable option for her son and she

should discuss it with her surgeon

C If it were your son you wouldnrsquot let him have surgery

D Other

Do all patients with appendicitis require surgery

Pilot study at Nationwide Childrenrsquos Hospital

102 patients

ndash 7 to 17 years of age

ndash Uncomplicated appendicitis defined by

Abdominal pain le 48 hours

White blood cell count le 18000

Ultrasound or CT scan showing appendicitis with an appendix le 11 centimeter thick and no

evidence of abscess or fecalith

Patients and families chose to have appendectomy or antibiotics alone

Non-operative management at least 24 hours of in-hospital observation and IV antibiotics until symptoms improved followed by completion of 10 days of treatment with antibiotics by mouth

Minneci PC Mahida JB et al The effectiveness of patient choice in non-operative

versus surgical management of uncomplicated acute appendicitis JAMA Surgery

2015 Dec 16

Results of non-operative management

65 families chose appendectomy 37 families chose non-operative management

Success rate of non-operative management (defined as not undergoing an appendectomy) 89 at 30 days 76 at 1 year

The 24 who failed did NOT have a higher rate of ruptured appendicitis compared to the patients who had immediate appendectomy

1 year follow-up the children managed non-operatively compared with the surgery group had fewer disability days (8 vs 21 days) lower appendicitis-related health care costs (median $4219 vs $5029) and no difference in health-related quality of life

Non-Operative Treatment of Appendicitis Rationale

Appendectomy is invasive

Children may miss up to two weeks of schoolactivities

Caregivers miss work

Postop complications after appendectomy for uncomplicated appendicitis 5-10

Serious complications (reoperations or readmissions) 1-7

Adult data suggest one-year success rates of 63-85 no difference in rates of complicated appendicitis

bull Meta-analysis of 10 articles reporting 413 children receiving non

operative treatment (NOT) for appendicitis ndash all published in past 10

years

bull 5 comparative

bull 1 RCT

bull 4 case series of NOT

Interpreting meta-analyses Forest plot

Favors

antibiotics

Favors

appendectomy

Overall success of NOT

97 during initial episode

Incidence of recurrent appendicitis during follow-up period (range 2-51

months)

Long term efficacy of NOT (no appendectomy at end of follow-up

period)

LOS shortened by about 05 days in those undergoing appendectomy

compared to NOT

Increased rate of complications for appendectomy compared to NOT

Authorsrsquo Conclusions

ldquoCurrent data suggest that NOT is safe It appears effective as initial treatment in 97 of children with AUA and the rate of recurrent appendicitis is 14rdquo

The study highlights the lack of robust evidence comparing NOT with appendectomy in children

Confirms a position of equipoise between treatment approaches

We recommend that NOT of children with AUA be reserved for those participating in carefully designed research studies

Take-home point

Lap appy is still standard of care for uncomplicated appendicitis

Once diagnosis of appendicitis has been establishedhellip

Non-ruptured

ndash Short duration of symptoms no suggestion of rupture on imaging

ndash Start antibiotics

ndash Perform appendectomy

Ruptured

ndash Have radiology evaluate for drainable fluid collection

ndash Antibiotics

ndash Operation in acute setting if above management fails

ndash Interval appendectomy 6-8 weeks laterhellip

ndash ALTERNATIVELY Just take out the appendix

CT ruptured appendix with abscess

After placement of percutaneous drain

How to counsel families of children with ruptured appendicitis

If kids tolerate a diet pain resolves fever resolves oral antibiotics and home

Plan interval appy 6-8 weeks later

Non-operative management may not work and surgery may be needed (Failure rate 20)

At home they should watch for signs of persistentrecurrent appendicitis

High-anxiety time for patients and families

A 10-year-old boy comes to see you in the office after recent

hospitalization for perforated appendicitis He has 3 more days of

antibiotics left His mom is worried because appetite is poor and his

energy level is low On exam he has diffuse lower abdominal

tenderness You recommend

A Extending course of oral antibiotics

B CT scan to evaluate for persistent or

recurrent appendicitis

C CT scan to evaluate for intra-abdominal

abscess

D Follow-up with surgeon

Is interval appendectomy necessary

In adults many surgeons do not do this operation

In kids data are limited

ndash 2-year follow-up of 96 patients

ndash Perforated appendicitis treated non-operatively with antibiotics

ndash 6 became worse 41 had interval appendectomy

ndash 49 received no further treatment

ndash 57 no recurrence

ndash 43 had recurrence within one month to 2 years

ndash Presence of appendicolith 72 rate of recurrence vs 26 in those without appendicolith

Ein SH1 Langer JC Daneman A Nonoperative management of pediatric ruptured appendix

with inflammatory mass or abscess presence of an appendicolith predicts recurrent

appendicitis J Pediatr Surg 2005 Oct40(10)1612-5

Approach to interval appendectomy

More strongly recommended if fecalith present

Observation is a reasonable option

Best choice for an individual patient depends on their anxiety and parental anxiety

Immediate operation for ruptured appendicitis perhaps a better

option

Meta-analysis operative vs non-operative management of pediatric

ruptured appendicitis

2 RCTs identified

Total of 171 pediatric patients

Compared early vs interval appendectomy

Early appendectomy reduced incidence of adverse event

Ruptured appendicitis may have abscess or phlegmon

Early surgery was more strongly favored when there was no abscess at

time of presentation

Decreased antibiotic duration length of stay and total charges for

abscess and no abscess groups

You are seeing a 7 year old with constipation A

fecalith was seen on abdominal X ray during recent

ED visit How would you counsel the family

A They should be referred to surgery for appendectomy

B Their child is at higher risk for appendicitis so they should be aware of this in case he develops symptoms

C It is uncertain whether this child is at higher risk for appendicitis

D A course of Miralax may help wash out the fecalith

Take home points

Uncomplicated appendicitis

ndash Lap appy is still standard of care

ndash Non-operative management may be an acceptable option but not enough is known about long-term risk of recurrent appendicitis

Complicated appendicitis

ndash Can be managed with immediate operation delayed appendectomy or no appendectomy

ndash Immediate operation is probably more efficient and less stressful for patients and parents

Thank you very much

Questions

Page 35: New approaches for evaluation and treatment of appendicitis · Ultrasound or CT scan showing appendicitis with an appendix ≤ 1.1 centimeter thick and no evidence of abscess or fecalith

You see a 7 year old male in your office with 1-day history of abdominal pain

now localized to RLQ with focal guarding and poor appetite You send him for

ultrasound and it shows appendicitis His mother has heard about some new

research and wants to know if you would recommend just treating with

antibiotics rather than surgery How would you counsel this mom

A This data is preliminary and surgery is still standard of

care

B This may be a reasonable option for her son and she

should discuss it with her surgeon

C If it were your son you wouldnrsquot let him have surgery

D Other

Do all patients with appendicitis require surgery

Pilot study at Nationwide Childrenrsquos Hospital

102 patients

ndash 7 to 17 years of age

ndash Uncomplicated appendicitis defined by

Abdominal pain le 48 hours

White blood cell count le 18000

Ultrasound or CT scan showing appendicitis with an appendix le 11 centimeter thick and no

evidence of abscess or fecalith

Patients and families chose to have appendectomy or antibiotics alone

Non-operative management at least 24 hours of in-hospital observation and IV antibiotics until symptoms improved followed by completion of 10 days of treatment with antibiotics by mouth

Minneci PC Mahida JB et al The effectiveness of patient choice in non-operative

versus surgical management of uncomplicated acute appendicitis JAMA Surgery

2015 Dec 16

Results of non-operative management

65 families chose appendectomy 37 families chose non-operative management

Success rate of non-operative management (defined as not undergoing an appendectomy) 89 at 30 days 76 at 1 year

The 24 who failed did NOT have a higher rate of ruptured appendicitis compared to the patients who had immediate appendectomy

1 year follow-up the children managed non-operatively compared with the surgery group had fewer disability days (8 vs 21 days) lower appendicitis-related health care costs (median $4219 vs $5029) and no difference in health-related quality of life

Non-Operative Treatment of Appendicitis Rationale

Appendectomy is invasive

Children may miss up to two weeks of schoolactivities

Caregivers miss work

Postop complications after appendectomy for uncomplicated appendicitis 5-10

Serious complications (reoperations or readmissions) 1-7

Adult data suggest one-year success rates of 63-85 no difference in rates of complicated appendicitis

bull Meta-analysis of 10 articles reporting 413 children receiving non

operative treatment (NOT) for appendicitis ndash all published in past 10

years

bull 5 comparative

bull 1 RCT

bull 4 case series of NOT

Interpreting meta-analyses Forest plot

Favors

antibiotics

Favors

appendectomy

Overall success of NOT

97 during initial episode

Incidence of recurrent appendicitis during follow-up period (range 2-51

months)

Long term efficacy of NOT (no appendectomy at end of follow-up

period)

LOS shortened by about 05 days in those undergoing appendectomy

compared to NOT

Increased rate of complications for appendectomy compared to NOT

Authorsrsquo Conclusions

ldquoCurrent data suggest that NOT is safe It appears effective as initial treatment in 97 of children with AUA and the rate of recurrent appendicitis is 14rdquo

The study highlights the lack of robust evidence comparing NOT with appendectomy in children

Confirms a position of equipoise between treatment approaches

We recommend that NOT of children with AUA be reserved for those participating in carefully designed research studies

Take-home point

Lap appy is still standard of care for uncomplicated appendicitis

Once diagnosis of appendicitis has been establishedhellip

Non-ruptured

ndash Short duration of symptoms no suggestion of rupture on imaging

ndash Start antibiotics

ndash Perform appendectomy

Ruptured

ndash Have radiology evaluate for drainable fluid collection

ndash Antibiotics

ndash Operation in acute setting if above management fails

ndash Interval appendectomy 6-8 weeks laterhellip

ndash ALTERNATIVELY Just take out the appendix

CT ruptured appendix with abscess

After placement of percutaneous drain

How to counsel families of children with ruptured appendicitis

If kids tolerate a diet pain resolves fever resolves oral antibiotics and home

Plan interval appy 6-8 weeks later

Non-operative management may not work and surgery may be needed (Failure rate 20)

At home they should watch for signs of persistentrecurrent appendicitis

High-anxiety time for patients and families

A 10-year-old boy comes to see you in the office after recent

hospitalization for perforated appendicitis He has 3 more days of

antibiotics left His mom is worried because appetite is poor and his

energy level is low On exam he has diffuse lower abdominal

tenderness You recommend

A Extending course of oral antibiotics

B CT scan to evaluate for persistent or

recurrent appendicitis

C CT scan to evaluate for intra-abdominal

abscess

D Follow-up with surgeon

Is interval appendectomy necessary

In adults many surgeons do not do this operation

In kids data are limited

ndash 2-year follow-up of 96 patients

ndash Perforated appendicitis treated non-operatively with antibiotics

ndash 6 became worse 41 had interval appendectomy

ndash 49 received no further treatment

ndash 57 no recurrence

ndash 43 had recurrence within one month to 2 years

ndash Presence of appendicolith 72 rate of recurrence vs 26 in those without appendicolith

Ein SH1 Langer JC Daneman A Nonoperative management of pediatric ruptured appendix

with inflammatory mass or abscess presence of an appendicolith predicts recurrent

appendicitis J Pediatr Surg 2005 Oct40(10)1612-5

Approach to interval appendectomy

More strongly recommended if fecalith present

Observation is a reasonable option

Best choice for an individual patient depends on their anxiety and parental anxiety

Immediate operation for ruptured appendicitis perhaps a better

option

Meta-analysis operative vs non-operative management of pediatric

ruptured appendicitis

2 RCTs identified

Total of 171 pediatric patients

Compared early vs interval appendectomy

Early appendectomy reduced incidence of adverse event

Ruptured appendicitis may have abscess or phlegmon

Early surgery was more strongly favored when there was no abscess at

time of presentation

Decreased antibiotic duration length of stay and total charges for

abscess and no abscess groups

You are seeing a 7 year old with constipation A

fecalith was seen on abdominal X ray during recent

ED visit How would you counsel the family

A They should be referred to surgery for appendectomy

B Their child is at higher risk for appendicitis so they should be aware of this in case he develops symptoms

C It is uncertain whether this child is at higher risk for appendicitis

D A course of Miralax may help wash out the fecalith

Take home points

Uncomplicated appendicitis

ndash Lap appy is still standard of care

ndash Non-operative management may be an acceptable option but not enough is known about long-term risk of recurrent appendicitis

Complicated appendicitis

ndash Can be managed with immediate operation delayed appendectomy or no appendectomy

ndash Immediate operation is probably more efficient and less stressful for patients and parents

Thank you very much

Questions

Page 36: New approaches for evaluation and treatment of appendicitis · Ultrasound or CT scan showing appendicitis with an appendix ≤ 1.1 centimeter thick and no evidence of abscess or fecalith

Do all patients with appendicitis require surgery

Pilot study at Nationwide Childrenrsquos Hospital

102 patients

ndash 7 to 17 years of age

ndash Uncomplicated appendicitis defined by

Abdominal pain le 48 hours

White blood cell count le 18000

Ultrasound or CT scan showing appendicitis with an appendix le 11 centimeter thick and no

evidence of abscess or fecalith

Patients and families chose to have appendectomy or antibiotics alone

Non-operative management at least 24 hours of in-hospital observation and IV antibiotics until symptoms improved followed by completion of 10 days of treatment with antibiotics by mouth

Minneci PC Mahida JB et al The effectiveness of patient choice in non-operative

versus surgical management of uncomplicated acute appendicitis JAMA Surgery

2015 Dec 16

Results of non-operative management

65 families chose appendectomy 37 families chose non-operative management

Success rate of non-operative management (defined as not undergoing an appendectomy) 89 at 30 days 76 at 1 year

The 24 who failed did NOT have a higher rate of ruptured appendicitis compared to the patients who had immediate appendectomy

1 year follow-up the children managed non-operatively compared with the surgery group had fewer disability days (8 vs 21 days) lower appendicitis-related health care costs (median $4219 vs $5029) and no difference in health-related quality of life

Non-Operative Treatment of Appendicitis Rationale

Appendectomy is invasive

Children may miss up to two weeks of schoolactivities

Caregivers miss work

Postop complications after appendectomy for uncomplicated appendicitis 5-10

Serious complications (reoperations or readmissions) 1-7

Adult data suggest one-year success rates of 63-85 no difference in rates of complicated appendicitis

bull Meta-analysis of 10 articles reporting 413 children receiving non

operative treatment (NOT) for appendicitis ndash all published in past 10

years

bull 5 comparative

bull 1 RCT

bull 4 case series of NOT

Interpreting meta-analyses Forest plot

Favors

antibiotics

Favors

appendectomy

Overall success of NOT

97 during initial episode

Incidence of recurrent appendicitis during follow-up period (range 2-51

months)

Long term efficacy of NOT (no appendectomy at end of follow-up

period)

LOS shortened by about 05 days in those undergoing appendectomy

compared to NOT

Increased rate of complications for appendectomy compared to NOT

Authorsrsquo Conclusions

ldquoCurrent data suggest that NOT is safe It appears effective as initial treatment in 97 of children with AUA and the rate of recurrent appendicitis is 14rdquo

The study highlights the lack of robust evidence comparing NOT with appendectomy in children

Confirms a position of equipoise between treatment approaches

We recommend that NOT of children with AUA be reserved for those participating in carefully designed research studies

Take-home point

Lap appy is still standard of care for uncomplicated appendicitis

Once diagnosis of appendicitis has been establishedhellip

Non-ruptured

ndash Short duration of symptoms no suggestion of rupture on imaging

ndash Start antibiotics

ndash Perform appendectomy

Ruptured

ndash Have radiology evaluate for drainable fluid collection

ndash Antibiotics

ndash Operation in acute setting if above management fails

ndash Interval appendectomy 6-8 weeks laterhellip

ndash ALTERNATIVELY Just take out the appendix

CT ruptured appendix with abscess

After placement of percutaneous drain

How to counsel families of children with ruptured appendicitis

If kids tolerate a diet pain resolves fever resolves oral antibiotics and home

Plan interval appy 6-8 weeks later

Non-operative management may not work and surgery may be needed (Failure rate 20)

At home they should watch for signs of persistentrecurrent appendicitis

High-anxiety time for patients and families

A 10-year-old boy comes to see you in the office after recent

hospitalization for perforated appendicitis He has 3 more days of

antibiotics left His mom is worried because appetite is poor and his

energy level is low On exam he has diffuse lower abdominal

tenderness You recommend

A Extending course of oral antibiotics

B CT scan to evaluate for persistent or

recurrent appendicitis

C CT scan to evaluate for intra-abdominal

abscess

D Follow-up with surgeon

Is interval appendectomy necessary

In adults many surgeons do not do this operation

In kids data are limited

ndash 2-year follow-up of 96 patients

ndash Perforated appendicitis treated non-operatively with antibiotics

ndash 6 became worse 41 had interval appendectomy

ndash 49 received no further treatment

ndash 57 no recurrence

ndash 43 had recurrence within one month to 2 years

ndash Presence of appendicolith 72 rate of recurrence vs 26 in those without appendicolith

Ein SH1 Langer JC Daneman A Nonoperative management of pediatric ruptured appendix

with inflammatory mass or abscess presence of an appendicolith predicts recurrent

appendicitis J Pediatr Surg 2005 Oct40(10)1612-5

Approach to interval appendectomy

More strongly recommended if fecalith present

Observation is a reasonable option

Best choice for an individual patient depends on their anxiety and parental anxiety

Immediate operation for ruptured appendicitis perhaps a better

option

Meta-analysis operative vs non-operative management of pediatric

ruptured appendicitis

2 RCTs identified

Total of 171 pediatric patients

Compared early vs interval appendectomy

Early appendectomy reduced incidence of adverse event

Ruptured appendicitis may have abscess or phlegmon

Early surgery was more strongly favored when there was no abscess at

time of presentation

Decreased antibiotic duration length of stay and total charges for

abscess and no abscess groups

You are seeing a 7 year old with constipation A

fecalith was seen on abdominal X ray during recent

ED visit How would you counsel the family

A They should be referred to surgery for appendectomy

B Their child is at higher risk for appendicitis so they should be aware of this in case he develops symptoms

C It is uncertain whether this child is at higher risk for appendicitis

D A course of Miralax may help wash out the fecalith

Take home points

Uncomplicated appendicitis

ndash Lap appy is still standard of care

ndash Non-operative management may be an acceptable option but not enough is known about long-term risk of recurrent appendicitis

Complicated appendicitis

ndash Can be managed with immediate operation delayed appendectomy or no appendectomy

ndash Immediate operation is probably more efficient and less stressful for patients and parents

Thank you very much

Questions

Page 37: New approaches for evaluation and treatment of appendicitis · Ultrasound or CT scan showing appendicitis with an appendix ≤ 1.1 centimeter thick and no evidence of abscess or fecalith

Results of non-operative management

65 families chose appendectomy 37 families chose non-operative management

Success rate of non-operative management (defined as not undergoing an appendectomy) 89 at 30 days 76 at 1 year

The 24 who failed did NOT have a higher rate of ruptured appendicitis compared to the patients who had immediate appendectomy

1 year follow-up the children managed non-operatively compared with the surgery group had fewer disability days (8 vs 21 days) lower appendicitis-related health care costs (median $4219 vs $5029) and no difference in health-related quality of life

Non-Operative Treatment of Appendicitis Rationale

Appendectomy is invasive

Children may miss up to two weeks of schoolactivities

Caregivers miss work

Postop complications after appendectomy for uncomplicated appendicitis 5-10

Serious complications (reoperations or readmissions) 1-7

Adult data suggest one-year success rates of 63-85 no difference in rates of complicated appendicitis

bull Meta-analysis of 10 articles reporting 413 children receiving non

operative treatment (NOT) for appendicitis ndash all published in past 10

years

bull 5 comparative

bull 1 RCT

bull 4 case series of NOT

Interpreting meta-analyses Forest plot

Favors

antibiotics

Favors

appendectomy

Overall success of NOT

97 during initial episode

Incidence of recurrent appendicitis during follow-up period (range 2-51

months)

Long term efficacy of NOT (no appendectomy at end of follow-up

period)

LOS shortened by about 05 days in those undergoing appendectomy

compared to NOT

Increased rate of complications for appendectomy compared to NOT

Authorsrsquo Conclusions

ldquoCurrent data suggest that NOT is safe It appears effective as initial treatment in 97 of children with AUA and the rate of recurrent appendicitis is 14rdquo

The study highlights the lack of robust evidence comparing NOT with appendectomy in children

Confirms a position of equipoise between treatment approaches

We recommend that NOT of children with AUA be reserved for those participating in carefully designed research studies

Take-home point

Lap appy is still standard of care for uncomplicated appendicitis

Once diagnosis of appendicitis has been establishedhellip

Non-ruptured

ndash Short duration of symptoms no suggestion of rupture on imaging

ndash Start antibiotics

ndash Perform appendectomy

Ruptured

ndash Have radiology evaluate for drainable fluid collection

ndash Antibiotics

ndash Operation in acute setting if above management fails

ndash Interval appendectomy 6-8 weeks laterhellip

ndash ALTERNATIVELY Just take out the appendix

CT ruptured appendix with abscess

After placement of percutaneous drain

How to counsel families of children with ruptured appendicitis

If kids tolerate a diet pain resolves fever resolves oral antibiotics and home

Plan interval appy 6-8 weeks later

Non-operative management may not work and surgery may be needed (Failure rate 20)

At home they should watch for signs of persistentrecurrent appendicitis

High-anxiety time for patients and families

A 10-year-old boy comes to see you in the office after recent

hospitalization for perforated appendicitis He has 3 more days of

antibiotics left His mom is worried because appetite is poor and his

energy level is low On exam he has diffuse lower abdominal

tenderness You recommend

A Extending course of oral antibiotics

B CT scan to evaluate for persistent or

recurrent appendicitis

C CT scan to evaluate for intra-abdominal

abscess

D Follow-up with surgeon

Is interval appendectomy necessary

In adults many surgeons do not do this operation

In kids data are limited

ndash 2-year follow-up of 96 patients

ndash Perforated appendicitis treated non-operatively with antibiotics

ndash 6 became worse 41 had interval appendectomy

ndash 49 received no further treatment

ndash 57 no recurrence

ndash 43 had recurrence within one month to 2 years

ndash Presence of appendicolith 72 rate of recurrence vs 26 in those without appendicolith

Ein SH1 Langer JC Daneman A Nonoperative management of pediatric ruptured appendix

with inflammatory mass or abscess presence of an appendicolith predicts recurrent

appendicitis J Pediatr Surg 2005 Oct40(10)1612-5

Approach to interval appendectomy

More strongly recommended if fecalith present

Observation is a reasonable option

Best choice for an individual patient depends on their anxiety and parental anxiety

Immediate operation for ruptured appendicitis perhaps a better

option

Meta-analysis operative vs non-operative management of pediatric

ruptured appendicitis

2 RCTs identified

Total of 171 pediatric patients

Compared early vs interval appendectomy

Early appendectomy reduced incidence of adverse event

Ruptured appendicitis may have abscess or phlegmon

Early surgery was more strongly favored when there was no abscess at

time of presentation

Decreased antibiotic duration length of stay and total charges for

abscess and no abscess groups

You are seeing a 7 year old with constipation A

fecalith was seen on abdominal X ray during recent

ED visit How would you counsel the family

A They should be referred to surgery for appendectomy

B Their child is at higher risk for appendicitis so they should be aware of this in case he develops symptoms

C It is uncertain whether this child is at higher risk for appendicitis

D A course of Miralax may help wash out the fecalith

Take home points

Uncomplicated appendicitis

ndash Lap appy is still standard of care

ndash Non-operative management may be an acceptable option but not enough is known about long-term risk of recurrent appendicitis

Complicated appendicitis

ndash Can be managed with immediate operation delayed appendectomy or no appendectomy

ndash Immediate operation is probably more efficient and less stressful for patients and parents

Thank you very much

Questions

Page 38: New approaches for evaluation and treatment of appendicitis · Ultrasound or CT scan showing appendicitis with an appendix ≤ 1.1 centimeter thick and no evidence of abscess or fecalith

Non-Operative Treatment of Appendicitis Rationale

Appendectomy is invasive

Children may miss up to two weeks of schoolactivities

Caregivers miss work

Postop complications after appendectomy for uncomplicated appendicitis 5-10

Serious complications (reoperations or readmissions) 1-7

Adult data suggest one-year success rates of 63-85 no difference in rates of complicated appendicitis

bull Meta-analysis of 10 articles reporting 413 children receiving non

operative treatment (NOT) for appendicitis ndash all published in past 10

years

bull 5 comparative

bull 1 RCT

bull 4 case series of NOT

Interpreting meta-analyses Forest plot

Favors

antibiotics

Favors

appendectomy

Overall success of NOT

97 during initial episode

Incidence of recurrent appendicitis during follow-up period (range 2-51

months)

Long term efficacy of NOT (no appendectomy at end of follow-up

period)

LOS shortened by about 05 days in those undergoing appendectomy

compared to NOT

Increased rate of complications for appendectomy compared to NOT

Authorsrsquo Conclusions

ldquoCurrent data suggest that NOT is safe It appears effective as initial treatment in 97 of children with AUA and the rate of recurrent appendicitis is 14rdquo

The study highlights the lack of robust evidence comparing NOT with appendectomy in children

Confirms a position of equipoise between treatment approaches

We recommend that NOT of children with AUA be reserved for those participating in carefully designed research studies

Take-home point

Lap appy is still standard of care for uncomplicated appendicitis

Once diagnosis of appendicitis has been establishedhellip

Non-ruptured

ndash Short duration of symptoms no suggestion of rupture on imaging

ndash Start antibiotics

ndash Perform appendectomy

Ruptured

ndash Have radiology evaluate for drainable fluid collection

ndash Antibiotics

ndash Operation in acute setting if above management fails

ndash Interval appendectomy 6-8 weeks laterhellip

ndash ALTERNATIVELY Just take out the appendix

CT ruptured appendix with abscess

After placement of percutaneous drain

How to counsel families of children with ruptured appendicitis

If kids tolerate a diet pain resolves fever resolves oral antibiotics and home

Plan interval appy 6-8 weeks later

Non-operative management may not work and surgery may be needed (Failure rate 20)

At home they should watch for signs of persistentrecurrent appendicitis

High-anxiety time for patients and families

A 10-year-old boy comes to see you in the office after recent

hospitalization for perforated appendicitis He has 3 more days of

antibiotics left His mom is worried because appetite is poor and his

energy level is low On exam he has diffuse lower abdominal

tenderness You recommend

A Extending course of oral antibiotics

B CT scan to evaluate for persistent or

recurrent appendicitis

C CT scan to evaluate for intra-abdominal

abscess

D Follow-up with surgeon

Is interval appendectomy necessary

In adults many surgeons do not do this operation

In kids data are limited

ndash 2-year follow-up of 96 patients

ndash Perforated appendicitis treated non-operatively with antibiotics

ndash 6 became worse 41 had interval appendectomy

ndash 49 received no further treatment

ndash 57 no recurrence

ndash 43 had recurrence within one month to 2 years

ndash Presence of appendicolith 72 rate of recurrence vs 26 in those without appendicolith

Ein SH1 Langer JC Daneman A Nonoperative management of pediatric ruptured appendix

with inflammatory mass or abscess presence of an appendicolith predicts recurrent

appendicitis J Pediatr Surg 2005 Oct40(10)1612-5

Approach to interval appendectomy

More strongly recommended if fecalith present

Observation is a reasonable option

Best choice for an individual patient depends on their anxiety and parental anxiety

Immediate operation for ruptured appendicitis perhaps a better

option

Meta-analysis operative vs non-operative management of pediatric

ruptured appendicitis

2 RCTs identified

Total of 171 pediatric patients

Compared early vs interval appendectomy

Early appendectomy reduced incidence of adverse event

Ruptured appendicitis may have abscess or phlegmon

Early surgery was more strongly favored when there was no abscess at

time of presentation

Decreased antibiotic duration length of stay and total charges for

abscess and no abscess groups

You are seeing a 7 year old with constipation A

fecalith was seen on abdominal X ray during recent

ED visit How would you counsel the family

A They should be referred to surgery for appendectomy

B Their child is at higher risk for appendicitis so they should be aware of this in case he develops symptoms

C It is uncertain whether this child is at higher risk for appendicitis

D A course of Miralax may help wash out the fecalith

Take home points

Uncomplicated appendicitis

ndash Lap appy is still standard of care

ndash Non-operative management may be an acceptable option but not enough is known about long-term risk of recurrent appendicitis

Complicated appendicitis

ndash Can be managed with immediate operation delayed appendectomy or no appendectomy

ndash Immediate operation is probably more efficient and less stressful for patients and parents

Thank you very much

Questions

Page 39: New approaches for evaluation and treatment of appendicitis · Ultrasound or CT scan showing appendicitis with an appendix ≤ 1.1 centimeter thick and no evidence of abscess or fecalith

bull Meta-analysis of 10 articles reporting 413 children receiving non

operative treatment (NOT) for appendicitis ndash all published in past 10

years

bull 5 comparative

bull 1 RCT

bull 4 case series of NOT

Interpreting meta-analyses Forest plot

Favors

antibiotics

Favors

appendectomy

Overall success of NOT

97 during initial episode

Incidence of recurrent appendicitis during follow-up period (range 2-51

months)

Long term efficacy of NOT (no appendectomy at end of follow-up

period)

LOS shortened by about 05 days in those undergoing appendectomy

compared to NOT

Increased rate of complications for appendectomy compared to NOT

Authorsrsquo Conclusions

ldquoCurrent data suggest that NOT is safe It appears effective as initial treatment in 97 of children with AUA and the rate of recurrent appendicitis is 14rdquo

The study highlights the lack of robust evidence comparing NOT with appendectomy in children

Confirms a position of equipoise between treatment approaches

We recommend that NOT of children with AUA be reserved for those participating in carefully designed research studies

Take-home point

Lap appy is still standard of care for uncomplicated appendicitis

Once diagnosis of appendicitis has been establishedhellip

Non-ruptured

ndash Short duration of symptoms no suggestion of rupture on imaging

ndash Start antibiotics

ndash Perform appendectomy

Ruptured

ndash Have radiology evaluate for drainable fluid collection

ndash Antibiotics

ndash Operation in acute setting if above management fails

ndash Interval appendectomy 6-8 weeks laterhellip

ndash ALTERNATIVELY Just take out the appendix

CT ruptured appendix with abscess

After placement of percutaneous drain

How to counsel families of children with ruptured appendicitis

If kids tolerate a diet pain resolves fever resolves oral antibiotics and home

Plan interval appy 6-8 weeks later

Non-operative management may not work and surgery may be needed (Failure rate 20)

At home they should watch for signs of persistentrecurrent appendicitis

High-anxiety time for patients and families

A 10-year-old boy comes to see you in the office after recent

hospitalization for perforated appendicitis He has 3 more days of

antibiotics left His mom is worried because appetite is poor and his

energy level is low On exam he has diffuse lower abdominal

tenderness You recommend

A Extending course of oral antibiotics

B CT scan to evaluate for persistent or

recurrent appendicitis

C CT scan to evaluate for intra-abdominal

abscess

D Follow-up with surgeon

Is interval appendectomy necessary

In adults many surgeons do not do this operation

In kids data are limited

ndash 2-year follow-up of 96 patients

ndash Perforated appendicitis treated non-operatively with antibiotics

ndash 6 became worse 41 had interval appendectomy

ndash 49 received no further treatment

ndash 57 no recurrence

ndash 43 had recurrence within one month to 2 years

ndash Presence of appendicolith 72 rate of recurrence vs 26 in those without appendicolith

Ein SH1 Langer JC Daneman A Nonoperative management of pediatric ruptured appendix

with inflammatory mass or abscess presence of an appendicolith predicts recurrent

appendicitis J Pediatr Surg 2005 Oct40(10)1612-5

Approach to interval appendectomy

More strongly recommended if fecalith present

Observation is a reasonable option

Best choice for an individual patient depends on their anxiety and parental anxiety

Immediate operation for ruptured appendicitis perhaps a better

option

Meta-analysis operative vs non-operative management of pediatric

ruptured appendicitis

2 RCTs identified

Total of 171 pediatric patients

Compared early vs interval appendectomy

Early appendectomy reduced incidence of adverse event

Ruptured appendicitis may have abscess or phlegmon

Early surgery was more strongly favored when there was no abscess at

time of presentation

Decreased antibiotic duration length of stay and total charges for

abscess and no abscess groups

You are seeing a 7 year old with constipation A

fecalith was seen on abdominal X ray during recent

ED visit How would you counsel the family

A They should be referred to surgery for appendectomy

B Their child is at higher risk for appendicitis so they should be aware of this in case he develops symptoms

C It is uncertain whether this child is at higher risk for appendicitis

D A course of Miralax may help wash out the fecalith

Take home points

Uncomplicated appendicitis

ndash Lap appy is still standard of care

ndash Non-operative management may be an acceptable option but not enough is known about long-term risk of recurrent appendicitis

Complicated appendicitis

ndash Can be managed with immediate operation delayed appendectomy or no appendectomy

ndash Immediate operation is probably more efficient and less stressful for patients and parents

Thank you very much

Questions

Page 40: New approaches for evaluation and treatment of appendicitis · Ultrasound or CT scan showing appendicitis with an appendix ≤ 1.1 centimeter thick and no evidence of abscess or fecalith

Interpreting meta-analyses Forest plot

Favors

antibiotics

Favors

appendectomy

Overall success of NOT

97 during initial episode

Incidence of recurrent appendicitis during follow-up period (range 2-51

months)

Long term efficacy of NOT (no appendectomy at end of follow-up

period)

LOS shortened by about 05 days in those undergoing appendectomy

compared to NOT

Increased rate of complications for appendectomy compared to NOT

Authorsrsquo Conclusions

ldquoCurrent data suggest that NOT is safe It appears effective as initial treatment in 97 of children with AUA and the rate of recurrent appendicitis is 14rdquo

The study highlights the lack of robust evidence comparing NOT with appendectomy in children

Confirms a position of equipoise between treatment approaches

We recommend that NOT of children with AUA be reserved for those participating in carefully designed research studies

Take-home point

Lap appy is still standard of care for uncomplicated appendicitis

Once diagnosis of appendicitis has been establishedhellip

Non-ruptured

ndash Short duration of symptoms no suggestion of rupture on imaging

ndash Start antibiotics

ndash Perform appendectomy

Ruptured

ndash Have radiology evaluate for drainable fluid collection

ndash Antibiotics

ndash Operation in acute setting if above management fails

ndash Interval appendectomy 6-8 weeks laterhellip

ndash ALTERNATIVELY Just take out the appendix

CT ruptured appendix with abscess

After placement of percutaneous drain

How to counsel families of children with ruptured appendicitis

If kids tolerate a diet pain resolves fever resolves oral antibiotics and home

Plan interval appy 6-8 weeks later

Non-operative management may not work and surgery may be needed (Failure rate 20)

At home they should watch for signs of persistentrecurrent appendicitis

High-anxiety time for patients and families

A 10-year-old boy comes to see you in the office after recent

hospitalization for perforated appendicitis He has 3 more days of

antibiotics left His mom is worried because appetite is poor and his

energy level is low On exam he has diffuse lower abdominal

tenderness You recommend

A Extending course of oral antibiotics

B CT scan to evaluate for persistent or

recurrent appendicitis

C CT scan to evaluate for intra-abdominal

abscess

D Follow-up with surgeon

Is interval appendectomy necessary

In adults many surgeons do not do this operation

In kids data are limited

ndash 2-year follow-up of 96 patients

ndash Perforated appendicitis treated non-operatively with antibiotics

ndash 6 became worse 41 had interval appendectomy

ndash 49 received no further treatment

ndash 57 no recurrence

ndash 43 had recurrence within one month to 2 years

ndash Presence of appendicolith 72 rate of recurrence vs 26 in those without appendicolith

Ein SH1 Langer JC Daneman A Nonoperative management of pediatric ruptured appendix

with inflammatory mass or abscess presence of an appendicolith predicts recurrent

appendicitis J Pediatr Surg 2005 Oct40(10)1612-5

Approach to interval appendectomy

More strongly recommended if fecalith present

Observation is a reasonable option

Best choice for an individual patient depends on their anxiety and parental anxiety

Immediate operation for ruptured appendicitis perhaps a better

option

Meta-analysis operative vs non-operative management of pediatric

ruptured appendicitis

2 RCTs identified

Total of 171 pediatric patients

Compared early vs interval appendectomy

Early appendectomy reduced incidence of adverse event

Ruptured appendicitis may have abscess or phlegmon

Early surgery was more strongly favored when there was no abscess at

time of presentation

Decreased antibiotic duration length of stay and total charges for

abscess and no abscess groups

You are seeing a 7 year old with constipation A

fecalith was seen on abdominal X ray during recent

ED visit How would you counsel the family

A They should be referred to surgery for appendectomy

B Their child is at higher risk for appendicitis so they should be aware of this in case he develops symptoms

C It is uncertain whether this child is at higher risk for appendicitis

D A course of Miralax may help wash out the fecalith

Take home points

Uncomplicated appendicitis

ndash Lap appy is still standard of care

ndash Non-operative management may be an acceptable option but not enough is known about long-term risk of recurrent appendicitis

Complicated appendicitis

ndash Can be managed with immediate operation delayed appendectomy or no appendectomy

ndash Immediate operation is probably more efficient and less stressful for patients and parents

Thank you very much

Questions

Page 41: New approaches for evaluation and treatment of appendicitis · Ultrasound or CT scan showing appendicitis with an appendix ≤ 1.1 centimeter thick and no evidence of abscess or fecalith

Overall success of NOT

97 during initial episode

Incidence of recurrent appendicitis during follow-up period (range 2-51

months)

Long term efficacy of NOT (no appendectomy at end of follow-up

period)

LOS shortened by about 05 days in those undergoing appendectomy

compared to NOT

Increased rate of complications for appendectomy compared to NOT

Authorsrsquo Conclusions

ldquoCurrent data suggest that NOT is safe It appears effective as initial treatment in 97 of children with AUA and the rate of recurrent appendicitis is 14rdquo

The study highlights the lack of robust evidence comparing NOT with appendectomy in children

Confirms a position of equipoise between treatment approaches

We recommend that NOT of children with AUA be reserved for those participating in carefully designed research studies

Take-home point

Lap appy is still standard of care for uncomplicated appendicitis

Once diagnosis of appendicitis has been establishedhellip

Non-ruptured

ndash Short duration of symptoms no suggestion of rupture on imaging

ndash Start antibiotics

ndash Perform appendectomy

Ruptured

ndash Have radiology evaluate for drainable fluid collection

ndash Antibiotics

ndash Operation in acute setting if above management fails

ndash Interval appendectomy 6-8 weeks laterhellip

ndash ALTERNATIVELY Just take out the appendix

CT ruptured appendix with abscess

After placement of percutaneous drain

How to counsel families of children with ruptured appendicitis

If kids tolerate a diet pain resolves fever resolves oral antibiotics and home

Plan interval appy 6-8 weeks later

Non-operative management may not work and surgery may be needed (Failure rate 20)

At home they should watch for signs of persistentrecurrent appendicitis

High-anxiety time for patients and families

A 10-year-old boy comes to see you in the office after recent

hospitalization for perforated appendicitis He has 3 more days of

antibiotics left His mom is worried because appetite is poor and his

energy level is low On exam he has diffuse lower abdominal

tenderness You recommend

A Extending course of oral antibiotics

B CT scan to evaluate for persistent or

recurrent appendicitis

C CT scan to evaluate for intra-abdominal

abscess

D Follow-up with surgeon

Is interval appendectomy necessary

In adults many surgeons do not do this operation

In kids data are limited

ndash 2-year follow-up of 96 patients

ndash Perforated appendicitis treated non-operatively with antibiotics

ndash 6 became worse 41 had interval appendectomy

ndash 49 received no further treatment

ndash 57 no recurrence

ndash 43 had recurrence within one month to 2 years

ndash Presence of appendicolith 72 rate of recurrence vs 26 in those without appendicolith

Ein SH1 Langer JC Daneman A Nonoperative management of pediatric ruptured appendix

with inflammatory mass or abscess presence of an appendicolith predicts recurrent

appendicitis J Pediatr Surg 2005 Oct40(10)1612-5

Approach to interval appendectomy

More strongly recommended if fecalith present

Observation is a reasonable option

Best choice for an individual patient depends on their anxiety and parental anxiety

Immediate operation for ruptured appendicitis perhaps a better

option

Meta-analysis operative vs non-operative management of pediatric

ruptured appendicitis

2 RCTs identified

Total of 171 pediatric patients

Compared early vs interval appendectomy

Early appendectomy reduced incidence of adverse event

Ruptured appendicitis may have abscess or phlegmon

Early surgery was more strongly favored when there was no abscess at

time of presentation

Decreased antibiotic duration length of stay and total charges for

abscess and no abscess groups

You are seeing a 7 year old with constipation A

fecalith was seen on abdominal X ray during recent

ED visit How would you counsel the family

A They should be referred to surgery for appendectomy

B Their child is at higher risk for appendicitis so they should be aware of this in case he develops symptoms

C It is uncertain whether this child is at higher risk for appendicitis

D A course of Miralax may help wash out the fecalith

Take home points

Uncomplicated appendicitis

ndash Lap appy is still standard of care

ndash Non-operative management may be an acceptable option but not enough is known about long-term risk of recurrent appendicitis

Complicated appendicitis

ndash Can be managed with immediate operation delayed appendectomy or no appendectomy

ndash Immediate operation is probably more efficient and less stressful for patients and parents

Thank you very much

Questions

Page 42: New approaches for evaluation and treatment of appendicitis · Ultrasound or CT scan showing appendicitis with an appendix ≤ 1.1 centimeter thick and no evidence of abscess or fecalith

Incidence of recurrent appendicitis during follow-up period (range 2-51

months)

Long term efficacy of NOT (no appendectomy at end of follow-up

period)

LOS shortened by about 05 days in those undergoing appendectomy

compared to NOT

Increased rate of complications for appendectomy compared to NOT

Authorsrsquo Conclusions

ldquoCurrent data suggest that NOT is safe It appears effective as initial treatment in 97 of children with AUA and the rate of recurrent appendicitis is 14rdquo

The study highlights the lack of robust evidence comparing NOT with appendectomy in children

Confirms a position of equipoise between treatment approaches

We recommend that NOT of children with AUA be reserved for those participating in carefully designed research studies

Take-home point

Lap appy is still standard of care for uncomplicated appendicitis

Once diagnosis of appendicitis has been establishedhellip

Non-ruptured

ndash Short duration of symptoms no suggestion of rupture on imaging

ndash Start antibiotics

ndash Perform appendectomy

Ruptured

ndash Have radiology evaluate for drainable fluid collection

ndash Antibiotics

ndash Operation in acute setting if above management fails

ndash Interval appendectomy 6-8 weeks laterhellip

ndash ALTERNATIVELY Just take out the appendix

CT ruptured appendix with abscess

After placement of percutaneous drain

How to counsel families of children with ruptured appendicitis

If kids tolerate a diet pain resolves fever resolves oral antibiotics and home

Plan interval appy 6-8 weeks later

Non-operative management may not work and surgery may be needed (Failure rate 20)

At home they should watch for signs of persistentrecurrent appendicitis

High-anxiety time for patients and families

A 10-year-old boy comes to see you in the office after recent

hospitalization for perforated appendicitis He has 3 more days of

antibiotics left His mom is worried because appetite is poor and his

energy level is low On exam he has diffuse lower abdominal

tenderness You recommend

A Extending course of oral antibiotics

B CT scan to evaluate for persistent or

recurrent appendicitis

C CT scan to evaluate for intra-abdominal

abscess

D Follow-up with surgeon

Is interval appendectomy necessary

In adults many surgeons do not do this operation

In kids data are limited

ndash 2-year follow-up of 96 patients

ndash Perforated appendicitis treated non-operatively with antibiotics

ndash 6 became worse 41 had interval appendectomy

ndash 49 received no further treatment

ndash 57 no recurrence

ndash 43 had recurrence within one month to 2 years

ndash Presence of appendicolith 72 rate of recurrence vs 26 in those without appendicolith

Ein SH1 Langer JC Daneman A Nonoperative management of pediatric ruptured appendix

with inflammatory mass or abscess presence of an appendicolith predicts recurrent

appendicitis J Pediatr Surg 2005 Oct40(10)1612-5

Approach to interval appendectomy

More strongly recommended if fecalith present

Observation is a reasonable option

Best choice for an individual patient depends on their anxiety and parental anxiety

Immediate operation for ruptured appendicitis perhaps a better

option

Meta-analysis operative vs non-operative management of pediatric

ruptured appendicitis

2 RCTs identified

Total of 171 pediatric patients

Compared early vs interval appendectomy

Early appendectomy reduced incidence of adverse event

Ruptured appendicitis may have abscess or phlegmon

Early surgery was more strongly favored when there was no abscess at

time of presentation

Decreased antibiotic duration length of stay and total charges for

abscess and no abscess groups

You are seeing a 7 year old with constipation A

fecalith was seen on abdominal X ray during recent

ED visit How would you counsel the family

A They should be referred to surgery for appendectomy

B Their child is at higher risk for appendicitis so they should be aware of this in case he develops symptoms

C It is uncertain whether this child is at higher risk for appendicitis

D A course of Miralax may help wash out the fecalith

Take home points

Uncomplicated appendicitis

ndash Lap appy is still standard of care

ndash Non-operative management may be an acceptable option but not enough is known about long-term risk of recurrent appendicitis

Complicated appendicitis

ndash Can be managed with immediate operation delayed appendectomy or no appendectomy

ndash Immediate operation is probably more efficient and less stressful for patients and parents

Thank you very much

Questions

Page 43: New approaches for evaluation and treatment of appendicitis · Ultrasound or CT scan showing appendicitis with an appendix ≤ 1.1 centimeter thick and no evidence of abscess or fecalith

Long term efficacy of NOT (no appendectomy at end of follow-up

period)

LOS shortened by about 05 days in those undergoing appendectomy

compared to NOT

Increased rate of complications for appendectomy compared to NOT

Authorsrsquo Conclusions

ldquoCurrent data suggest that NOT is safe It appears effective as initial treatment in 97 of children with AUA and the rate of recurrent appendicitis is 14rdquo

The study highlights the lack of robust evidence comparing NOT with appendectomy in children

Confirms a position of equipoise between treatment approaches

We recommend that NOT of children with AUA be reserved for those participating in carefully designed research studies

Take-home point

Lap appy is still standard of care for uncomplicated appendicitis

Once diagnosis of appendicitis has been establishedhellip

Non-ruptured

ndash Short duration of symptoms no suggestion of rupture on imaging

ndash Start antibiotics

ndash Perform appendectomy

Ruptured

ndash Have radiology evaluate for drainable fluid collection

ndash Antibiotics

ndash Operation in acute setting if above management fails

ndash Interval appendectomy 6-8 weeks laterhellip

ndash ALTERNATIVELY Just take out the appendix

CT ruptured appendix with abscess

After placement of percutaneous drain

How to counsel families of children with ruptured appendicitis

If kids tolerate a diet pain resolves fever resolves oral antibiotics and home

Plan interval appy 6-8 weeks later

Non-operative management may not work and surgery may be needed (Failure rate 20)

At home they should watch for signs of persistentrecurrent appendicitis

High-anxiety time for patients and families

A 10-year-old boy comes to see you in the office after recent

hospitalization for perforated appendicitis He has 3 more days of

antibiotics left His mom is worried because appetite is poor and his

energy level is low On exam he has diffuse lower abdominal

tenderness You recommend

A Extending course of oral antibiotics

B CT scan to evaluate for persistent or

recurrent appendicitis

C CT scan to evaluate for intra-abdominal

abscess

D Follow-up with surgeon

Is interval appendectomy necessary

In adults many surgeons do not do this operation

In kids data are limited

ndash 2-year follow-up of 96 patients

ndash Perforated appendicitis treated non-operatively with antibiotics

ndash 6 became worse 41 had interval appendectomy

ndash 49 received no further treatment

ndash 57 no recurrence

ndash 43 had recurrence within one month to 2 years

ndash Presence of appendicolith 72 rate of recurrence vs 26 in those without appendicolith

Ein SH1 Langer JC Daneman A Nonoperative management of pediatric ruptured appendix

with inflammatory mass or abscess presence of an appendicolith predicts recurrent

appendicitis J Pediatr Surg 2005 Oct40(10)1612-5

Approach to interval appendectomy

More strongly recommended if fecalith present

Observation is a reasonable option

Best choice for an individual patient depends on their anxiety and parental anxiety

Immediate operation for ruptured appendicitis perhaps a better

option

Meta-analysis operative vs non-operative management of pediatric

ruptured appendicitis

2 RCTs identified

Total of 171 pediatric patients

Compared early vs interval appendectomy

Early appendectomy reduced incidence of adverse event

Ruptured appendicitis may have abscess or phlegmon

Early surgery was more strongly favored when there was no abscess at

time of presentation

Decreased antibiotic duration length of stay and total charges for

abscess and no abscess groups

You are seeing a 7 year old with constipation A

fecalith was seen on abdominal X ray during recent

ED visit How would you counsel the family

A They should be referred to surgery for appendectomy

B Their child is at higher risk for appendicitis so they should be aware of this in case he develops symptoms

C It is uncertain whether this child is at higher risk for appendicitis

D A course of Miralax may help wash out the fecalith

Take home points

Uncomplicated appendicitis

ndash Lap appy is still standard of care

ndash Non-operative management may be an acceptable option but not enough is known about long-term risk of recurrent appendicitis

Complicated appendicitis

ndash Can be managed with immediate operation delayed appendectomy or no appendectomy

ndash Immediate operation is probably more efficient and less stressful for patients and parents

Thank you very much

Questions

Page 44: New approaches for evaluation and treatment of appendicitis · Ultrasound or CT scan showing appendicitis with an appendix ≤ 1.1 centimeter thick and no evidence of abscess or fecalith

LOS shortened by about 05 days in those undergoing appendectomy

compared to NOT

Increased rate of complications for appendectomy compared to NOT

Authorsrsquo Conclusions

ldquoCurrent data suggest that NOT is safe It appears effective as initial treatment in 97 of children with AUA and the rate of recurrent appendicitis is 14rdquo

The study highlights the lack of robust evidence comparing NOT with appendectomy in children

Confirms a position of equipoise between treatment approaches

We recommend that NOT of children with AUA be reserved for those participating in carefully designed research studies

Take-home point

Lap appy is still standard of care for uncomplicated appendicitis

Once diagnosis of appendicitis has been establishedhellip

Non-ruptured

ndash Short duration of symptoms no suggestion of rupture on imaging

ndash Start antibiotics

ndash Perform appendectomy

Ruptured

ndash Have radiology evaluate for drainable fluid collection

ndash Antibiotics

ndash Operation in acute setting if above management fails

ndash Interval appendectomy 6-8 weeks laterhellip

ndash ALTERNATIVELY Just take out the appendix

CT ruptured appendix with abscess

After placement of percutaneous drain

How to counsel families of children with ruptured appendicitis

If kids tolerate a diet pain resolves fever resolves oral antibiotics and home

Plan interval appy 6-8 weeks later

Non-operative management may not work and surgery may be needed (Failure rate 20)

At home they should watch for signs of persistentrecurrent appendicitis

High-anxiety time for patients and families

A 10-year-old boy comes to see you in the office after recent

hospitalization for perforated appendicitis He has 3 more days of

antibiotics left His mom is worried because appetite is poor and his

energy level is low On exam he has diffuse lower abdominal

tenderness You recommend

A Extending course of oral antibiotics

B CT scan to evaluate for persistent or

recurrent appendicitis

C CT scan to evaluate for intra-abdominal

abscess

D Follow-up with surgeon

Is interval appendectomy necessary

In adults many surgeons do not do this operation

In kids data are limited

ndash 2-year follow-up of 96 patients

ndash Perforated appendicitis treated non-operatively with antibiotics

ndash 6 became worse 41 had interval appendectomy

ndash 49 received no further treatment

ndash 57 no recurrence

ndash 43 had recurrence within one month to 2 years

ndash Presence of appendicolith 72 rate of recurrence vs 26 in those without appendicolith

Ein SH1 Langer JC Daneman A Nonoperative management of pediatric ruptured appendix

with inflammatory mass or abscess presence of an appendicolith predicts recurrent

appendicitis J Pediatr Surg 2005 Oct40(10)1612-5

Approach to interval appendectomy

More strongly recommended if fecalith present

Observation is a reasonable option

Best choice for an individual patient depends on their anxiety and parental anxiety

Immediate operation for ruptured appendicitis perhaps a better

option

Meta-analysis operative vs non-operative management of pediatric

ruptured appendicitis

2 RCTs identified

Total of 171 pediatric patients

Compared early vs interval appendectomy

Early appendectomy reduced incidence of adverse event

Ruptured appendicitis may have abscess or phlegmon

Early surgery was more strongly favored when there was no abscess at

time of presentation

Decreased antibiotic duration length of stay and total charges for

abscess and no abscess groups

You are seeing a 7 year old with constipation A

fecalith was seen on abdominal X ray during recent

ED visit How would you counsel the family

A They should be referred to surgery for appendectomy

B Their child is at higher risk for appendicitis so they should be aware of this in case he develops symptoms

C It is uncertain whether this child is at higher risk for appendicitis

D A course of Miralax may help wash out the fecalith

Take home points

Uncomplicated appendicitis

ndash Lap appy is still standard of care

ndash Non-operative management may be an acceptable option but not enough is known about long-term risk of recurrent appendicitis

Complicated appendicitis

ndash Can be managed with immediate operation delayed appendectomy or no appendectomy

ndash Immediate operation is probably more efficient and less stressful for patients and parents

Thank you very much

Questions

Page 45: New approaches for evaluation and treatment of appendicitis · Ultrasound or CT scan showing appendicitis with an appendix ≤ 1.1 centimeter thick and no evidence of abscess or fecalith

Increased rate of complications for appendectomy compared to NOT

Authorsrsquo Conclusions

ldquoCurrent data suggest that NOT is safe It appears effective as initial treatment in 97 of children with AUA and the rate of recurrent appendicitis is 14rdquo

The study highlights the lack of robust evidence comparing NOT with appendectomy in children

Confirms a position of equipoise between treatment approaches

We recommend that NOT of children with AUA be reserved for those participating in carefully designed research studies

Take-home point

Lap appy is still standard of care for uncomplicated appendicitis

Once diagnosis of appendicitis has been establishedhellip

Non-ruptured

ndash Short duration of symptoms no suggestion of rupture on imaging

ndash Start antibiotics

ndash Perform appendectomy

Ruptured

ndash Have radiology evaluate for drainable fluid collection

ndash Antibiotics

ndash Operation in acute setting if above management fails

ndash Interval appendectomy 6-8 weeks laterhellip

ndash ALTERNATIVELY Just take out the appendix

CT ruptured appendix with abscess

After placement of percutaneous drain

How to counsel families of children with ruptured appendicitis

If kids tolerate a diet pain resolves fever resolves oral antibiotics and home

Plan interval appy 6-8 weeks later

Non-operative management may not work and surgery may be needed (Failure rate 20)

At home they should watch for signs of persistentrecurrent appendicitis

High-anxiety time for patients and families

A 10-year-old boy comes to see you in the office after recent

hospitalization for perforated appendicitis He has 3 more days of

antibiotics left His mom is worried because appetite is poor and his

energy level is low On exam he has diffuse lower abdominal

tenderness You recommend

A Extending course of oral antibiotics

B CT scan to evaluate for persistent or

recurrent appendicitis

C CT scan to evaluate for intra-abdominal

abscess

D Follow-up with surgeon

Is interval appendectomy necessary

In adults many surgeons do not do this operation

In kids data are limited

ndash 2-year follow-up of 96 patients

ndash Perforated appendicitis treated non-operatively with antibiotics

ndash 6 became worse 41 had interval appendectomy

ndash 49 received no further treatment

ndash 57 no recurrence

ndash 43 had recurrence within one month to 2 years

ndash Presence of appendicolith 72 rate of recurrence vs 26 in those without appendicolith

Ein SH1 Langer JC Daneman A Nonoperative management of pediatric ruptured appendix

with inflammatory mass or abscess presence of an appendicolith predicts recurrent

appendicitis J Pediatr Surg 2005 Oct40(10)1612-5

Approach to interval appendectomy

More strongly recommended if fecalith present

Observation is a reasonable option

Best choice for an individual patient depends on their anxiety and parental anxiety

Immediate operation for ruptured appendicitis perhaps a better

option

Meta-analysis operative vs non-operative management of pediatric

ruptured appendicitis

2 RCTs identified

Total of 171 pediatric patients

Compared early vs interval appendectomy

Early appendectomy reduced incidence of adverse event

Ruptured appendicitis may have abscess or phlegmon

Early surgery was more strongly favored when there was no abscess at

time of presentation

Decreased antibiotic duration length of stay and total charges for

abscess and no abscess groups

You are seeing a 7 year old with constipation A

fecalith was seen on abdominal X ray during recent

ED visit How would you counsel the family

A They should be referred to surgery for appendectomy

B Their child is at higher risk for appendicitis so they should be aware of this in case he develops symptoms

C It is uncertain whether this child is at higher risk for appendicitis

D A course of Miralax may help wash out the fecalith

Take home points

Uncomplicated appendicitis

ndash Lap appy is still standard of care

ndash Non-operative management may be an acceptable option but not enough is known about long-term risk of recurrent appendicitis

Complicated appendicitis

ndash Can be managed with immediate operation delayed appendectomy or no appendectomy

ndash Immediate operation is probably more efficient and less stressful for patients and parents

Thank you very much

Questions

Page 46: New approaches for evaluation and treatment of appendicitis · Ultrasound or CT scan showing appendicitis with an appendix ≤ 1.1 centimeter thick and no evidence of abscess or fecalith

Authorsrsquo Conclusions

ldquoCurrent data suggest that NOT is safe It appears effective as initial treatment in 97 of children with AUA and the rate of recurrent appendicitis is 14rdquo

The study highlights the lack of robust evidence comparing NOT with appendectomy in children

Confirms a position of equipoise between treatment approaches

We recommend that NOT of children with AUA be reserved for those participating in carefully designed research studies

Take-home point

Lap appy is still standard of care for uncomplicated appendicitis

Once diagnosis of appendicitis has been establishedhellip

Non-ruptured

ndash Short duration of symptoms no suggestion of rupture on imaging

ndash Start antibiotics

ndash Perform appendectomy

Ruptured

ndash Have radiology evaluate for drainable fluid collection

ndash Antibiotics

ndash Operation in acute setting if above management fails

ndash Interval appendectomy 6-8 weeks laterhellip

ndash ALTERNATIVELY Just take out the appendix

CT ruptured appendix with abscess

After placement of percutaneous drain

How to counsel families of children with ruptured appendicitis

If kids tolerate a diet pain resolves fever resolves oral antibiotics and home

Plan interval appy 6-8 weeks later

Non-operative management may not work and surgery may be needed (Failure rate 20)

At home they should watch for signs of persistentrecurrent appendicitis

High-anxiety time for patients and families

A 10-year-old boy comes to see you in the office after recent

hospitalization for perforated appendicitis He has 3 more days of

antibiotics left His mom is worried because appetite is poor and his

energy level is low On exam he has diffuse lower abdominal

tenderness You recommend

A Extending course of oral antibiotics

B CT scan to evaluate for persistent or

recurrent appendicitis

C CT scan to evaluate for intra-abdominal

abscess

D Follow-up with surgeon

Is interval appendectomy necessary

In adults many surgeons do not do this operation

In kids data are limited

ndash 2-year follow-up of 96 patients

ndash Perforated appendicitis treated non-operatively with antibiotics

ndash 6 became worse 41 had interval appendectomy

ndash 49 received no further treatment

ndash 57 no recurrence

ndash 43 had recurrence within one month to 2 years

ndash Presence of appendicolith 72 rate of recurrence vs 26 in those without appendicolith

Ein SH1 Langer JC Daneman A Nonoperative management of pediatric ruptured appendix

with inflammatory mass or abscess presence of an appendicolith predicts recurrent

appendicitis J Pediatr Surg 2005 Oct40(10)1612-5

Approach to interval appendectomy

More strongly recommended if fecalith present

Observation is a reasonable option

Best choice for an individual patient depends on their anxiety and parental anxiety

Immediate operation for ruptured appendicitis perhaps a better

option

Meta-analysis operative vs non-operative management of pediatric

ruptured appendicitis

2 RCTs identified

Total of 171 pediatric patients

Compared early vs interval appendectomy

Early appendectomy reduced incidence of adverse event

Ruptured appendicitis may have abscess or phlegmon

Early surgery was more strongly favored when there was no abscess at

time of presentation

Decreased antibiotic duration length of stay and total charges for

abscess and no abscess groups

You are seeing a 7 year old with constipation A

fecalith was seen on abdominal X ray during recent

ED visit How would you counsel the family

A They should be referred to surgery for appendectomy

B Their child is at higher risk for appendicitis so they should be aware of this in case he develops symptoms

C It is uncertain whether this child is at higher risk for appendicitis

D A course of Miralax may help wash out the fecalith

Take home points

Uncomplicated appendicitis

ndash Lap appy is still standard of care

ndash Non-operative management may be an acceptable option but not enough is known about long-term risk of recurrent appendicitis

Complicated appendicitis

ndash Can be managed with immediate operation delayed appendectomy or no appendectomy

ndash Immediate operation is probably more efficient and less stressful for patients and parents

Thank you very much

Questions

Page 47: New approaches for evaluation and treatment of appendicitis · Ultrasound or CT scan showing appendicitis with an appendix ≤ 1.1 centimeter thick and no evidence of abscess or fecalith

Take-home point

Lap appy is still standard of care for uncomplicated appendicitis

Once diagnosis of appendicitis has been establishedhellip

Non-ruptured

ndash Short duration of symptoms no suggestion of rupture on imaging

ndash Start antibiotics

ndash Perform appendectomy

Ruptured

ndash Have radiology evaluate for drainable fluid collection

ndash Antibiotics

ndash Operation in acute setting if above management fails

ndash Interval appendectomy 6-8 weeks laterhellip

ndash ALTERNATIVELY Just take out the appendix

CT ruptured appendix with abscess

After placement of percutaneous drain

How to counsel families of children with ruptured appendicitis

If kids tolerate a diet pain resolves fever resolves oral antibiotics and home

Plan interval appy 6-8 weeks later

Non-operative management may not work and surgery may be needed (Failure rate 20)

At home they should watch for signs of persistentrecurrent appendicitis

High-anxiety time for patients and families

A 10-year-old boy comes to see you in the office after recent

hospitalization for perforated appendicitis He has 3 more days of

antibiotics left His mom is worried because appetite is poor and his

energy level is low On exam he has diffuse lower abdominal

tenderness You recommend

A Extending course of oral antibiotics

B CT scan to evaluate for persistent or

recurrent appendicitis

C CT scan to evaluate for intra-abdominal

abscess

D Follow-up with surgeon

Is interval appendectomy necessary

In adults many surgeons do not do this operation

In kids data are limited

ndash 2-year follow-up of 96 patients

ndash Perforated appendicitis treated non-operatively with antibiotics

ndash 6 became worse 41 had interval appendectomy

ndash 49 received no further treatment

ndash 57 no recurrence

ndash 43 had recurrence within one month to 2 years

ndash Presence of appendicolith 72 rate of recurrence vs 26 in those without appendicolith

Ein SH1 Langer JC Daneman A Nonoperative management of pediatric ruptured appendix

with inflammatory mass or abscess presence of an appendicolith predicts recurrent

appendicitis J Pediatr Surg 2005 Oct40(10)1612-5

Approach to interval appendectomy

More strongly recommended if fecalith present

Observation is a reasonable option

Best choice for an individual patient depends on their anxiety and parental anxiety

Immediate operation for ruptured appendicitis perhaps a better

option

Meta-analysis operative vs non-operative management of pediatric

ruptured appendicitis

2 RCTs identified

Total of 171 pediatric patients

Compared early vs interval appendectomy

Early appendectomy reduced incidence of adverse event

Ruptured appendicitis may have abscess or phlegmon

Early surgery was more strongly favored when there was no abscess at

time of presentation

Decreased antibiotic duration length of stay and total charges for

abscess and no abscess groups

You are seeing a 7 year old with constipation A

fecalith was seen on abdominal X ray during recent

ED visit How would you counsel the family

A They should be referred to surgery for appendectomy

B Their child is at higher risk for appendicitis so they should be aware of this in case he develops symptoms

C It is uncertain whether this child is at higher risk for appendicitis

D A course of Miralax may help wash out the fecalith

Take home points

Uncomplicated appendicitis

ndash Lap appy is still standard of care

ndash Non-operative management may be an acceptable option but not enough is known about long-term risk of recurrent appendicitis

Complicated appendicitis

ndash Can be managed with immediate operation delayed appendectomy or no appendectomy

ndash Immediate operation is probably more efficient and less stressful for patients and parents

Thank you very much

Questions

Page 48: New approaches for evaluation and treatment of appendicitis · Ultrasound or CT scan showing appendicitis with an appendix ≤ 1.1 centimeter thick and no evidence of abscess or fecalith

Once diagnosis of appendicitis has been establishedhellip

Non-ruptured

ndash Short duration of symptoms no suggestion of rupture on imaging

ndash Start antibiotics

ndash Perform appendectomy

Ruptured

ndash Have radiology evaluate for drainable fluid collection

ndash Antibiotics

ndash Operation in acute setting if above management fails

ndash Interval appendectomy 6-8 weeks laterhellip

ndash ALTERNATIVELY Just take out the appendix

CT ruptured appendix with abscess

After placement of percutaneous drain

How to counsel families of children with ruptured appendicitis

If kids tolerate a diet pain resolves fever resolves oral antibiotics and home

Plan interval appy 6-8 weeks later

Non-operative management may not work and surgery may be needed (Failure rate 20)

At home they should watch for signs of persistentrecurrent appendicitis

High-anxiety time for patients and families

A 10-year-old boy comes to see you in the office after recent

hospitalization for perforated appendicitis He has 3 more days of

antibiotics left His mom is worried because appetite is poor and his

energy level is low On exam he has diffuse lower abdominal

tenderness You recommend

A Extending course of oral antibiotics

B CT scan to evaluate for persistent or

recurrent appendicitis

C CT scan to evaluate for intra-abdominal

abscess

D Follow-up with surgeon

Is interval appendectomy necessary

In adults many surgeons do not do this operation

In kids data are limited

ndash 2-year follow-up of 96 patients

ndash Perforated appendicitis treated non-operatively with antibiotics

ndash 6 became worse 41 had interval appendectomy

ndash 49 received no further treatment

ndash 57 no recurrence

ndash 43 had recurrence within one month to 2 years

ndash Presence of appendicolith 72 rate of recurrence vs 26 in those without appendicolith

Ein SH1 Langer JC Daneman A Nonoperative management of pediatric ruptured appendix

with inflammatory mass or abscess presence of an appendicolith predicts recurrent

appendicitis J Pediatr Surg 2005 Oct40(10)1612-5

Approach to interval appendectomy

More strongly recommended if fecalith present

Observation is a reasonable option

Best choice for an individual patient depends on their anxiety and parental anxiety

Immediate operation for ruptured appendicitis perhaps a better

option

Meta-analysis operative vs non-operative management of pediatric

ruptured appendicitis

2 RCTs identified

Total of 171 pediatric patients

Compared early vs interval appendectomy

Early appendectomy reduced incidence of adverse event

Ruptured appendicitis may have abscess or phlegmon

Early surgery was more strongly favored when there was no abscess at

time of presentation

Decreased antibiotic duration length of stay and total charges for

abscess and no abscess groups

You are seeing a 7 year old with constipation A

fecalith was seen on abdominal X ray during recent

ED visit How would you counsel the family

A They should be referred to surgery for appendectomy

B Their child is at higher risk for appendicitis so they should be aware of this in case he develops symptoms

C It is uncertain whether this child is at higher risk for appendicitis

D A course of Miralax may help wash out the fecalith

Take home points

Uncomplicated appendicitis

ndash Lap appy is still standard of care

ndash Non-operative management may be an acceptable option but not enough is known about long-term risk of recurrent appendicitis

Complicated appendicitis

ndash Can be managed with immediate operation delayed appendectomy or no appendectomy

ndash Immediate operation is probably more efficient and less stressful for patients and parents

Thank you very much

Questions

Page 49: New approaches for evaluation and treatment of appendicitis · Ultrasound or CT scan showing appendicitis with an appendix ≤ 1.1 centimeter thick and no evidence of abscess or fecalith

CT ruptured appendix with abscess

After placement of percutaneous drain

How to counsel families of children with ruptured appendicitis

If kids tolerate a diet pain resolves fever resolves oral antibiotics and home

Plan interval appy 6-8 weeks later

Non-operative management may not work and surgery may be needed (Failure rate 20)

At home they should watch for signs of persistentrecurrent appendicitis

High-anxiety time for patients and families

A 10-year-old boy comes to see you in the office after recent

hospitalization for perforated appendicitis He has 3 more days of

antibiotics left His mom is worried because appetite is poor and his

energy level is low On exam he has diffuse lower abdominal

tenderness You recommend

A Extending course of oral antibiotics

B CT scan to evaluate for persistent or

recurrent appendicitis

C CT scan to evaluate for intra-abdominal

abscess

D Follow-up with surgeon

Is interval appendectomy necessary

In adults many surgeons do not do this operation

In kids data are limited

ndash 2-year follow-up of 96 patients

ndash Perforated appendicitis treated non-operatively with antibiotics

ndash 6 became worse 41 had interval appendectomy

ndash 49 received no further treatment

ndash 57 no recurrence

ndash 43 had recurrence within one month to 2 years

ndash Presence of appendicolith 72 rate of recurrence vs 26 in those without appendicolith

Ein SH1 Langer JC Daneman A Nonoperative management of pediatric ruptured appendix

with inflammatory mass or abscess presence of an appendicolith predicts recurrent

appendicitis J Pediatr Surg 2005 Oct40(10)1612-5

Approach to interval appendectomy

More strongly recommended if fecalith present

Observation is a reasonable option

Best choice for an individual patient depends on their anxiety and parental anxiety

Immediate operation for ruptured appendicitis perhaps a better

option

Meta-analysis operative vs non-operative management of pediatric

ruptured appendicitis

2 RCTs identified

Total of 171 pediatric patients

Compared early vs interval appendectomy

Early appendectomy reduced incidence of adverse event

Ruptured appendicitis may have abscess or phlegmon

Early surgery was more strongly favored when there was no abscess at

time of presentation

Decreased antibiotic duration length of stay and total charges for

abscess and no abscess groups

You are seeing a 7 year old with constipation A

fecalith was seen on abdominal X ray during recent

ED visit How would you counsel the family

A They should be referred to surgery for appendectomy

B Their child is at higher risk for appendicitis so they should be aware of this in case he develops symptoms

C It is uncertain whether this child is at higher risk for appendicitis

D A course of Miralax may help wash out the fecalith

Take home points

Uncomplicated appendicitis

ndash Lap appy is still standard of care

ndash Non-operative management may be an acceptable option but not enough is known about long-term risk of recurrent appendicitis

Complicated appendicitis

ndash Can be managed with immediate operation delayed appendectomy or no appendectomy

ndash Immediate operation is probably more efficient and less stressful for patients and parents

Thank you very much

Questions

Page 50: New approaches for evaluation and treatment of appendicitis · Ultrasound or CT scan showing appendicitis with an appendix ≤ 1.1 centimeter thick and no evidence of abscess or fecalith

After placement of percutaneous drain

How to counsel families of children with ruptured appendicitis

If kids tolerate a diet pain resolves fever resolves oral antibiotics and home

Plan interval appy 6-8 weeks later

Non-operative management may not work and surgery may be needed (Failure rate 20)

At home they should watch for signs of persistentrecurrent appendicitis

High-anxiety time for patients and families

A 10-year-old boy comes to see you in the office after recent

hospitalization for perforated appendicitis He has 3 more days of

antibiotics left His mom is worried because appetite is poor and his

energy level is low On exam he has diffuse lower abdominal

tenderness You recommend

A Extending course of oral antibiotics

B CT scan to evaluate for persistent or

recurrent appendicitis

C CT scan to evaluate for intra-abdominal

abscess

D Follow-up with surgeon

Is interval appendectomy necessary

In adults many surgeons do not do this operation

In kids data are limited

ndash 2-year follow-up of 96 patients

ndash Perforated appendicitis treated non-operatively with antibiotics

ndash 6 became worse 41 had interval appendectomy

ndash 49 received no further treatment

ndash 57 no recurrence

ndash 43 had recurrence within one month to 2 years

ndash Presence of appendicolith 72 rate of recurrence vs 26 in those without appendicolith

Ein SH1 Langer JC Daneman A Nonoperative management of pediatric ruptured appendix

with inflammatory mass or abscess presence of an appendicolith predicts recurrent

appendicitis J Pediatr Surg 2005 Oct40(10)1612-5

Approach to interval appendectomy

More strongly recommended if fecalith present

Observation is a reasonable option

Best choice for an individual patient depends on their anxiety and parental anxiety

Immediate operation for ruptured appendicitis perhaps a better

option

Meta-analysis operative vs non-operative management of pediatric

ruptured appendicitis

2 RCTs identified

Total of 171 pediatric patients

Compared early vs interval appendectomy

Early appendectomy reduced incidence of adverse event

Ruptured appendicitis may have abscess or phlegmon

Early surgery was more strongly favored when there was no abscess at

time of presentation

Decreased antibiotic duration length of stay and total charges for

abscess and no abscess groups

You are seeing a 7 year old with constipation A

fecalith was seen on abdominal X ray during recent

ED visit How would you counsel the family

A They should be referred to surgery for appendectomy

B Their child is at higher risk for appendicitis so they should be aware of this in case he develops symptoms

C It is uncertain whether this child is at higher risk for appendicitis

D A course of Miralax may help wash out the fecalith

Take home points

Uncomplicated appendicitis

ndash Lap appy is still standard of care

ndash Non-operative management may be an acceptable option but not enough is known about long-term risk of recurrent appendicitis

Complicated appendicitis

ndash Can be managed with immediate operation delayed appendectomy or no appendectomy

ndash Immediate operation is probably more efficient and less stressful for patients and parents

Thank you very much

Questions

Page 51: New approaches for evaluation and treatment of appendicitis · Ultrasound or CT scan showing appendicitis with an appendix ≤ 1.1 centimeter thick and no evidence of abscess or fecalith

How to counsel families of children with ruptured appendicitis

If kids tolerate a diet pain resolves fever resolves oral antibiotics and home

Plan interval appy 6-8 weeks later

Non-operative management may not work and surgery may be needed (Failure rate 20)

At home they should watch for signs of persistentrecurrent appendicitis

High-anxiety time for patients and families

A 10-year-old boy comes to see you in the office after recent

hospitalization for perforated appendicitis He has 3 more days of

antibiotics left His mom is worried because appetite is poor and his

energy level is low On exam he has diffuse lower abdominal

tenderness You recommend

A Extending course of oral antibiotics

B CT scan to evaluate for persistent or

recurrent appendicitis

C CT scan to evaluate for intra-abdominal

abscess

D Follow-up with surgeon

Is interval appendectomy necessary

In adults many surgeons do not do this operation

In kids data are limited

ndash 2-year follow-up of 96 patients

ndash Perforated appendicitis treated non-operatively with antibiotics

ndash 6 became worse 41 had interval appendectomy

ndash 49 received no further treatment

ndash 57 no recurrence

ndash 43 had recurrence within one month to 2 years

ndash Presence of appendicolith 72 rate of recurrence vs 26 in those without appendicolith

Ein SH1 Langer JC Daneman A Nonoperative management of pediatric ruptured appendix

with inflammatory mass or abscess presence of an appendicolith predicts recurrent

appendicitis J Pediatr Surg 2005 Oct40(10)1612-5

Approach to interval appendectomy

More strongly recommended if fecalith present

Observation is a reasonable option

Best choice for an individual patient depends on their anxiety and parental anxiety

Immediate operation for ruptured appendicitis perhaps a better

option

Meta-analysis operative vs non-operative management of pediatric

ruptured appendicitis

2 RCTs identified

Total of 171 pediatric patients

Compared early vs interval appendectomy

Early appendectomy reduced incidence of adverse event

Ruptured appendicitis may have abscess or phlegmon

Early surgery was more strongly favored when there was no abscess at

time of presentation

Decreased antibiotic duration length of stay and total charges for

abscess and no abscess groups

You are seeing a 7 year old with constipation A

fecalith was seen on abdominal X ray during recent

ED visit How would you counsel the family

A They should be referred to surgery for appendectomy

B Their child is at higher risk for appendicitis so they should be aware of this in case he develops symptoms

C It is uncertain whether this child is at higher risk for appendicitis

D A course of Miralax may help wash out the fecalith

Take home points

Uncomplicated appendicitis

ndash Lap appy is still standard of care

ndash Non-operative management may be an acceptable option but not enough is known about long-term risk of recurrent appendicitis

Complicated appendicitis

ndash Can be managed with immediate operation delayed appendectomy or no appendectomy

ndash Immediate operation is probably more efficient and less stressful for patients and parents

Thank you very much

Questions

Page 52: New approaches for evaluation and treatment of appendicitis · Ultrasound or CT scan showing appendicitis with an appendix ≤ 1.1 centimeter thick and no evidence of abscess or fecalith

A 10-year-old boy comes to see you in the office after recent

hospitalization for perforated appendicitis He has 3 more days of

antibiotics left His mom is worried because appetite is poor and his

energy level is low On exam he has diffuse lower abdominal

tenderness You recommend

A Extending course of oral antibiotics

B CT scan to evaluate for persistent or

recurrent appendicitis

C CT scan to evaluate for intra-abdominal

abscess

D Follow-up with surgeon

Is interval appendectomy necessary

In adults many surgeons do not do this operation

In kids data are limited

ndash 2-year follow-up of 96 patients

ndash Perforated appendicitis treated non-operatively with antibiotics

ndash 6 became worse 41 had interval appendectomy

ndash 49 received no further treatment

ndash 57 no recurrence

ndash 43 had recurrence within one month to 2 years

ndash Presence of appendicolith 72 rate of recurrence vs 26 in those without appendicolith

Ein SH1 Langer JC Daneman A Nonoperative management of pediatric ruptured appendix

with inflammatory mass or abscess presence of an appendicolith predicts recurrent

appendicitis J Pediatr Surg 2005 Oct40(10)1612-5

Approach to interval appendectomy

More strongly recommended if fecalith present

Observation is a reasonable option

Best choice for an individual patient depends on their anxiety and parental anxiety

Immediate operation for ruptured appendicitis perhaps a better

option

Meta-analysis operative vs non-operative management of pediatric

ruptured appendicitis

2 RCTs identified

Total of 171 pediatric patients

Compared early vs interval appendectomy

Early appendectomy reduced incidence of adverse event

Ruptured appendicitis may have abscess or phlegmon

Early surgery was more strongly favored when there was no abscess at

time of presentation

Decreased antibiotic duration length of stay and total charges for

abscess and no abscess groups

You are seeing a 7 year old with constipation A

fecalith was seen on abdominal X ray during recent

ED visit How would you counsel the family

A They should be referred to surgery for appendectomy

B Their child is at higher risk for appendicitis so they should be aware of this in case he develops symptoms

C It is uncertain whether this child is at higher risk for appendicitis

D A course of Miralax may help wash out the fecalith

Take home points

Uncomplicated appendicitis

ndash Lap appy is still standard of care

ndash Non-operative management may be an acceptable option but not enough is known about long-term risk of recurrent appendicitis

Complicated appendicitis

ndash Can be managed with immediate operation delayed appendectomy or no appendectomy

ndash Immediate operation is probably more efficient and less stressful for patients and parents

Thank you very much

Questions

Page 53: New approaches for evaluation and treatment of appendicitis · Ultrasound or CT scan showing appendicitis with an appendix ≤ 1.1 centimeter thick and no evidence of abscess or fecalith

Is interval appendectomy necessary

In adults many surgeons do not do this operation

In kids data are limited

ndash 2-year follow-up of 96 patients

ndash Perforated appendicitis treated non-operatively with antibiotics

ndash 6 became worse 41 had interval appendectomy

ndash 49 received no further treatment

ndash 57 no recurrence

ndash 43 had recurrence within one month to 2 years

ndash Presence of appendicolith 72 rate of recurrence vs 26 in those without appendicolith

Ein SH1 Langer JC Daneman A Nonoperative management of pediatric ruptured appendix

with inflammatory mass or abscess presence of an appendicolith predicts recurrent

appendicitis J Pediatr Surg 2005 Oct40(10)1612-5

Approach to interval appendectomy

More strongly recommended if fecalith present

Observation is a reasonable option

Best choice for an individual patient depends on their anxiety and parental anxiety

Immediate operation for ruptured appendicitis perhaps a better

option

Meta-analysis operative vs non-operative management of pediatric

ruptured appendicitis

2 RCTs identified

Total of 171 pediatric patients

Compared early vs interval appendectomy

Early appendectomy reduced incidence of adverse event

Ruptured appendicitis may have abscess or phlegmon

Early surgery was more strongly favored when there was no abscess at

time of presentation

Decreased antibiotic duration length of stay and total charges for

abscess and no abscess groups

You are seeing a 7 year old with constipation A

fecalith was seen on abdominal X ray during recent

ED visit How would you counsel the family

A They should be referred to surgery for appendectomy

B Their child is at higher risk for appendicitis so they should be aware of this in case he develops symptoms

C It is uncertain whether this child is at higher risk for appendicitis

D A course of Miralax may help wash out the fecalith

Take home points

Uncomplicated appendicitis

ndash Lap appy is still standard of care

ndash Non-operative management may be an acceptable option but not enough is known about long-term risk of recurrent appendicitis

Complicated appendicitis

ndash Can be managed with immediate operation delayed appendectomy or no appendectomy

ndash Immediate operation is probably more efficient and less stressful for patients and parents

Thank you very much

Questions

Page 54: New approaches for evaluation and treatment of appendicitis · Ultrasound or CT scan showing appendicitis with an appendix ≤ 1.1 centimeter thick and no evidence of abscess or fecalith

Approach to interval appendectomy

More strongly recommended if fecalith present

Observation is a reasonable option

Best choice for an individual patient depends on their anxiety and parental anxiety

Immediate operation for ruptured appendicitis perhaps a better

option

Meta-analysis operative vs non-operative management of pediatric

ruptured appendicitis

2 RCTs identified

Total of 171 pediatric patients

Compared early vs interval appendectomy

Early appendectomy reduced incidence of adverse event

Ruptured appendicitis may have abscess or phlegmon

Early surgery was more strongly favored when there was no abscess at

time of presentation

Decreased antibiotic duration length of stay and total charges for

abscess and no abscess groups

You are seeing a 7 year old with constipation A

fecalith was seen on abdominal X ray during recent

ED visit How would you counsel the family

A They should be referred to surgery for appendectomy

B Their child is at higher risk for appendicitis so they should be aware of this in case he develops symptoms

C It is uncertain whether this child is at higher risk for appendicitis

D A course of Miralax may help wash out the fecalith

Take home points

Uncomplicated appendicitis

ndash Lap appy is still standard of care

ndash Non-operative management may be an acceptable option but not enough is known about long-term risk of recurrent appendicitis

Complicated appendicitis

ndash Can be managed with immediate operation delayed appendectomy or no appendectomy

ndash Immediate operation is probably more efficient and less stressful for patients and parents

Thank you very much

Questions

Page 55: New approaches for evaluation and treatment of appendicitis · Ultrasound or CT scan showing appendicitis with an appendix ≤ 1.1 centimeter thick and no evidence of abscess or fecalith

Immediate operation for ruptured appendicitis perhaps a better

option

Meta-analysis operative vs non-operative management of pediatric

ruptured appendicitis

2 RCTs identified

Total of 171 pediatric patients

Compared early vs interval appendectomy

Early appendectomy reduced incidence of adverse event

Ruptured appendicitis may have abscess or phlegmon

Early surgery was more strongly favored when there was no abscess at

time of presentation

Decreased antibiotic duration length of stay and total charges for

abscess and no abscess groups

You are seeing a 7 year old with constipation A

fecalith was seen on abdominal X ray during recent

ED visit How would you counsel the family

A They should be referred to surgery for appendectomy

B Their child is at higher risk for appendicitis so they should be aware of this in case he develops symptoms

C It is uncertain whether this child is at higher risk for appendicitis

D A course of Miralax may help wash out the fecalith

Take home points

Uncomplicated appendicitis

ndash Lap appy is still standard of care

ndash Non-operative management may be an acceptable option but not enough is known about long-term risk of recurrent appendicitis

Complicated appendicitis

ndash Can be managed with immediate operation delayed appendectomy or no appendectomy

ndash Immediate operation is probably more efficient and less stressful for patients and parents

Thank you very much

Questions

Page 56: New approaches for evaluation and treatment of appendicitis · Ultrasound or CT scan showing appendicitis with an appendix ≤ 1.1 centimeter thick and no evidence of abscess or fecalith

Meta-analysis operative vs non-operative management of pediatric

ruptured appendicitis

2 RCTs identified

Total of 171 pediatric patients

Compared early vs interval appendectomy

Early appendectomy reduced incidence of adverse event

Ruptured appendicitis may have abscess or phlegmon

Early surgery was more strongly favored when there was no abscess at

time of presentation

Decreased antibiotic duration length of stay and total charges for

abscess and no abscess groups

You are seeing a 7 year old with constipation A

fecalith was seen on abdominal X ray during recent

ED visit How would you counsel the family

A They should be referred to surgery for appendectomy

B Their child is at higher risk for appendicitis so they should be aware of this in case he develops symptoms

C It is uncertain whether this child is at higher risk for appendicitis

D A course of Miralax may help wash out the fecalith

Take home points

Uncomplicated appendicitis

ndash Lap appy is still standard of care

ndash Non-operative management may be an acceptable option but not enough is known about long-term risk of recurrent appendicitis

Complicated appendicitis

ndash Can be managed with immediate operation delayed appendectomy or no appendectomy

ndash Immediate operation is probably more efficient and less stressful for patients and parents

Thank you very much

Questions

Page 57: New approaches for evaluation and treatment of appendicitis · Ultrasound or CT scan showing appendicitis with an appendix ≤ 1.1 centimeter thick and no evidence of abscess or fecalith

Early appendectomy reduced incidence of adverse event

Ruptured appendicitis may have abscess or phlegmon

Early surgery was more strongly favored when there was no abscess at

time of presentation

Decreased antibiotic duration length of stay and total charges for

abscess and no abscess groups

You are seeing a 7 year old with constipation A

fecalith was seen on abdominal X ray during recent

ED visit How would you counsel the family

A They should be referred to surgery for appendectomy

B Their child is at higher risk for appendicitis so they should be aware of this in case he develops symptoms

C It is uncertain whether this child is at higher risk for appendicitis

D A course of Miralax may help wash out the fecalith

Take home points

Uncomplicated appendicitis

ndash Lap appy is still standard of care

ndash Non-operative management may be an acceptable option but not enough is known about long-term risk of recurrent appendicitis

Complicated appendicitis

ndash Can be managed with immediate operation delayed appendectomy or no appendectomy

ndash Immediate operation is probably more efficient and less stressful for patients and parents

Thank you very much

Questions

Page 58: New approaches for evaluation and treatment of appendicitis · Ultrasound or CT scan showing appendicitis with an appendix ≤ 1.1 centimeter thick and no evidence of abscess or fecalith

Ruptured appendicitis may have abscess or phlegmon

Early surgery was more strongly favored when there was no abscess at

time of presentation

Decreased antibiotic duration length of stay and total charges for

abscess and no abscess groups

You are seeing a 7 year old with constipation A

fecalith was seen on abdominal X ray during recent

ED visit How would you counsel the family

A They should be referred to surgery for appendectomy

B Their child is at higher risk for appendicitis so they should be aware of this in case he develops symptoms

C It is uncertain whether this child is at higher risk for appendicitis

D A course of Miralax may help wash out the fecalith

Take home points

Uncomplicated appendicitis

ndash Lap appy is still standard of care

ndash Non-operative management may be an acceptable option but not enough is known about long-term risk of recurrent appendicitis

Complicated appendicitis

ndash Can be managed with immediate operation delayed appendectomy or no appendectomy

ndash Immediate operation is probably more efficient and less stressful for patients and parents

Thank you very much

Questions

Page 59: New approaches for evaluation and treatment of appendicitis · Ultrasound or CT scan showing appendicitis with an appendix ≤ 1.1 centimeter thick and no evidence of abscess or fecalith

Early surgery was more strongly favored when there was no abscess at

time of presentation

Decreased antibiotic duration length of stay and total charges for

abscess and no abscess groups

You are seeing a 7 year old with constipation A

fecalith was seen on abdominal X ray during recent

ED visit How would you counsel the family

A They should be referred to surgery for appendectomy

B Their child is at higher risk for appendicitis so they should be aware of this in case he develops symptoms

C It is uncertain whether this child is at higher risk for appendicitis

D A course of Miralax may help wash out the fecalith

Take home points

Uncomplicated appendicitis

ndash Lap appy is still standard of care

ndash Non-operative management may be an acceptable option but not enough is known about long-term risk of recurrent appendicitis

Complicated appendicitis

ndash Can be managed with immediate operation delayed appendectomy or no appendectomy

ndash Immediate operation is probably more efficient and less stressful for patients and parents

Thank you very much

Questions

Page 60: New approaches for evaluation and treatment of appendicitis · Ultrasound or CT scan showing appendicitis with an appendix ≤ 1.1 centimeter thick and no evidence of abscess or fecalith

Decreased antibiotic duration length of stay and total charges for

abscess and no abscess groups

You are seeing a 7 year old with constipation A

fecalith was seen on abdominal X ray during recent

ED visit How would you counsel the family

A They should be referred to surgery for appendectomy

B Their child is at higher risk for appendicitis so they should be aware of this in case he develops symptoms

C It is uncertain whether this child is at higher risk for appendicitis

D A course of Miralax may help wash out the fecalith

Take home points

Uncomplicated appendicitis

ndash Lap appy is still standard of care

ndash Non-operative management may be an acceptable option but not enough is known about long-term risk of recurrent appendicitis

Complicated appendicitis

ndash Can be managed with immediate operation delayed appendectomy or no appendectomy

ndash Immediate operation is probably more efficient and less stressful for patients and parents

Thank you very much

Questions

Page 61: New approaches for evaluation and treatment of appendicitis · Ultrasound or CT scan showing appendicitis with an appendix ≤ 1.1 centimeter thick and no evidence of abscess or fecalith

You are seeing a 7 year old with constipation A

fecalith was seen on abdominal X ray during recent

ED visit How would you counsel the family

A They should be referred to surgery for appendectomy

B Their child is at higher risk for appendicitis so they should be aware of this in case he develops symptoms

C It is uncertain whether this child is at higher risk for appendicitis

D A course of Miralax may help wash out the fecalith

Take home points

Uncomplicated appendicitis

ndash Lap appy is still standard of care

ndash Non-operative management may be an acceptable option but not enough is known about long-term risk of recurrent appendicitis

Complicated appendicitis

ndash Can be managed with immediate operation delayed appendectomy or no appendectomy

ndash Immediate operation is probably more efficient and less stressful for patients and parents

Thank you very much

Questions

Page 62: New approaches for evaluation and treatment of appendicitis · Ultrasound or CT scan showing appendicitis with an appendix ≤ 1.1 centimeter thick and no evidence of abscess or fecalith

Take home points

Uncomplicated appendicitis

ndash Lap appy is still standard of care

ndash Non-operative management may be an acceptable option but not enough is known about long-term risk of recurrent appendicitis

Complicated appendicitis

ndash Can be managed with immediate operation delayed appendectomy or no appendectomy

ndash Immediate operation is probably more efficient and less stressful for patients and parents

Thank you very much

Questions

Page 63: New approaches for evaluation and treatment of appendicitis · Ultrasound or CT scan showing appendicitis with an appendix ≤ 1.1 centimeter thick and no evidence of abscess or fecalith

Thank you very much

Questions